Sunteți pe pagina 1din 125

Online Instructors Manual

to accompany
Quality Management
Second Edition
Donna C. S. Summers
University of Dayton
Upper Saddle River, New Jersey
Columbus, Ohio
__________________________________________________________________________________
Copyright 200 !y "earson Education# Inc.# $pper Saddle %i&er# 'e( )ersey 0*+,-.
Pearson Prentice Hall !ll ri"hts reserved Printed in the United States o# !merica $his
publication is protected by Copyri"ht and permission should be obtained #rom the publisher prior
to any prohibited reproduction, stora"e in a retrieval system, or transmission in any #orm or by
any means, electronic, mechanical, photocopyin", recordin", or li%ewise &or in#ormation
re"ardin" permission's(, write to) Ri"hts and Permissions *epartment
"earson "rentice .all/ is a trademar% o# Pearson +ducation, ,nc
"earson
0
is a re"istered trademar% o# Pearson plc
"rentice .all
0
is a re"istered trademar% o# Pearson +ducation, ,nc
,nstructors o# classes usin" *onna Summers- Quality Management, Second Edition, may
reproduce material #rom the instructor-s manual #or classroom use
./ 0 1 2 3 4 5 6 7 .
,S8N9.6) 0219/9.694//4..91
,S8N9./) /9.694//4..93
6
$able o# Contents
!nswers to Selected Chapter :uestions
Chapter .7
Chapter 75
Chapter 62
Chapter 5.7
Chapter 4.4
Chapter 3.1
Chapter 277
Chapter 174
Chapter 072
Chapter ./65
Chapter ..62
Chapter .75/
Chapter .656
Chapter .544
Chapter .437
!ppendi; 434
$est 8an%
Chapter .2/
Chapter 727
Chapter 625
Chapter 520
Chapter 416
Chapter 311
Chapter 207
Chapter 102
Chapter 000
Chapter ././7
Chapter .../5
Chapter .7./2
Chapter .6../
Chapter .5..4
Chapter .4..1
1ns(ers to Selected Chapter Questions
Chapter 2 Organi3ational E44ecti&eness Questions
. <hy would an or"ani=ation want to be e##ective>
!n e##ective or"ani=ation is able to produce more with its e;istin" resources throu"h an
improved customer #ocus and streamlined wor% processes <ith its increased awareness o# its
internal and e;ternal customers, there is a "reater #ocus on what really needs to be accomplished
in order to meet their customers- needs and e;pectations, there#ore increasin" their pro#itability
throu"h increased customer retention 8ein" able to meet customer e;pectations the #irst time
and every time will enable the or"ani=ation to increase its mar%et share as new customers see%
them out Since an e##ective or"ani=ation has #ocused and streamlined its wor% processes they
will bene#it #rom lower costs because o# reduced waste and rewor% One o# the ma?or savin"s
that occurs is #ewer customer complaints and warranty claims @ore satis#ied customers results
in "reater mar%et share 8ecause o# improved communication and teamwor%, e##ective
or"ani=ations have "ood mana"ement9employee relations !s problems are solved and the
or"ani=ation be"ins to run more smoothly, employee involvement and satis#action will increase,
which will lower turnover and absenteeism
7 How would you describe or de#ine or"ani=ational e##ectiveness>
+##ective or"ani=ations #ocus on the %ey processes that provide the or"ani=ation-s customers
with products or services !n or"ani=ation is a compilation o# a wide variety o# activities !n
or"ani=ation needs to be loo%ed at #rom all perspectivesA manu#acturin", mar%etin", in#ormation
technolo"y, research and development must wor% well to"ether in an e##ective or"ani=ation $he
#ocus o# these #unctions must be on the elements o# the or"ani=ation that ma%e the "reatest
contribution to the or"ani=ation-s success, however that is measured +##ective or"ani=ations
adhere to a customer9centered philosophy that includes payin" attention to or"ani=ational,
strate"ic, environmental, and people #actors
+##ective or"ani=ations have developed an or"ani=ational philosophy utili=in" a Buality
mana"ement system that creates a customer #ocus and "enerates customer satis#action +##ective
or"ani=ations implement or"ani=ational leadership, create strate"ic plans, develop and mana"e
the or"ani=ation-s human element, measure or"ani=ational success, improve processes, utili=e
Buality tools, mana"e pro?ects, and trac% %ey business results
6 <ho is in char"e o# creatin" or"ani=ational e##ectiveness>
Or"ani=ational leadership is responsible #or or"ani=ational e##ectiveness Ceadership, %nowin"
the or"ani=ation-s current levels o# per#ormance, creates strate"ic plans #or the #uture Ceaders in
an e##ective or"ani=ation develop a customer9oriented approach, studyin" how their product or
7
service is used #rom the moment a customer #irst becomes aware o# the product or service until
the moment the product is disposed o# or the service is complete Ceaders in e##ective
or"ani=ations see% to optimi=e business processes by ta%in" a systems approach, emphasi=in" the
improvement o# systems and processes that enable a company to provide products or services #or
their customer

5 ,# you initiated chan"es to improve or"ani=ational e##ectiveness, how would you veri#y the
improvement>
8y as%in" DHow do you %now>E @easures o# per#ormance #ocused on %ey issues related to the
customer enable an or"ani=ation to ?ud"e whether or not it is meetin" its "oals and ob?ectives
4 How would you measure the relative overall or"ani=ational e##ectiveness o# any or"ani=ation>
8y studyin" the answers to the #ollowin" Buestions)
. <hat is the mission o# the or"ani=ation>
7 <hat business is the or"ani=ation really in>
6 <hat are the primary products or services the or"ani=ation provides to their
customers>
5 <ho are their customers>
4 <hat do their customers e;pect and need>
3 How does the or"ani=ation %now what their needs and e;pectations are>
2 How well does the or"ani=ation meet the needs and e;pectations o# their customers>
1 How does the or"ani=ation %now how well it-s doin"> <hat is the proo#> <hat are
the indicators>
0 *o mana"ement-s strate"ies and actions support the business and support the
or"ani=ation in meetin" the customer-s needs and e;pectations>
./ *o the employees %now how the wor% they do speci#ically bene#its the ultimate,
e;ternal customer>
.. <hat improvements have been made based on the answers to these Buestions>
.7 <hat is mana"ement doin" to support improvement e##orts>
3 <hat drove PCC, #rom +;ample .., to ma%e chan"es>
<hen PCC ,nc be"an operations three decades a"o, their ?ob shop speciali=ed in machinin"
lar"e #or"in"s into #inished products $heir ori"inal manu#acturin" methods resulted in
si"ni#icant scrap and rewor% costs $he type o# inspection scheme used by PCC ,nc was very
costly because it involved not only the costs o# the de#ective aspect o# the parts, but also the
labor cost o# per#ormin" wor% on a de#ective part by later wor%stations
8y the mid9.01/s, the companies they did business with be"an reBuirin" statistical process
control in#ormation $hey reali=ed that the prevention o# de#ects could ma%e a si"ni#icant impact
on their pro#it per#ormance 8y trac%in" the critical part dimensions, those monitorin" the
processes were able to identi#y chan"es that a##ected the Buality o# the product and ad?ust the
process accordin"ly $hey sou"ht additional ways to reduce the variation present in the process
6
that prevented them #rom producin" parts as close to the nominal dimension as possible $hus,
emphasis shi#ted away #rom inspectin" Buality into the parts and toward ma%in" process
improvements by desi"nin" and machinin" Buality into the product
Chapter 2 Organi3ational "hilosophy Questions
. <hy is an or"ani=ational philosophy #ocusin" on deli"htin" customers %ey to or"ani=ational
success>
Customers are the reason an or"ani=ation is in business !n or"ani=ation is always see%in" to
%eep the customers they have and "ain new customers ,n order to do this consistently, e##ective
or"ani=ations must #ocus on creatin" success#ul, satis#ied customers each time an interaction
occurs
6 *escribe, in your own words, the two types o# variation that Shewhart identi#ied
Controlled variation 'common causes( is variation present in a process due to the very nature o#
the process $his type o# variation can be removed #rom the process only by chan"in" the
process &or e;ample, consider a person who has driven the same route to wor% do=ens o# times
and determined that it ta%es about 7/ minutes to "et #rom home to wor% re"ardless o# minor
chan"es in weather or tra##ic conditions ,# this is the case, then the only way the person can
improve upon this time is to chan"e the process by #indin" a new route
Uncontrolled variation 'special or assi"nable causes( comes #rom sources e;ternal to the process
$his type o# variation is not normally part o# the process ,t can be identi#ied and isolated as the
cause o# the chan"e in the behavior o# the process &or instance, the commuter described above
would e;perience uncontrolled variation i# a ma?or tra##ic accident stopped tra##ic or a bli==ard
made travelin" nearly impossible
3 How do *r *emin"-s #ourteen points interact with each other>
*emin"-s #ourteen points describe an overall chan"e in mana"ement and or"ani=ational
philosophy +ach o# the points #ocuses on a wea%ness in companies that needs to be overcome in
order to improve or"ani=ational e##ectiveness $he points support overall systems improvement,
the creation o# non9#aulty systems
2 How do the steerin"Fdia"nostic arms o# *r Juran-s pro"ram wor% to"ether>
,n the pro?ect9by9pro?ect implementation procedure '$able 77(, pro?ect teams are set up to
investi"ate and solve speci#ic problems $o "uide the pro?ect teams, a steerin" committee e;ists
$he steerin" committee serves several purposes) to ensure emphasis on the company-s "oals, to
"rant authority to dia"nose and investi"ate problems, and to enact chan"es *r Juran-s pro?ect
teams are encoura"ed to use a systematic approach to problem9solvin" Group members use a
variety o# investi"ative Buality tools to clari#y the symptoms and locate the true cause's( o# the
problem <hen the cause is determined, #indin" a solution becomes a process o# proposin"
remedies, testin" them, and institutin" the remedy that most e##ectively solves the problem
5
Controllin" the process once chan"es have been made is important to ensure that the e##orts have
not been wasted
1 ,n your own words, describe the di##erence between bi" : and little B Use e;amples #rom
your own e;periences to bac% up your description
Juran-s de#inition o# Buality "oes beyond the immediate product or moment o# service $o *r
Juran, Buality is a concept that needs to be #ound in all aspects o# a business $o more clearly
communicate this concept, *r Juran contrasts bi" : and little B to show the broad applicability
o# Buality concepts Note that bi" : e;tends to all aspects o# any or"ani=ation, re"ardless o#
type
0 People tend to ma%e #ive erroneous assumptions about Buality <hat are two o# these
assumptions and how would you ar"ue a"ainst them>
. :uality means "oodness, or lu;ury, or shininess or wei"ht, which ma%es Buality a relative
term
7 :uality is intan"ible and there#ore not measurable ,# ?ud"ed in terms o# D"oodness,E then
Buality is intan"ibleA however, Buality is measurable by the costs o# doin" thin"s wron"
6 $here e;ists Dan economics o# BualityE
5 <or%ers are to blame #or bein" the cause o# Buality problems <ithout the proper tools,
eBuipment, raw materials, and trainin", wor%ers cannot produce Buality products or services
@ana"ement must ensure that the necessary items are available to allow wor%ers to per#orm
their ?obs well
4 :uality ori"inates in the Buality department !ccordin" to Crosby, the Buality department-s
responsibilities revolve around educatin" and assistin" other departments in monitorin" and
improvin" Buality
.. How are the teachin"s o# each o# the people in this chapter similar> <here do they a"ree>
*emin", Juran, and Crosby encoura"e or"ani=ations to)
9determine who their customers are
9determine the %ey critical success #actors #or meetin" their customers- needs,
reBuirements, and e;pectations
9establish e##ective processes that enable them to provide products and services that meet
their customers- needs, reBuirements, and e;pectations
9#ocus on process measurement and improvement
9provide the mana"ement involvement and commitment reBuired #or or"ani=ational
success
.7 How are the teachin"s o# each o# the people in this chapter di##erent> <here do they
disa"ree>
*emin", Juran, and Crosby discusses di##erent ways o# reachin" the "oals *emin" and Juran
#ocus primarily on leadership e##orts and mana"ement commitment and involvement Juran has
4
his ?ourney #rom symptom to cause and *emin" has his .5 points Crosby #ocuses on pursuin"
=ero de#ects throu"h process improvement Shewhart concentrated on Buality improvement
throu"h reduction in variation
$heir de#initions o# Buality are di##erent, too &ei"enbaum-s de#inition is more complete 'see
te;t( *emin" #ocuses on non9#aulty systems Crosby #ocuses on con#ormance to reBuirements
Crosby-s is the narrowest Buality de#inition
.6 8rie#ly summari=e the concept *r $a"uchi is tryin" to "et across with his loss #unction
He uses his concept and dia"ram to point out that any deviation #rom the tar"et speci#ication
results in loss Or"ani=ations should strive to determine and then meet their customers- tar"et
reBuirements
.5 *escribe *r $a"uchi-s loss #unction versus the traditional approach to Buality
$he traditional approach to Buality emphasi=es con#ormance to reBuirements, usually stated as a
speci#ication with HF9 limits $he $a"uchi Coss &unction points out that any deviation #rom the
tar"et speci#ication results in loss
3
Chapter 5 Quality Systems Questions
. *escribe what is meant by the term DBuality systemE
<ithin a Buality mana"ement system, the necessary in"redients e;ist to enable the or"ani=ation-s
employees to identi#y, desi"n, develop, produce, deliver, and support products or services that
the customer wants
7 <hy is a Buality system critical to providin" a Buality product or service>
+##ective Buality mana"ement systems are dynamic, able to adapt and chan"e to meet the needs,
reBuirements, and e;pectations o# its customers :uality systems provide "uidance #or
establishin" an or"ani=ation-s processes #or maintainin" records, improvin" processes and
systems, and meetin" customer e;pectations ! systems approach enables an or"ani=ation to
"ain and retain customers $hese systems enable them to improve overall or"ani=ational
e##ectiveness and pro#itability
6 <hat attributes would you e;pect to be present in a company that has a sound Buality system>
Customer #ocus
Good record9%eepin"
$imely deliveries
Good process mana"ement and control
Sound human resources mana"ement
Sound business results
Sound measurement systems
Good pro?ect mana"ement practices
Statistical process control
Continuous improvementFproblem9solvin" e##orts
Stron" leadership
,nvolvement o# people
Systems approach to mana"ement
&actual approach to decision ma%in"
@utually bene#icial supplier relationships
5 *escribe why ,SO 0/// is important #or an e##ective or"ani=ation
$he purpose o# the ,SO standards is to #acilitate the multinational e;chan"e o# products and
services by providin" a clear set o# Buality system reBuirements Companies competin" on a
"lobal basis #ind it necessary to adopt and adhere to these standards $he standards provide a
baseline a"ainst which an or"ani=ation-s Buality system can be ?ud"ed $his baseline has at its
#oundation the achievement o# customer satis#action throu"h multidisciplinary participation in
Buality9improvement e##orts, documentation o# systems and procedures, and other basic
structural elements necessary to Buality systems $he "eneric nature o# the standards allows
interested companies to determine the speci#ics o# how the standards apply to its or"ani=ation
@any companies use ,SO 0/// as the #oundation #or their continuous improvement e##orts
2
4 <hy is record %eepin" important> <hat types o# records do e##ective or"ani=ations %eep>
,n ,SO 0/// a "reat deal o# emphasis is placed on the need #or e;cellent record %eepin" ,n most
cases, since the product has le#t the manu#acturin" #acility or the service has been per#ormed,
only clearly %ept records can serve as evidence o# product or service Buality Sloppy or poorly
maintained records "ive the impression o# poor Buality Hi"h9Buality records are easy to
retrieve, le"ible, appropriate, accurate, and complete Necessary records may ori"inate internally
or be produced e;ternally Customer or technical speci#ications and re"ulatory reBuirements are
considered e;ternal records +;amples o# internally produced records include #orms, reports,
drawin"s, meetin" minutes, problem9solvin" documentation, and process control charts
3 <hy is it important to document wor% methods and procedures>
*ocumentation describes how wor% must be accomplished Structured correctly, documentation
will apply to a variety o# situations, not ?ust speci#ic products *ocuments serve as "uides and
ensure that wor% is per#ormed consistently Sound documentation can be used to determine and
correct the causes o# poor Buality *ocumentation de#ines e;istin" wor% methods and provides a
#oundation #or improvement ! hi"h9Buality documentation control system will contain records
that are easily identi#ied and used in the decision9ma%in" process
2 *escribe the ,SO 0/// re"istration process
$he #i"ure in the te;t shows the #low o# a typical re"istration process
1 <hat are the bene#its o# certi#ication>
$he bene#its an or"ani=ation will receive by usin" the reBuirements as a "uide to improve their
processes include) more consistent Buality, the percenta"e o# Ddone ri"ht the #irst timeE ?obs
increases, enhanced procedures, up"raded record9%eepin", and the removal o# redundant
operations also dramatically improve an or"ani=ation-s e##ectiveness ,SO 0/// standards
#acilitate international trade
./ *escribe ,SO .5/// to someone who has not heard o# it
$he overall ob?ective o# the ,SO .5/// +nvironmental @ana"ement Standard is to encoura"e
environmental protection and the prevention o# pollution while ta%in" into account the economic
needs o# society $he standards can be #ollowed by any or"ani=ation interested in limitin" its
ne"ative impact on the environment ! company with an environmental mana"ement system
li%e ,SO .5/// is less li%ely to have environmental problems O#ten, #irms #ollowin" ,SO .5///
incur si"ni#icant savin"s throu"h better overall resource mana"ement and waste reduction
,SO .5/// is divided into two main classi#ications, Or"ani=ationFProcess9Oriented Standards
and Product9Oriented Standards ! company complyin" with these standards is monitorin" its
processes and products to determine their e##ect on the environment <ithin the two
classi#ications, si; topic areas are covered) +nvironmental @ana"ement Systems, +nvironmental
1
Per#ormance +valuation, +nvironmental !uditin", Ci#e9Cycle !ssessment, +nvironmental
Cabelin", and +nvironmental !spects in Product Standards $he ,SO .5/// series o# standards
enables a company to improve environmental mana"ement voluntarily $he standards do not
establish product or per#ormance standards, establish mandates #or emissions or pollutant levels,
or speci#y test methods $he standards do not e;pand upon e;istin" "overnment re"ulations
,SO .5/// is a "uide #or environmentally conscious companies wishin" to lessen their impact on
the environment
.. <hat do Si; Si"ma pro?ects #ocus on> <hy>
+ssentially, Si; Si"ma is about results, enhancin" pro#itability throu"h improved Buality and
e##iciency ,mprovement pro?ects are chosen based on their ability to contribute to the bottom
line on a company-s income statement by bein" connected to the strate"ic ob?ectives and "oals o#
the corporation Pro?ects that do not directly tie to customer issues or #inancial results are o#ten
di##icult to sell to mana"ement Si; Si"ma pro?ects are easy to identi#y Since the Si; Si"ma
methodolo"y see%s to reduce the variability present in processes, pro?ect teams see% out sources
o# waste, such as overtime and warranty claimsA investi"ate production bac%lo"s or areas in need
o# more capacityA and #ocus on customer and environmental issues
.7 *escribe the Si; Si"ma methodolo"y to someone who has not heard o# it
Si; Si"ma pro?ects have ei"ht essential phases) reco"ni=e, de#ine, measure, analy=e, improve,
control, standardi=e, and inte"rate $his cycle is sometimes e;pressed as *@!,C 'de#ine,
measure, analy=e, improve, and control( $he tools utili=ed durin" a pro?ect include statistical
process control techniBues, customer input, &ailure @odes and +##ects !nalysis, *esi"n o#
+;periments, process mappin", cause and e##ect dia"rams, multivariate analysis, pre9control, and
desi"n #or manu#acturability Si; Si"ma also places a heavy reliance on "raphical methods #or
analysis Statistical methods, teamwor%, and pro?ect mana"ement are %ey components o# Si;
Si"ma
.6 How does Shewhart-s Plan9*o9Study9!ct cycle compare with Si; Si"ma-s *@!,C cycle>
!s the #i"ure in the te;t shows, the "eneric steps #or Si; Si"ma pro?ect implementation are
similar to the Plan9*o9Study9!ct problem9solvin" cycle espoused by *rs *emin" and
Shewhart
.5 <hat is the @alcolm 8aldri"e National :uality !ward>
$he @alcolm 8aldri"e National :uality !ward '@8N:!( was established in .012 by the
United States Con"ress Similar to Japan-s *emin" Pri=e, it sets a national standard #or Buality
e;cellence $he award is open to companies in three areas) 8usiness, +ducation, and Health
Care ,t is mana"ed by the !merican Society #or :uality 'wwwasBor"( !s stated in the
chapter introduction, @otorola was the #irst company to win the @alcolm 8aldri"e !ward
+very year, this ri"orous award attracts several do=en applicants in each cate"ory ! "roup o#
Buali#ied e;aminers compares and contrasts each application with the criteria #or up to 6//
0
hours Only a very select #ew reach the site9visit sta"e o# the award process 8y the completion
o# the on9site visits, a company may have been e;amined #or as many as ./// hours
.4 <hy would an or"ani=ation apply #or the @alcolm 8aldri"e National :uality !ward>
$he award allows or"ani=ations to ?ud"e their overall or"ani=ation e##ectiveness a"ainst ri"id
standards $hese standards have been used by innumerable companies to clari#y their
understandin" o# their current per#ormance levels $he standards also provide clues as to what
activities an or"ani=ation must en"a"e in in order to be e##ective
.3 *escribe each o# the @alcolm 8aldri"e National :uality !ward criteria
2.0 6eadership $he criteria in Section ./ are used to e;amine senior9level mana"ement-s
commitment to and involvement in process improvement Company leaders are e;pected to
develop and sustain a customer #ocus supported by visible actions and values on their part $his
section also e;amines how the or"ani=ation addresses its responsibilities to the public and
e;hibits "ood citi=enship Subcate"ories include senior leadership direction, or"ani=ational
per#ormance review, responsibilities to the public, and support o# %ey communities
2.0 Strategic "lanning $o score well in this cate"ory, a company needs to have sound strate"ic
ob?ectives and action plans $he e;aminers also investi"ate how the company-s strate"ic
ob?ectives and action plans are deployed and pro"ress measured $hey study the or"ani=ation-s
strate"y development process, strate"ic ob?ectives, and action plan development and deployment
e##orts
5.0 Customer and Mar7et 8ocus $he third cate"ory o# the 8aldri"e !ward criteria deals with
the company-s relationship with their customers $his cate"ory #ocuses on a company-s
%nowled"e o# customer reBuirements, e;pectations, and pre#erences, as well as mar%etplace
competitiveness Reviewers also determine i# the company has put this %nowled"e to wor% in
the improvement o# their products, processes, systems, and services Success#ul e##orts in this
cate"ory lead to improved customer acBuisition, satis#action, and retention $his cate"ory
clari#ies a company-s commitment to their customers, an or"ani=ation-s customer and mar%et
%nowled"e, customer relationships, and customer satis#action determination
+.0 Measurement# 1nalysis# and 9no(ledge Management $he award reco"ni=es that
in#ormation is only use#ul when it is put to wor% to identi#y areas #or improvement $his
cate"ory investi"ates a company-s use o# in#ormation and per#ormance measurement systems to
encoura"e e;cellence Per#ormance in#ormation must be used to improve operational
competitiveness Per#ormance measurement, per#ormance analysis, data availability, hardware
and so#tware Buality, competitive comparisons, and benchmar%in" are encoura"ed
,.0 :or74orce 8ocus <ithin the wor%#orce #ocus section, reviewers #or the 8aldri"e !ward are
interested in a company-s plans and actions that enable their wor%#orce to per#orm to its #ullest
potential in ali"nment with the company-s overall strate"ic ob?ectives +mployee involvement,
education, trainin", and reco"nition are considered in this cate"ory ! company-s wor%
environment receives care#ul scrutiny in an e##ort to determine how the company has built and
./
maintains a wor% environment conducive to per#ormance e;cellence as well as personal and
or"ani=ational "rowth !lso investi"ated are wor% systems, employee education, trainin" and
development, wor% environment, and employee support and satis#action
;.0 "rocess Management <ithin this cate"ory, the company is ?ud"ed on its process
mana"ement abilities Companies must provide details on their %ey business process related to
customers, products and service delivery, desi"n, productionFdelivery, business mana"ement, and
support processes
*.0 %esults Ultimately, the purpose o# bein" in business is to stay in business $his cate"ory
e;amines a company-s per#ormance and improvement in several %ey business areas includin"
customer satis#action, product and service per#ormance, #inancial and mar%etplace per#ormance,
wor%#orce, and operational per#ormance 8enchmar%in" is encoura"ed to see how the company
compares with its competitors ,nvesti"ators study customer results, product and service results,
#inancial and mar%et results, wor%#orce results, operational results, and public responsibility and
citi=enship results
.2 <hy is the @alcolm 8aldri"e National :uality !ward the most comprehensive "uide to
or"ani=ation9wide improvement> <hat does it have that ,SO 0/// and Si; Si"ma lac%>
See $able 6.
..
Chapter + Creating a Customer 8ocus Questions
1. Why would an organization want to be effective at maintaining a customer focus?
$he current "lobal business environment is e;tremely competitive $oday-s consumers are more
than willin" to switch #rom supplier to supplier in search o# better service or availability or
courtesy or #eatures or #or any variety o# reasons $o attract and retain customers, e##ective
or"ani=ations need to #ocus on determinin" and providin" what their customers want and value
+##ective or"ani=ations survive because they tal% to customers, translate what their customers
say into appropriate actions, and ali"n their %ey business processes to support what their
customers want
2. What must an organization do to maintain a customer focus?
Or"ani=ations practicin" total Buality mana"ement principles create a customer9#ocused
mana"ement system and company culture that see%s to meet their customers- needs the #irst time
and every time +##ective or"ani=ations analy=e their customer-s needs, wants, and e;pectations,
translate them into technical speci#ics, and or"ani=e their %ey business operations accordin"ly
$hese or"ani=ations ensure that their leadership creates and implements strate"ic plans that #ocus
on what is important to their customers and mar%ets
+##ective or"ani=ations need an accurate understandin" o# what their customers e;pect $hey
also need to identi#y the "ap between their current per#ormance and what the customer reBuires i#
they are "oin" to properly tar"et improvement activities $hey reco"ni=e the importance o#
studyin" both customer value perceptions and customer satis#action Customer perceived value,
the result o# comparin" purchasin" alternatives, loo%s toward the #uture and is proactive,
allowin" a company to chan"e its #uture product or service o##erin"s to better suit its customers
Customer satis#action, comparin" past e;perience or e;pectations to the realities e;perienced, is
reactive and retrospective ,n#ormation about both can be used to help improve e;istin"
processes
3. What are the benefits of maintaining a customer focus?
Customer satis#action, or"ani=ation credibility and reputation, customer perception o# value,
success#ul customers, competitiveness, "rowth, attract and retain customers, survival, business
success
6. Describe the difference between satisfaction and perceived value.
ustomer perceived value is the result of comparing purchasing alternatives! allowing a
customer to change its future product or service offerings to better suit its customers.
ustomer satisfaction! comparing past e"perience or e"pectations to the realities
e"perienced! is reactive and retrospective. #rganizations offer product or service
features to their customers! but what the customers are really buying is the benefits
those products or services offer. $erceived value is the customer%s viewpoint of those
benefits. ustomer satisfaction! on the other hand! centers around how they felt the last
.7
time they bought a product or service from a company. &t is a comparison between
customer e"pectations and customer e"perience. $erceived value goes beyond
customer satisfaction and concentrates on future transactions. onsumers% perception
of the value they have received in the recent transaction will affect their future decision
to purchase the same thing again. &f they perceive their overall e"perience with the
product or service as valuable! they will most li'ely purchase in the future! if they do not!
they won%t. (ffective organizations realize that how the customer perceives the value of
that transaction will determine whether or not they will buy from the same organization
the ne"t time.
). Describe the principle parts of a *uality function deployment matri".
+ ,-D has two principle parts. .he horizontal component records information related to
the customer. .he customer%s wants and needs are captured by listening to the voice of
the customer and translating their wants and needs into operational goals. ustomers
are as'ed to ran' these operational goals from least to most important. .he horizontal
component also captures information about customer preferences by comparing
competitors. .he vertical component records the organization%s technical response to
these customer inputs. (ssentially! a ,-D matri" clearly shows what the customer
wants and how the organization is going to achieve those wants. .he essential steps to
a ,-D are shown in the chapter figure.
1/. 0ow is each of the principle parts of a ,-D matri" created? What does each part
hope to provide to the users?
,-D begins with the customer. 1urveys and focus groups are used to gather
information from the customers about their wants! needs! and e"pectations. 1everal
'ey areas that should be investigated include performance! features! reliability!
conformance! durability! serviceability! aesthetics! and perceived *uality. #ften!
customer information! specifically! the way they say it! must be translated into
actionable wording for the organization.
#nce this information is organized into a matri"! the customers are contacted to rate the
importance of each of the identified wants and needs. &nformation is also gathered
about how customers rate the company%s product or service against the competition.
-ollowing this input from the customers! technical re*uirements are developed. .hese
technical aspects define how the customer%s needs! wants! and e"pectations will be
met.
11. Why would a company choose to use a ,-D?
,-D allows for preventive action rather than a reactive action to customer demands.
When a company uses the ,-D format when designing a product or service! they stop
developing products and services based solely on their own interpretation of what the
.6
customer wants. &nstead! they utilize actual customer information in the design and
development process. .wo of the main benefits of ,-D are the reduced number of
engineering changes and fewer production problems. ,-D provides 'ey action items
for improving customer satisfaction and perceived value. + ,-D can enable the launch
of a new product or service to go more smoothly because customer issues and
e"pectations have been dealt with in advance. .ranslating customers% what%s into an
organization%s how%s is paramount to the success of any organization see'ing to align
their products! services! and the processes that provide them with what the customer
wants. #rganizations that ignore the relationship between what a customer wants and
how the organization is going to provide that want can never be effective.
.5
Chapter , Organi3ational 6eadership Questions
. <hy is it important to translate the vision and mission into day9to9day activities>
+##ective leaders communicate the values o# the or"ani=ation to their employees by translatin"
the vision and mission into day9to9day activities $o do this e##ectively, leaders tal% with
customers, identi#y the or"ani=ation-s critical success #actors, and share this in#ormation about
the thin"s the or"ani=ation absolutely must do well in order to attract and retain customers
Creatin" ali"nment is essentially policy deployment, the step9by9step process o# translatin" the
or"ani=ation-s vision and mission into strate"ies supported by "oals and ob?ectives that in turn
become wor% activities #or the employees Ceaders ensure that the or"ani=ation-s vision, mission
and strate"ies, "oals and daily activities remain #ocused on these critical #actors <ithout this
#ocus, the or"ani=ation will not be as e##ective at creatin" and maintainin" a customer #ocus
./ Iour boss has ?ust called a meetin" to discuss lost wor% orders and customer order
in#ormation She has sited messy des%s and a poorly used #ilin" system as the root cause #or the
lost in#ormation Since this in#ormation is critical #or the #irm, she has decreed that each des%
and o##ice will be cleaned and or"ani=ed, startin" today !s you leave the meetin", you notice
that her o##ice is very disor"ani=ed and that she has made no e##ort to clean up <ill you ma%e
the chan"e> <hy> <hy not> <hat would you do as an e##ective leader to ma%e the chan"e
possible>
Iou probably won-t ma%e the chan"e because the leader has not set an e;ample to #ollow
+##ective leaders Dwal% the tal%E $heir actions display to other members o# the or"ani=ation
what is e;pected !s *wi"ht +isenhower so aptly stated) D$hey never listened to what , said,
they always watch what , doE Since employees #ocus on the activities o# their leaders,
leadership involvement in %ey activities is crucial when encoura"in" people to wor% in a manner
that creates value #or their #irm +##ective leaders can encoura"e people to create value by
supportin" %ey activities drivin" or"ani=ational success $hese %ey activities, whether it be
providin" customers with easy access to support in#ormation or developin" a new manu#acturin"
process, reBuire investment o# both #inancial resources and time +##ective leaders show their
commitment and involvement by providin" #inancial support, becomin" involved in
improvement e##orts, and ma%in" sure that employees have time available in their wor%day to
pursue the necessary activities +##ective leaders %now what needs to be done and provide the
tools to enable the wor%#orce it
.. Iour start time at wor% is 2)./ am However, the boss doesn-t come in until 2)6/, even
thou"h he, too, has a start time o# 2)./ am ,t has come to his attention that people are not
arrivin" at 2)./ He wants to see this chan"e in the #uture <ill you ma%e the chan"e> <hy>
<hy not> <hat would you do as an e##ective leader to ma%e the chan"e possible>
Iou probably won-t ma%e the chan"e because the leader has not set an e;ample to #ollow
+##ective leaders Dwal% the tal%E $heir actions display to other members o# the or"ani=ation
what is e;pected !s *wi"ht +isenhower so aptly stated) D$hey never listened to what , said,
they always watch what , doE Since employees #ocus on the activities o# their leaders,
leadership involvement in %ey activities is crucial when encoura"in" people to wor% in a manner
.4
that creates value #or their #irm +##ective leaders can encoura"e people to create value by
supportin" %ey activities drivin" or"ani=ational success $hese %ey activities, whether it be
providin" customers with easy access to support in#ormation or developin" a new manu#acturin"
process, reBuire investment o# both #inancial resources and time +##ective leaders show their
commitment and involvement by providin" #inancial support, becomin" involved in
improvement e##orts, and ma%in" sure that employees have time available in their wor%day to
pursue the necessary activities +##ective leaders %now what needs to be done and provide the
tools to enable the wor%#orce it
.7 <hich o# *r *emin"-s #ourteen points deal with leadership> Give an e;ample o# where
you have seen these points applied 'or where they need to be applied(
. Create a constancy o# purpose toward improvement o# product and service, with the
aim to become competitive and to stay in business and to provide ?obs
3 ,nstitute trainin" on the ?ob
2 ,nstitute leadership
1 *rive our #ear
0 8rea% down barriers between departments
.7 Remove the barriers that rob people o# their ri"ht to pride o# wor%manship
.6 ,nstitute a vi"orous pro"ram o# education and sel#9improvement
.5 Put everybody in the company to wor% to accomplish the trans#ormation
+##ective leaders welcome the opportunity to listen to their employees and act on valid
su""estions and resolve %ey issues *r *emin" also points out the need to remove barriers that
rob individuals o# the ri"ht o# pride in wor%manship 8arriers are any aspect o# a ?ob that
prevent employees #rom doin" their ?obs well 8y removin" them, leadership creates an
environment supportive o# their employees and the continuous improvement o# their day9to9day
activities ,mproved mana"ement9employee interaction, as well as increased communication
between departments, will lead to more e##ective solutions to the challen"es o# creatin" a product
or providin" a service +ducation and trainin" also play an inte"ral part in *r *emin"-s plan
Continual education creates an atmosphere that encoura"es the discovery o# new ideas and
methods $his translates to innovative solutions to problems $rainin" ensures that products and
services are provided that meet standards established by customer reBuirements
.6 <hat does it mean to mana"e by #act and with a %nowled"e o# variation>
$he theory o# pro#ound %nowled"e involves usin" data to understand situations *r *emin"
encoura"ed the use o# #act9based in#ormation when ma%in" decisions +##ective leaders "ather
and analy=e in#ormation #or trends, patterns, and anomalies be#ore reachin" conclusions
@ana"in" by #act and with a %nowled"e o# variation means bein" able to distin"uish between
controlled and uncontrolled variation $his involves usin" data to understand situations
Ceadership must ma%e sure that employees in an or"ani=ation are not blamed #or #aulty
per#ormance when in actuality it is the system that is #aulty @ana"in" by #act and with a
%nowled"e o# variation enables companies to e;pand beyond small process9improvement e##orts
and to optimi=e their systems in their entirety +##ective leaders have an appreciation #or the
systems that wor% to"ether to create their or"ani=ation-s products and services +##ective leaders
.3
also see% to create ali"nment between their customers- needs, reBuirements, and e;pectations,
the systems that produce products and services and their or"ani=ation-s purpose $his ali"nment
enables these or"ani=ations to do the ri"ht thin"s ri"ht
.2
Chapter ; Strategic "lanning Questions
. How is strate"ic Buality plannin" di##erent #rom traditional strate"ic plannin">
Strategic Quality "lanning &ersus <raditional Strategic "lanning
Strate"ic :uality Plannin" $raditional Strate"ic Plannin"
&ocus on customers &ocus not de#ined or is spread
amon" many considerations
*etermination o# critical success Cac% o# understandin" o# #actors
#actors critical to success
Goals and ob?ectives are process Goals and ob?ectives are results
and results oriented oriented
Goals and ob?ectives are based on data Goals and ob?ectives may be
and driven by trend or pattern analysis based on hunches or "uesses
Process #ocus Product #ocus
!li"nment between critical success #actors, No ali"nment
mission, vision, "oals, ob?ectives,
day9to9day activities
+veryone %nows how their day9to9day &ew people %now how their day9
activities ali"n with critical success #actors, to9day activities #it in with the plan
mission, vision, "oals, ob?ectives
,mprovement activities are #ocused on ,mprovement activities lac% #ocus
activities critical to success
,mprovement activities are both within ,mprovement activities are
and across #unctional areas usually within #unctional areas
.1
7 <hy does an e##ective or"ani=ation need a strate"ic plan>
Strate"ic plans allow leadership to put down in writin" the direction the or"ani=ation is headin"
and how it plans to "et there ,n a competitive business environment, an e##ective or"ani=ation
utili=es care#ully desi"ned strate"ic plans in order to create and sustain its competitive
advanta"es and pro#it position ! well9structured strate"ic plan outlines the rational #or bestin"
the competition in the mar%et by e;ploitin" mar%et opportunities, ma;imi=in" or"ani=ational
stren"ths, and playin" o## o# the competitions- wea%nesses +##orts to address these issues
typically result in a product or service that provides the customer with "reater value, either
throu"h improved Buality, #avorable economics, or enhanced service or per#ormance Strate"ic
plans are the battle plans that enable an or"ani=ation to accomplish their ob?ectives 8y
implementin" these plans, or"ani=ations are able to better place their products or services in the
mar%et Strate"ic plans establish a direction #or the or"ani=ation, the results o# implementin"
these plans is dependent upon the plans themselves, the individuals implementin" them, and the
#orces at wor% in the mar%et
6 <hat are the bene#its o# a strate"ic plan>
8ecause o# their understandin" o# their mar%ets and customers, e##ective or"ani=ations are able
to create and maintain a distinctive customer base Customer needs, wants, and e;pectations
translate directly into reBuirements #or ma?or desi"n parameters to develop, produce, deliver, and
service the product or service ! strate"ic plan uses this in#ormation and incorporates strate"ies
#or improvin" customer satis#action throu"h providin" better products, services, economics,
delivery, and Buality
5 *escribe each o# the elements needed #or the strate"ic plannin" process
! strate"ic plan de#ines the !usiness the organi3ation intends to !e in, the %ind o# or"ani=ation
it wants to be, and the %ind o# economic and non9economic contribution it will ma%e to its
sta%eholders, employees, customers, and community $he plan spells out the organi3ations
goals and o!=ecti&es and ho( the organi3ation (ill achie&e these goals and o!=ecti&es. $he
strate"ic plan concentrates on the critical success 4actors >CS8s? #or the or"ani=ation, providin"
plans #or closin" the "aps between what the or"ani=ation is currently capable o# doin" versus
what it needs to be able to do Usin" indicators or per4ormance measures, the or"ani=ation
will monitor its pro"ress toward meetin" the short9term, mid9term, and ultimately lon"9term
"oals ! "ood strate"ic plan also includes contingency plans in case some o# the basic
assumptions are in error or si"ni#icant chan"es in the mar%et occur

4 *escribe the steps necessary to create a strate"ic plan
$he strate"ic plan is essentially a #ramewor% that assists the or"ani=ation in achievin" its vision
while allowin" #le;ibility to deal with un#oreseen chan"es in the business environment $o
create a strate"ic plan, the #ollowin" must be identi#ied)
.0
. $he Jision)
$he or"ani=ation-s strate"ic direction #or the #oreseeable #uture
7 $he @ission)
$he translation o# the or"ani=ation-s vision into strate"ic actions
6 $he Critical Success &actors)
$he three to ten thin"s, as identi#ied by customers, that absolutely must be done
well i# the company is "oin" to thrive
5 $he Goals
$he thin"s that must be achieved in order to support the critical success #actors
4 $he Ob?ectives)
$he speci#ic and Buantitative actions that must be ta%en in order to support the
accomplishment o# the "oals and ultimately the mission, and vision
3 $he ,ndicators)
$he per#ormance measures that indicate whether or not the or"ani=ation is
movin" toward meetin" their ob?ectives, "oals, mission and vision
2 $he Contin"ency Plans
$he plans in place that enable an or"ani=ation to remain #le;ible in a comple;,
competitive environment
3 <hat does an or"ani=ation need to %now about itsel# be#ore creatin" a strate"ic plan> <hy is
this in#ormation important>
,n preparation #or creatin" a strate"ic plan, the or"ani=ation-s leaders should determine)

. $he or"ani=ation-s business '$he business they are really in>(
7 $he principal #indin"s #rom the internal and e;ternal assessments)
! Stren"ths and wea%nesses
8 Customer in#ormation
C +conomic environment in#ormation
* Competition in#ormation
+ Government reBuirements
& $echnolo"ical environment
2 <hy is strate"y deployment as important as strate"ic plannin">
$o be e##ective, a strate"ic plan must be deployed !s livin" documents, they are not meant to
sit on a shel#, only to be touched when it is time #or an annual revision <ithout deployment, the
wor% on the strate"ic plan remains un%nown to the employees o# an or"ani=ation $hey are not
able to act and react to mar%et #orces in an or"ani=ed, e##ective manner
1 How is ali"nment created in a strate"ic plan>
7/
Alignment means that if you push on one end, the other end will move in the direction you want.
+##ective leaders enable members o# the or"ani=ation to ma%e the transition between the strate"ic
plan and daily business activities by translatin" <H!$ needs to be accomplished into HO< it
will be accomplished +##ective leaders ma%e sure that the day9to9day activities and the "oals o#
the strate"ic plans o# the or"ani=ation are in harmony and #ocused on what is critical to the
success o# the or"ani=ation $hey want to ensure that i# they push on the strate"ic plan, the
actions o# their employees will "o in the desired direction &or this reason, the strate"y must be
clearly communicated throu"hout the or"ani=ation +##ective leaders ma%e sure that the strate"ic
plan contains clear ob?ectives, provides and utili=es measures o# per#ormance, assi"ns
responsibilities to speci#ic individuals, and denotes timin"
7.
Chapter * .uman %esource De&elopment Questions
4 Havin" chan"ed somethin" about yoursel#, how would you encoura"e someone else to ma%e a
chan"e>
Ceadership must communicate to the employees the desired chan"e and motivate individuals to
ma%e the chan"e People resist chan"e because humans are control9oriented and when their
environments are disrupted, they perceive that they have lost the ability to control their lives
Resistance to chan"e can be based on the individual-s #rame o# re#erence, their individual values,
emotions, %nowled"e, and behavior Ceaders need to understand that resistance to chan"e is a
natural human reaction +##ective leaders %now that in order #or chan"e to occur, resistance to
those chan"es must be e;pected and planned #or Ceaders must clearly communicate the new
e;pectations and the reasons behind the chan"es so that people understand why they are bein"
as%ed to chan"e +##ective leaders provide trainin" and time to ma%e the chan"e $hey also
structure their reward system to support the chan"e +##ective leaders reali=e that employees
mold their behavior accordin" to their interpretations o# the si"nals leadership sends them
$hese si"nals may come #rom policies, reBuests, edicts, or #rom the day9to9day actions ta%en by
leadership +##ective leaders reco"ni=e that actions spea% louder than words Communicatin"
throu"h leadership actions and e;amples is paramount to chan"in" behavior +##ective leaders
reco"ni=e that the tools need to be in place to support the desired chan"e $o ma;imi=e the
chan"e process, e##ective leaders ensure that the wor%erFmachineFcomputer inter#ace, as well as
the wor%er9to9wor%er inter#ace, is compatible with the needs, capabilities, and limitations o# the
wor%er &urther, e##ective leaders ensure that the reward system matches the desired
e;pectations in order to chan"e behavior !li"nment must e;ist between rewards, e;pectations,
leadership actions, and customer needs 8oth rewards and punishments must rein#orce behavior
e;pectations
What is the desired end result !an you picture it $o chan"e, a person must understand what the
ultimate outcome o# that chan"e will be
What actions will you ta"e to ma"e the change $o chan"e, a person ta%es a series o# action that
produces a result that moves the individual toward the desired outcome
What is the time frame for the change Chan"e ta%es time
#ow will you stay motivated $o chan"e, it is necessary to stay motivated
#ow will you "now you have changed What will your indicators $e ! person needs to have
some sort o# #eedbac% that enables them to understand how they are pro"ressin" toward the
desired outcome
3 *escribe the phases o# team development
&ormation sta"e is usually e;perienced in the #irst #ew meetin"s *urin" this time, the team
establishes its "oals and ob?ectives ,t also determines the "round rules #or team per#ormance
&or teams to wor% well, leadership must set clear "oals that are ali"ned with the mission and
77
strate"ic direction o# the #irm <hen leadership sets the direction, the team is much more
#ocused and tends not to "et bo""ed down in the problem9selection process $he team must
%now the scope and boundaries it must wor% within Ceadership must communicate how the
team-s pro"ress and per#ormance will be measured
*urin" the stormy sta"e, the team "ets acBuainted with each others- idiosyncrasies and the demands
o# the pro?ect $he "oals and scope o# the team may be Buestioned Since a team is composed o#
a "roup o# individuals who are united by a common "oal, the best teamwor% will occur when the
individuals #ocus on the team-s ob?ectives rather than personal motives <hile wor%in" to"ether,
team members must understand and a"ree on the "oals o# the team $hey must establish and
adhere to team "round rules #or behavior and per#ormance e;pectations $o ensure harmony in
the team, all members must participate and the responsibilities and duties must be #airly
distributed $his means that each team member must understand what hisFher role is in the
completion o# the pro?ect Knowled"e o# how internal or e;ternal constraints a##ect the pro?ect is
also help#ul $eam members must possess a variety o# s%ills, includin" problem9solvin" s%ills,
plannin" s%ills, #acilitation and communication s%ills, and #eedbac% and con#lict mana"ement
s%ills
$he per#ormin" sta"e is when the team starts to wor% to"ether smoothly $o be success#ul, teams
need the appropriate s%ills in a supportive or"ani=ational culture and the authority to do the ?ob
that they have been as%ed to do Ceadership can do a lot to rid the team o# the barriers that
inhibit its per#ormance $hese barriers include) inadeBuate release time, territorial behavior #rom
involved #unctional areas, lac% o# trainin", inadeBuate support systems, lac% o# "uidance or
direction, and lac% o# reco"nition Senior leadership-s sincere interest and support in the
resolution o# the problem is evidenced by their willin"ness to commit money and time #or
trainin" in problem9solvin" and #acilitation ,n any case, senior leadership must monitor and
encoura"e their teams to solve problems $he teams will Buic%ly become unmotivated i# the
solutions they propose are consistently turned down or i"nored Ceadership support will be
obvious in mana"ement-s visibility, dia"nostic support, reco"nition, and limited inter#erence
$he concludin" sta"e occurs as the team #inishes its pro?ect *urin" this phase, team members
draw the pro?ect to its conclusion, veri#y the results, and disband the team Several %ey events
ta%e place durin" this time $he team, havin" ta%en action, perhaps by implementin" a solution
to a problem, must veri#y that what they planned to do "ot done and what they did actually
wor%ed $eams are not #inished when they have proposed a plan o# action, teamwor% is #inished
when the plans have been acted on and the results ?ud"ed e##ective Until then, the team cannot
be disbanded
./ <hat is the di##erence between education and trainin"> <hy is it important to have both>
$rainin" re#ers to ?ob9related s%ill trainin" and is usually a combination o# on9the9?ob trainin" with
classroom9type instruction +##ective employees are provided with the appropriate trainin" to
"ive them the s%ills and %nowled"e set needed to e;cel in their ?obs Job9related s%ill trainin"
prepares wor%ers #or the daily activities involved with their ?ob Such trainin" should also
include in#ormation that helps the employee deal with e;periences they may rarely encounter ,#
trainin" does not include problems that may arise in#reBuently, then the wor%er will be reBuired
76
to handle those situations as they arise with their own problem9solvin" and decision9ma%in"
s%ills ,# these in#reBuent situations are unsa#e in any way, this could result in an accident or
in?ury &ollow9up or re#resher trainin" is also %ey to s%ill acBuisition $his type o# trainin"
enables employees to maintain hi"her s%ill and per#ormance levels by re9#amiliari=in" employees
with the best practices and eliminatin" poor habits
Compared to trainin", education is more broad9based +ducation provides individuals with a
broader base o# %nowled"e $his allows individuals to loo% at a situation #rom other dimensions
$he education an individual receives may not be immediately applicable to the activities they are
currently per#ormin"
.6 <hat are the %ey items that must be in place in order to have an e##ective meetin">
,n order to have e##ective meetin"s, care should be ta%en to incorporate several principles)
. *etermine the ob?ective o# the meetin"
9<hy is the meetin" "oin" to be held>
7 *etermine who should participate
9<ho can in#luence the #ul#illment o# the meetin" ob?ective>
6 Set an a"enda
9<hat is the plan o# action #or the meetin">
5 Prepare #or the meetin"
9<hat will be needed to accomplish, provide answers, and save time> +;perts>
,n#ormation> Key participants>
4 Run the meetin"
9How will the a"enda be used to %eep the meetin" on trac%>
3 @a%e decisions
9How will the issues be dealt with>
2 !ssi"n responsibilities
9<ho will be responsible #or achievin" results>
1 &ollowin" the meetin", veri#y that the issues have been dealt with by the assi"ned individuals
.5 <hat is an a"enda> How does it help create an e##ective meetin">
Havin" and #ollowin" an a"enda is one o# the %ey aspects to havin" a success#ul meetin" !
"ood a"enda will state
9the ob?ective o# the meetin"
9the issues that will be discussed
9the be"innin" and endin" time o# the meetin"
9the location
9the participants
9the preparation e;pected o# the participant
75
Chapter - Managing the Supply Chain Questions
. <hat is a supply chain>
! supply chain is the networ% o# or"ani=ations involved in the movement o# materials,
in#ormation, and money as raw materials #low #rom their source throu"h production until they
are delivered as a #inished product or service to the #inal customer !n e##ective supply chain
must be desi"ned with Buality, cost, #le;ibility, speed, and customer service in mind
7 <hat are the bene#its o# a well9mana"ed supply chain>
Supply chains create value in two ways) the physical movement o# materials and the e;chan"e o#
in#ormation about those materials +##ective supply chains improve supply Buality, delivery, and
price
6 <hat are the ob?ectives o# e##ective supply chain mana"ement>
8y mana"in" the supply chain, e##ective or"ani=ations improve business operations, mana"e
inventories, and reduce costs $heir primary ob?ectives are to move product throu"h the supply
chain in a cost e##ective manner $he critical elements o# a supply chain are inventory
mana"ement, in#ormation sharin", e9commerce, lo"istics, and purchasin"
5 <hat or"ani=ations mi"ht be involved in a supply chain>
!ny or"ani=ation that needs to move product or services will have supply chains
4 <hat do or"ani=ations involved in a supply chain trans#er bac% and #orth>
,n#ormation, material, and money are all trans#erred bac% and #orth in a supply chain
3 <hy is in#ormation so critical to supply chain mana"ement>
,n#ormation is critical to supply chain mana"ement because or"ani=ations in a supply chain need
to %now what is needed, how much is needed, and when it is needed
2 <hat role does in#ormation play in a supply chain>
,n#ormation is the inte"ral lin% between upstream and downstream suppliers and customers
1 <hat role does e9commerce play in supply chain mana"ement>
+9commerce is chan"in" the #ace o# business $hese chan"es a##ect the supply chain by
reBuirin" new methods, new trainin", new eBuipment, and new approaches to doin" business
Or"ani=ations must utili=e technolo"y to enhance their supply chains
74
0 <hat are the challen"es to creatin" an e##ective supply chain>
Supply chains must reliably provide products and services #or customers ,n#ormation, money,
and materials must #low easily throu"h supply chains $his #low must be synchroni=ed with
demand *isruptions in the supply chain, such as a loss o# a company in the chain or a lac% o#
in#ormation, can destroy the e##ectiveness o# the supply chain +##ective or"ani=ations #ocus on
communication, in#ormation sharin", improvin" business operations, mana"in" inventories, and
reducin" costs
./ *escribe the role that purchasin" plays in the supply chain
Purchasin" has evolved #rom simple purchases o# products #rom individual suppliers to
purchases o# entire supply chains lin%in" their or"ani=ation with the raw materials their products
need Purchasin" #ocuses on creatin" business9to9business relationships that must be monitored,
improved, optimi=ed, and mana"ed Purchasin" is responsible #or #indin" out what is out there,
what the options are, how much it costs, and when it can be delivered
.. *escribe the purchasin" cycle
See #i"ure in te;t
.7 <hat is meant by lo"istics>
Co"istics is the process o# determinin" the best methods o# procurin", maintainin", pac%a"in",
transportin", and storin" o# materials and personnel in order to satis#y customer demand
Co"istics is the physical side o# the supply chain and includes) material handlin", pac%a"in",
warehousin", sta"in", and transportation o# eBuipment, parts, subassemblies, tools, #uels,
lubricants, o##ice supplies, in#ormation, and anythin" else that is needed to %eep the or"ani=ation
#unctionin"
.6 <hy does lo"istics play a %ey role in supply chain mana"ement>
Co"istics mana"es the #low o# materials and in#ormation within a #acility, and incomin" and
out"oin" #rom that #acility Co"istics is the physical side o# the supply chain and includes)
material handlin", pac%a"in", warehousin", sta"in", and transportation o# eBuipment, parts,
subassemblies, tools, #uels, lubricants, o##ice supplies, in#ormation, and anythin" else that is
needed to %eep the or"ani=ation #unctionin"
73
Chapter Measures o4 Organi3ational Success Questions
. How do e##ective or"ani=ations use per#ormance measures>
Ceaders use measures o# per#ormance to ensure ali"nment between the or"ani=ation-s mission,
strate"y, values, and behavior @easures o# per#ormance enable e##ective or"ani=ations to
de#ine the meanin" o# success numerically +##ective per#ormance measurement systems are
used #or understandin", ali"nin", and improvin" per#ormance at all levels and in all parts o# the
or"ani=ation ,n order to %now how they are doin" in %ey areas that a##ect their customers- value
perceptions, or"ani=ations must select and trac% indicators o# their per#ormance e##orts
7 <hat is the di##erence between process and results measures>
Processes are the activities that must ta%e place in order to produce a product or provide a
service Since processes are how or"ani=ations do the wor% that they do, process measures
monitor operational activities or how the wor% is done Results relate to both or"ani=ations and
their customers $o an or"ani=ation, results are the ob?ectives the or"ani=ation wants to achieve
&rom a customer point o# view, results represent what they hope to obtain by doin" business with
the or"ani=ation, whether it be throu"h a product or a service Per#ormance measures related to
or"ani=ational results #ocus on strate"ic intent
6 <hy is an e##ective per#ormance measurement system necessary>
One o# the chie# "oals o# a per#ormance measurement system is to provide leaders with a multi9
dimensional and Bualitative view o# their or"ani=ation ! measurement system is a critical
element in the strate"ic plannin" process because it allows an or"ani=ation to measure pro"ress
toward "oals and ob?ectives Per#ormance measures are decision tools that enable leaders to lin%
their strate"y with day9to9day operations +##ective or"ani=ations measure the per#ormance o#
areas that the or"ani=ation values the most @easurement systems allow e##ective or"ani=ations
to)
*etermine that a "ap e;ists between desired and actual per#ormance
*etermine the root cause o# the "ap
*etermine the necessary corrective action to eliminate root cause o# the "ap
*etermine whether or not the corrective actions eliminated the root cause and closed the
"ap between the actual and desired per#ormance
5 Create a set o# measures, based on the 8alanced Scorecard and the e;ample in the te;t, #or a
#ast #ood restaurant
Customer measures)
Results measures) Overall customer satis#action
@ar%et share
9number o# customers
72
9number o# repeat customers
9number o# new customers
Process measures) Chan"es in customer reBuirements versus
Chan"es in processes to serve customers
,mprovements to processes critical to servin" customers
&inancial measures)
Results measures) Cost per #ood item
Pro#itability
Return on ,nvestment
Process measures) Cost avoidance 'sa#ety, hy"iene(
,nternal measures)
Results measures) ,mprovements in hours paid versus dollar amount o# #ood served
Process measures) ,mprovements in order turn9around time
'Cycle time reductionFremoval o# non9value9added activities(
,mprovements in #irst time throu"h Buality
'Reduction in rewor%Fscrap(
Cearnin" and Growth measures)
Results measures) ,mprovements in employee retention
Process measures) Pro"ress toward cross9trainin" "oals #or critical processes as
identi#ied by customers
4 Create a set o# measures, based on the 8alanced Scorecard and the e;ample in the te;t, #or a
movie theater
Customer measures)
Results measures) Overall customer satis#action
@ar%et share
9number o# customers
9number o# repeat customers
9number o# new customers
Process measures) ,mprovements to processes critical to servin" customers
&inancial measures)
Results measures) Cost per movie
Pro#itability
Return on ,nvestment
Process measures) Cost avoidance 'movie #ilm strip Buality(
,nternal measures)
Results measures) ,mprovements in hours paid versus tic%ets sold
Process measures) ,mprovements in theater turn9around time
'Cycle time reductionFremoval o# non9value9added activities(
,mprovements in #irst time throu"h Buality
'Reduction in rewor%Fscrap in #oodservice area(
71
Cearnin" and Growth measures)
Results measures) ,mprovements in employee retention
Process measures) Pro"ress toward cross9trainin" "oals #or critical processes as
identi#ied by customers
3 <hat is a prevention cost> How can it be reco"ni=ed> *escribe where prevention costs can
be #ound
Prevention costs are those costs that occur when a company is per#ormin" activities desi"ned to
prevent poor Buality in products or services Prevention costs are o#ten seen as #ront9end costs
desi"ned to ensure that the product or service is created to meet the customer reBuirements
+;amples o# such costs are desi"n reviews, education and trainin", supplier selection and
capability reviews, and process improvement pro?ects Prevention activities must be reviewed to
determine i# they truly brin" about improvement in the most cost9e##ective manner
Prevention e##orts try to determine the root causes o# problems and eliminate them at the source
so reoccurrences do not happen Preventin" poor Buality stops companies #rom incurrin" the
cost o# doin" it over a"ain +ssentially, i# they had done it ri"ht the #irst time, they would not
have to repeat their e##orts $he initial investment in improvin" processes is more than
compensated by the resultin" cost savin"s
2 <hat is an appraisal cost> How can it be reco"ni=ed> *escribe where appraisal costs can be
#ound
!ppraisal costs are the costs associated with measurin", evaluatin", or auditin" products or
services to ma%e sure that they con#orm to speci#ications or reBuirements !ppraisal costs are
the costs o# evaluatin" the product or service durin" the production o# the product or the
providin" o# the service to determine i#, in its un#inished or #inished state, it is capable o#
meetin" the reBuirements set by the customer !ppraisal activities are necessary in an
environment where product, process, or service problems are #ound !ppraisal costs can be
associated with raw materials inspection, wor%9in9process 'activities9in9process #or the service
industries( evaluation, or #inished product reviews +;amples o# appraisal costs include
incomin" inspection, wor%9in9process inspection, #inal inspection or testin", material reviews,
and calibration o# measurin" or testin" eBuipment <hen the Buality o# the product or service
reaches hi"h levels, then appraisal costs can be reduced
1 <hat are the two types o# #ailure costs> How can they be reco"ni=ed> *escribe where these
costs can be #ound
&ailure costs occur when the complete product or service does not con#orm to customer
reBuirements $wo types e;ist) internal and e;ternal ,nternal #ailure costs are those costs
associated with product noncon#ormities or service #ailures #ound be#ore the product is shipped
or the service is provided to the customer ,nternal #ailure costs are the costs o# correctin" the
situation $he #ailure costs may ta%e the #orm o# scrap, rewor%, rema%in", re9inspection, or
retestin" +;ternal #ailure costs are the costs that occur when a noncon#ormin" product or
70
service reaches the customer +;ternal #ailure costs include the costs associated with customer
returns and complaints, warranty claims, product recalls, or product liability claims Since
e;ternal #ailure costs have the "reatest impact on the corporate poc%etboo%, they must be reduced
to =ero 8ecause they are hi"hly visible, e;ternal costs o#ten receive the most attention
Un#ortunately, internal #ailure costs may be seen as necessary evils in the process o# providin"
"ood9Buality products or services to the consumer Nothin" could be more #alse *oin" the
wor% twice, throu"h rewor% or scrap, is not a success#ul strate"y #or operatin" in today-s
economic environment
0 *escribe the relationship amon" prevention costs, appraisal costs, and #ailure costs <here
should a company-s e##orts be #ocused> <hy>
$he #our types o# Buality costs are interrelated ,n summary, total Buality costs are considered to
be the sum o# prevention costs, appraisal costs, #ailure costs, and intan"ible costs ,nvestments
made to prevent poor Buality will reduce internal and e;ternal #ailure costs Consistently hi"h
Buality reduces the need #or many appraisal activities Suppliers with stron" Buality systems in
place can reduce incomin" inspection costs Hi"h appraisal costs combined with hi"h internal
#ailure costs si"nal that poor9Buality products or services are bein" provided +##orts made to
reduce e;ternal #ailure costs will involve chan"es to e##orts bein" made to prevent poor Buality
,nternal #ailure costs are a portion o# the total production costs, ?ust as e;ternal #ailure costs
reduce overall pro#itability ! trade9o## to be aware o# when dealin" with Buality costs is the
need to ensure that appraisal costs are well spent Companies with a stron" appraisal system
need to balance two points o# view) ,s the company spendin" too much on appraisal #or its "iven
level o# Buality per#ormance or is the company ris%in" e;cessive #ailure costs by under9#undin"
an appraisal pro"ram> ,n all three areasLprevention, appraisal, and #ailure costsLthe activities
underta%en must be evaluated to ensure that the e##orts are "ainin" #urther improvement in a
cost9e##ective manner
./ How are Buality costs used #or decision9ma%in">
:uality costs can be used as ?usti#ication #or actions ta%en to improve the product or service
$ypically, investments in new eBuipment, materials, or #acilities reBuire the pro?ect sponsor to
determine which pro?ects will provide the "reatest return on investment $hese calculations
traditionally include in#ormation on labor savin"s, production time savin"s, and ability to
produce a "reater variety o# products with better Buality $he Dbetter BualityE aspect o# these
calculations can be Buanti#ied by investi"atin" the costs o# Buality, particularly the #ailure costs
,t is important to determine the costs o# in9process and incomin" material inspection, sortin",
repair, and scrap, as well as the intan"ible costs associated with havin" a noncon#ormin" product
or service, reach the customer @a%in" a decision with more complete Buality in#ormation, such
as product appraisal costs, can help determine the true pro#itability o# a product or service
,denti#yin" and Buanti#yin" Buality costs has a two#old bene#it Cost savin"s are identi#ied and
Buality is improved 8y improvin" the Buality per#ormance o# a company, the company also
improves its Buality costs
.. <hat should a Buality cost system emphasi=e>
6/
! Buality cost system should emphasi=e the identi#ication and elimination o# unnecessary costs
o# Buality, speci#ically #ailure costs and intan"ible costs associated with poor Buality $he
Buality cost system should also determine where prevention costs should be spent to ma;imi=e
value and whether or not appraisal costs can be reduced or eliminated throu"h ?udicious
prevention cost spendin"
.7 <hat are the bene#its o# havin", #indin", or determinin" Buality costs>
:uality costs are the costs that would disappear i# every activity was per#ormed without de#ects
every time 8ein" able to #ind or determine Buality costs provides an or"ani=ation with a
roadmap showin" where to #ocus improvement e##orts
.6 *escribe the steps involved in benchmar%in"
. Determine the focus !t the be"innin" o# a benchmar%in" e;perience, those involved
must determine what aspect o# their company will be the #ocus o# the study $he #ocus may be
based on customer reBuirements, on standards, or on a "eneral continuous improvement process
,n#ormation "athered durin" the benchmar%in" e;perience should support the or"ani=ation-s
overall mission, "oals, and ob?ectives 8e aware that benchmar%in" and "atherin" in#ormation
about processes is o# "reater value than #ocusin" on metrics
7. Understand your organi%ation Critical to any process is de#inin" and understandin"
all aspects o# a situation ,ndividuals involved in the process need to develop an understandin"
o# their company $o create a plan and conduct the process, in#ormation concernin" the e;ternal
customers, internal customers, and their ma?or inputs and outputs is vital to achieve an
understandin" o# the system under study O#ten this step receives less attention than it should
6 Determine what to measure Once an understandin" o# the systems present in a
company has been "ained, it is time to determine the measures o# per#ormance $hese measures
will allow those conductin" the benchmar%in" assessment to ?ud"e the per#ormance o# the
company $his is the time to de#ine what is truly critical #or the company to remain competitive
$hese critical #actors #or success will be supported by standards #or procedures, processes, and
behaviors 8enchmar%in" will pinpoint Buestions to be answered and issues to be resolved, as
well as processes and procedures to be improved
5 Determine whom or what to $enchmar" against Choices #or whom to benchmar%
a"ainst should be made by considerin" the activities and operations under investi"ation, the si=e
o# the company, the number and types o# customers, the types o# transactions, even the locations
o# #acilities Care#ul attention should be paid to selectin" appropriate companies Similarities in
si=e and types o# transactions or products may be more important in some instances than
selectin" a competitor
4 &enchmar" $he areas o# the company that have been chosen #or the benchmar%in"
assessment should be noti#ied prior to be"innin" the process $he authori=ation to proceed with
the process should be obtained and noti#ication should come #rom the hi"hest levels o# the
6.
company to ensure cooperation *urin" the benchmar%in" process, investi"ators collect and
analy=e data pertainin" to the measures established in step 6
3 'mprove performance Once the data and in#ormation have been "athered, a report
summari=in" the si"ni#icant stren"ths and wea%nesses o# the area under study is created ,n this
report, the "ap between the e;istin" and the desired levels o# per#ormance is documented !
"ood report will #ocus on patterns o# standards violations and elements missin" #rom a stron"
system
67
Chapter 20 @enchmar7ing Questions
. <hy would a company be interested in benchmar%in">
,n a success#ul benchmar%in" e;perience, the #inal report becomes a wor%in" document to aid
the continuous improvement process $he in#ormation "athered in this report is used to
investi"ate root causes, solve them, reduce process variation, and establish systems to prevent the
occurrence o# noncon#ormities $he benchmar%in" document is a power customer #eedbac% tool
and should be used accordin"ly
7 <hat would they hope to "ain by benchmar%in">
8enchmar%in" will point out areas where improvements are needed 8enchmar%in" can be used
to determine whether or not a company-s Buality systems are able to #ul#ill the reBuirements
appropriate to meet ,SO 0/// or Buality award standards 8enchmar%in" can also answer such
Buestions as) !re the company-s processes properly constructed and documented> !re systems
in place to allocate resources and #undin" appropriately> <hich areas have the "reatest
improvement needs> <hat are our internal and e;ternal customer needs>
6 *escribe the di##erence between internal and e;ternal benchmar%in"
,nternal benchmar%in" #ocuses on activities within the or"ani=ation One area o# the
or"ani=ation is compared with another +;ternal benchmar%in" can either be competitive or
#unctional ,n competitive benchmar%in", an or"ani=ation #ocuses on companies within their
own mar%et, sometimes direct competitors, studyin" their business per#ormance and processes
&unctional benchmar%in" is per#ormed by companies wantin" to study a particular process
$hey choose or"ani=ations with similar processes re"ardless o# their industry
5 *escribe why an or"ani=ation would want to benchmar% a"ainst an industry leader>
Comparin" one-s per#ormance a"ainst industry leaders can be a power#ul tool #or companies
wishin" to improve their position in the mar%etplace 8y comparin" their own per#ormance with
that o# the mar%et leader, companies can better understand their own assets and capabilities as
well as the areas where they need improvement
4 *escribe the steps involved in benchmar%in"
. Determine the focus !t the be"innin" o# a benchmar%in" e;perience, those involved
must determine what aspect o# their company will be the #ocus o# the study $he #ocus may be
based on customer reBuirements, on standards, or on a "eneral continuous improvement process
,n#ormation "athered durin" the benchmar%in" e;perience should support the or"ani=ation-s
overall mission, "oals, and ob?ectives 8e aware that benchmar%in" and "atherin" in#ormation
about processes is o# "reater value than #ocusin" on metrics
7. Understand your organi%ation Critical to any process is de#inin" and understandin"
all aspects o# a situation ,ndividuals involved in the process need to develop an understandin"
66
o# their company $o create a plan and conduct the process, in#ormation concernin" the e;ternal
customers, internal customers, and their ma?or inputs and outputs is vital to achieve an
understandin" o# the system under study O#ten this step receives less attention than it should
6 Determine what to measure Once an understandin" o# the systems present in a
company has been "ained, it is time to determine the measures o# per#ormance $hese measures
will allow those conductin" the benchmar%in" assessment to ?ud"e the per#ormance o# the
company $his is the time to de#ine what is truly critical #or the company to remain competitive
$hese critical #actors #or success will be supported by standards #or procedures, processes, and
behaviors 8enchmar%in" will pinpoint Buestions to be answered and issues to be resolved, as
well as processes and procedures to be improved
5 Determine whom or what to $enchmar" against Choices #or whom to benchmar%
a"ainst should be made by considerin" the activities and operations under investi"ation, the si=e
o# the company, the number and types o# customers, the types o# transactions, even the locations
o# #acilities Care#ul attention should be paid to selectin" appropriate companies Similarities in
si=e and types o# transactions or products may be more important in some instances than
selectin" a competitor
4 &enchmar" $he areas o# the company that have been chosen #or the benchmar%in"
assessment should be noti#ied prior to be"innin" the process $he authori=ation to proceed with
the process should be obtained and noti#ication should come #rom the hi"hest levels o# the
company to ensure cooperation *urin" the benchmar%in" process, investi"ators collect and
analy=e data pertainin" to the measures established in step 6
3 'mprove performance Once the data and in#ormation have been "athered, a report
summari=in" the si"ni#icant stren"ths and wea%nesses o# the area under study is created ,n this
report, the "ap between the e;istin" and the desired levels o# per#ormance is documented !
"ood report will #ocus on patterns o# standards violations and elements missin" #rom a stron"
system
3 How does as%in" the 4 why-s and 7 how-s #acilitate the improvement process>
$he 4 why-s and 7 how-s help people #ocus on the situation $hey serve as clari#yin" Buestions
Since they are easy to remember, they can be easily used to #acilitate the improvement process
3 <hat types o# Buestions do the why-s and how-s answer> Can you thin% o# any others>
1ee figure in te"t.
2 How would you choose a particular company or "roup o# companies to benchmar% a"ainst>
65
Ceaders will choose companies that are success#ul, innovative, and ori"inal $hey will
constantly see% the company o# other or"ani=ation leaders who are aware o# who is doin" what
in the mar%et place $hey read, they attend industrial "atherin"s, all the time tryin" to locate
who does what well Ceaders o# or"ani=ations that use benchmar%in" can be heard to remar%
that they search out and shamelessly borrow "ood ideas #rom other businesses and cultures
$hey are not cau"ht up in the myopic Dnot invented hereE syndrome $hey are always on the
loo%out #or improvement ideas that can be inte"rated into their own processes

1 Study the @8N:! criteria Cite and discuss speci#ic sections where benchmar%in" could be
applied
@a%e sure the answer ties in true evidence o# e##ectiveness throu"h comparisons with the
business results criteria o# the @8N:!C See sections 5., 37, and 2
64
Chapter 22 "rocess Management Questions
. <hat is the di##erence between a #unctionally9structured or"ani=ation and a process9
#ocused or"ani=ation>
,n a #unctionally9structured or"ani=ation, #unctional activities are "rouped to"ether and
mana"ed as separate entities $his means that people or machines that per#orm similar
activities are "rouped to"ether and are overseen by a mana"er #or their unit !s each
person or machine completes their #unction 'activity(, the item is handed o## to the ne;t
#unction ,n a #unctionally structured or"ani=ation, mana"ement boundaries are clear
Ci%e activities are clearly "rouped into an individual department, each department
containin" its own mana"er, sta##, supplies, bud"et, eBuipment, and speciali=ed duties
Since the wor% is divided into distinct activities, people become specialists in their ?obs
and only their ?obs Some traits o# a #unctional or"ani=ation are shown in the #i"ure in the
te;t
! process or"ani=ation is arran"ed accordin" to processes or product lines $he
or"ani=ation #ocuses on the %ey business processes that they must absolutely do well in
order to "ain, satis#y, and retain customers $raditional departmental boundaries are
blurred or non9e;istent in a process oriented company ! process9oriented or"ani=ation is
by its very nature #le;ible ,ndividuals wor% toward completin" an overall process rather
than one particular activity &or this reason, people are cross9trained as well as aware o#
all the steps in the process o# providin" a product or service #or the customer Resources
such as materials and in#ormation #low throu"h the process to where they are needed
Process mana"ers are responsible #or the overall process and what it produces
,ndividuals in the process are ?ud"ed based on their contribution to the overall process and
what it produces Some traits o# a process9oriented or"ani=ation are shown in the #i"ure in
the te;t
7 How does a #ocus on processes and process improvement help an or"ani=ation become
more e##ective>
! process orientation #orces people to become aware o# the lin%s between the activities in
the process ! process orientation enables an or"ani=ation to achieve its overall mission
and ob?ectives more easily because it "ets everyone in the or"ani=ation ali"ned with the
%ey business processes that absolutely must ta%e place in order #or the company to attract
and retain customers +##ort is #ocused on improvin" the lin%s between activities People
understand how the entire process they are wor%in" on operates and they are also much
more aware o# how their day9to9day activities support this process $hey can clearly see
the relationships between various activities $hey concentrate on supportin" these
activities rather than their own particular specialty ,# a disconnect e;ists involvin"
people, materials, in#ormation, or eBuipment, it is readily identi#ied and an e##ort is made
to correct the situation ! process #ocus supports the use o# problem9solvin" and sel#9
directed wor% teams Since individuals in the or"ani=ation can see the lin%s between
activities, they are more li%ely to wor% to"ether to enhance their activities and the lin%s
63
between them &or this reason, continuous improvement is easier within a process9
oriented or"ani=ation
6 How would you reco"ni=e that a process needed to be improved>
@any processes develop over time, with little concern #or whether or not it is the most
e##ective manner in which to provide a product or service <hen customer complaints
arise or #inancial results aren-t as e;pected, see% out waste#ul processes and improve them
$he processes providin" the products and services will need to be improved with the aim
o# preventin" de#ects and increasin" productivity by reducin" process cycle times and
eliminatin" waste $he %ey to re#inin" processes is concentratin" on the process #rom the
customer-s point9o#9view and identi#yin" and eliminatin" non9value added activities
5 <hat is meant by a value9added operation>
! value9added activity or operation is one that accomplishes somethin" that moves the
process toward its #inal "oal $here is a direct relationship between the activity and the
end result desired
4 How would you reco"ni=e a non9value added activity>
! non9value added activity costs time, money, and e##ort $hese detractors are waste#ul
and are easily identi#ied because they are activities that result in no bene#it to the
or"ani=ation Non9value activities in processes stand out because they are o#ten labeled
with an Dre,E as in) rewor%, redo, repeat, recycle, reclaim, resubmit, etc +liminatin" non9
value added activities clari#ies the process and allows it to #ocus on meetin" the needs,
reBuirements, and e;pectations o# the customer
3 <hy is process mappin" an important tool #or improvin" processes>
,n most or"ani=ations, very #ew people truly understand the myriad o# activities in a
process that it ta%es to create a product or service Process maps are power#ul
communication tools that provide a clear understandin" o# how business is conducted
within the or"ani=ation ,denti#yin" and writin" down the process in pictorial #orm helps
people understand ?ust how they do the wor% that they do Process maps have the ability
to accurately portray current operations and can be used to evaluate these operations !
process map also identi#ies the activities that have been added to the process over time in
order to adapt older process to chan"es in the business Once chan"es have been
proposed, process maps are eBually power#ul #or communicatin" the proposed chan"es to
the process
2 <hy is it important to determine the process boundaries>
$he process should be studied #rom the customer-s point9o#9view ,t is the customer who
determines the boundaries o# the process Once it is understood how the customer views
the process, the process boundaries can be established $rue process improvement comes
62
#rom a %nowled"e o# what adds value or meanin" #or a customer Not loo%in" at the
process #rom a customer viewpoint o#ten leads to a narrow de#inition o# the process,
narrow process boundaries $his narrow de#inition o# the process boundaries limits
improvement e##orts because it #ails to study the customer e;perience
1 <hy is it important to study the process #rom the customer-s point o# view>
$he process should be studied #rom the customer-s point9o#9view $rue process
improvement comes #rom a %nowled"e o# what adds value or meanin" #or a customer
Not loo%in" at the process #rom a customer viewpoint o#ten leads to a narrow de#inition o#
the process $his narrow de#inition o# the process limits improvement e##orts because it
#ails to study the customer e;perience
0 <hat is meant by process ownership>
Process ownership re#ers to identi#yin" who is ultimately responsible #or seein" that the process is
completed in a manner that results in customer satis#action $hese individuals are in a position to
and have the power to ma%e chan"es to the process
./ <ho should be involved in process improvement e##orts> How should the team be
structured>
8uy9in Iou want the people who are "oin" to have to live with the new process to be the
ones who #i; it ,# they are involved in identi#yin", creatin", and ma%in" the necessary
chan"es, chances are very "ood that they will live with those chan"es and wor% to ma%e
them permanent ,nvolvement #rom all the %ey activities also brea%s down barriers
between e;istin" departments and provides everyone with a clearer understandin" o# how
wor% "ets done in the or"ani=ation
61
Chapter 22 6ean <ools and <echniAues Questions
. *escribe the "oals o# lean manu#acturin"
Cean manu#acturin" is a mindset best described as a relentless war on waste Some o# the
bene#its o# lean thin%in" include shorter lead times, less handlin", lower costs #or stora"e and
subseBuently #loor space, increased process capability, shorter set9up times, improved
maintenance, less variability, improved communication, improved wor% methods, #aster cycle
time, improved Buality, increased throu"hput, lower inventory, and #ewer customer service
activities
7 How does lean thin%in" support Buality mana"ement>
+##ective Buality mana"ement utili=es sound Buality assurance methods and lean thin%in" to
improve processes :uality assurance techniBues concentrate on reducin" process variation
Cean thin%in" #ocuses on elimination o# waste

6 Review the seven sources o# waste Consider where you wor% and provide e;amples o#
each o# these types o# waste in your or"ani=ation
$he answer should include a discussion o#) overproduction, idle time waste, delivery waste,
waste in the wor% itsel#, inventory, wasted operator motion, and waste o# re?ected parts
5 *escribe the #ive steps o# lean process improvement in your own words
Study the process by directly observin" the wor% activities, their connections and #lows
Study the process to systematically eliminate waste#ul activities, their connections and #lows
+stablish a"reement amon" those a##ected by the process in terms o# what the process needs to
accomplish and how the process will accomplish it
!ttac% and solve problems usin" a systematic method
,nte"rate the above approach throu"hout the or"ani=ation

4 *escribe the Kai=en concept
Kai=en-s "uidin" words are) combine, simpli#y, and eliminate Kai=en see%s to standardi=e
processes while eliminatin" waste Kai=en practitioners "o to the actual wor% area, wor% with
the actual part or service, and learn the actual activities reBuired in the wor% situation $hey then
combine tas%s, simpli#y tas%s and processes, and eliminate any unnecessary activities, processes,
or tas%s
3 <hat is value stream process mappin">
Jalue streams are the actions reBuired to create a product or service #rom raw material until it
reaches the customer Jalue stream maps should #ocus on what is important to meetin"
customer reBuirements Jalue stream process mappin" ma%es the current production situation
60
clear by drawin" the material and in#ormation #lows related to the process ,t provides a clearer
understandin" o# the process by allowin" users to visuali=e the process, reco"ni=e sources o#
waste, and eliminate non9value added activities
2 *escribe each o# the #ive Ss
Seiri) separate Keep only what is essential in the wor%place
Seiton) strai"htenForderliness Put thin"s where they can most easily be used @a%e the
wor%place neat
Seiso) scrubFcleanliness Root out the sources o# waste and uncleanliness Keep the dirt #rom
comin" bac%
Sei%etsu) standardi=e and repeat +stablish a new way o# wor%in"
Shitsu%i) systemati=eFdiscipline @aintain a"reed upon procedures and conditions over time

1 How do Kanban cards wor%>
$he purpose o# Kanban is to mana"e inventory, creatin" only what is needed <hen a part is
needed, the customer 'which may be the ne;t wor%station on the line( issues a Kanban reBuestin"
the production o# the part $he wor%station creatin" the part can only produce as many parts as
they have Kanban cards #or $his eliminates ma%e to stoc% or e;cess inventory
0 *escribe a process, system, or product that you have error9proo#ed
See +;ample .76
./ <hy is preventive and predictive maintenance critical to overall system or process
per#ormance>
Preventive and predictive maintenance see% to prevent problems $hey see% to increase overall
eBuipment e##ectiveness and machine reliability $his means that machines are ready to ma%e
parts when they need to, no une;pected downtime Not only are these machines ready to wor%,
but they are also able to ma%e "ood parts because they are maintained to hold tolerances well
$hey e;perience very little variation when producin" parts
.. <hat tas%s do you per#orm daily> How would you "o about reducin" setup times
involvin" these tas%s> Select a tas%M
Coadin" and unloadin" a dishwasher Unloadin" the bottom level #irst so that any moisture on
the top level doesn-t drip down and wet the bottom layer while unloadin" Puttin" the %nives,
#or%s, and spoons in an or"ani=ed pattern in the silverware tray Placin" the dishwasher, sin%,
and dish stora"e area ne;t to each other to eliminate wal%in" or carryin"
.7 <hat are the bene#its o# producin" smaller batch si=es>
5/
Reduced lot si=es result in shorter lead times, which increases inventory turns, enablin"
continuous #low, reducin" wor%9in9process inventories, reducin" delays, reducin" need #or
stora"e, reduce delays
.6 *escribe what is meant by the term Dline9balancin"E
Cine balancin" occurs when wor% is per#ormed evenly over time with no pea%s or valleys
placin" undue burdens on employees or machines +ach machine and each operator ma%es what
is needed time to match when it is needed
.5 <hat is meant by the term Dta%t timeE
$a%t time is how o#ten a sin"le part should be produced
5.
Chapter 25 "ro!lem Sol&ing <ools and <echniAues Questions
. <hy is a well9written problem statement necessary>
! structured problem9solvin" process allows us to wor% systematically throu"h an issue in
search o# the real reasons behind the issues ! structured problem9solvin" process %eeps the
#ocus on #indin" a true solution ! structured problem9solvin" process insists on chec%in" the
solution, too &ollow the steps shown in the #i"ure in the te;t
7 Create a problem statement How will an improvement team use the problem statement>
$he purpose o# a well9written problem statement is to serve as a "uide #or the problem9solvin"
e##orts ! problem statement helps team members stay #ocused on the tas% at hand Problem
statement) $here is a need to #ind a way to prevent bi%e the#ts on campus
6 Create a Pareto *ia"ram
Customer Satisfaction Survey Results
/
2/
2/
6/
3/
1//
12/

o
l
d

-
o
o
d
1
a
l
a
d

n
o
t
-
r
e
s
h
4
a
c
'

o
f

l
e
a
n
l
i
n
e
s
s
-
l
i
m
s
y
5
t
e
n
s
i
l
s
$
o
o
r
1
e
r
v
i
c
e
-
o
o
d
.
a
s
t
e
s

6
a
d
-
o
o
d
7
r
e
a
s
y
4
a
c
'

o
f

o
u
r
t
e
s
y
Categories
C
o
u
n
t
5 Create a Pareto *ia"ram
Reasons to Phone the Bank
/
8/
1//
18/
2//
28/
3//
38/
2//
6alance , #ther
$ayment
,
.ransfer
all
$ayment
9eceipt ,
-inance
harges ,
Categories
C
o
u
n
t
Once the Pareto chart has been created, the team should #ocus on the most li%ely area in need o#
improvement ,n this case, the si=e o# the DOtherE column is nearly as lar"e as that o# D8alance
57
:uestionsE Steps to use this Pareto dia"ram would include #urther investi"ation into why
DotherE has such a lar"e number o# occurrences, as well as an investi"ation into what can be done
to reduce the number o# customers who have Buestions concernin" their balance
4 Create Pareto *ia"rams
Finish Flaws
/
2
2
6
3
1/
1cratches 1urf ace
-inish
Dent Damage to
asing
Wrong olor
Categories
C
o
u
n
t
s
Operational Flaws
/
2
2
6
3
1/
12
:
o
u
n
t

o
f
f
;
c
e
n
t
e
r
:
o
t
o
r

-
a
i
l
u
r
e
<
o
n
f
u
n
c
t
i
o
n
a
l
(
l
e
c
t
r
i
c
+
c
t
i
v
a
t
i
o
n
1
w
i
t
c
h
:
a
l
f
u
n
c
t
i
o
n
Catagories
C
o
u
n
t
s
Once the Pareto chart has been created, the team should #ocus on the most li%ely area in need o#
improvement ,n this case, the cate"ory D@ountin" Plate Cocation O##9CenterE is lar"er than o#
DScratchesE Steps to use this Pareto dia"ram would include #urther investi"ation into why
D@ountin" PlateE has such a lar"e number o# occurrences ,t would also be interestin" to see
how much each o# these problems cost 'see problem 3(
3 Create a Pareto *ia"ram
56
Costs of Fixing Nonconformity
/
1///
2///
3///
2///
8///
6///
=///
D
a
m
a
g
e

t
o

a
s
i
n
g
1
u
r
f
a
c
e
-
i
n
i
s
h
+
c
t
i
v
a
t
i
o
n
1
w
i
t
c
h
D
e
n
t
:
o
u
n
t

o
f
f
;
c
e
n
t
e
r
Categories
D
o
l
l
a
r
s
0 Create a <hy9<hy *ia"ram
<or%ers not Hasn-t
scheduled at Pea% times been No
pea% times not %nown studied #undin"
$oll9#ree service
<aitin" on ,nsu##icient considered a Unaware o#
the phone operators Cow pay Dnecessary evilE importance
to place an available by mana"ement o# phone orders
order
@any customers Customers- <or%FSchedule
callin" at same time pre#erred orderin"
time
!ll catalo"s
shipped at !lways done No one
same time that way thou"ht
about it
+ctions>
reate measures and document importance of phone orders to bottom line. $resent to management at
monthly meetings.
$rovide funding to study timing of calls.
5tilize information about timing of calls and *ueuing theory to schedule employees
55
./ Create a <hy9<hy *ia"ram
<or%ers not Hasn-t
scheduled at Pea% times been No
pea% times not %nown studied need
reco"ni=ed
Customers ,nsu##icient $i"ht Unaware o#
leave cler%s Cow pay pro#it importance
store without available mar"in o# cler%s
ma%in" purchase to help customers
Uninterestin" Customers- Not
merchandise pre#erence not studied
%nown
!ll merchandise shipped !lways No one
#rom central location done that thou"ht
way about it
+ctions>
reate measures and document importance of customer input leading to increased customer sales
leading to improved bottom line. $resent to management at monthly meetings.
$rovide funding to study customer preferences.
5tilize information about timing of customer pea' times and *ueuing theory to schedule employees.
.. <hat role does a cause9and9e##ect dia"ram play in #indin" a root cause o# a problem>
! cause9and9e##ect dia"ram can help users #ind the root cause o# a problem by #ocusin"
discussions about the problem 8y or"ani=in" their collective thou"hts, team members can
create a clearer picture o# the intricacies o# the issues surroundin" the problem

54
.7 Create a cause9and9e##ect dia"ram
In4ormation Method
Not shared between Not tau"ht Prescribed method to
departments lead customers throu"h call
*etails not available ,nconsistent
,ndividual
operator styles

Not always correct Unclear
Non9responsive
En&ironment :hat ma7es
a customer
Uncom#ortable (aitB
Poorly lit
Sta##in" Part timers
So#tware
$ime o## $oo many screens
Cac% o# Slow to search #or in#o
e;perience Comple;
*oesn-t
Cimited $rainin" provide Old computers
all in#o slows process
needed
"eople EAuipment
.5 Create a histo"ram
R N Oh P Ol N /3404 9 /3464 N ///3
@Pl N /3464
///3
$ry i N ////6A h N LLLL H . N 7.
////6
///3
$ry i N ////4A h N LLLL H . N .6
////4
53
///3
$ry i N ////2A h N LLLL H . N ./
////2
///3
$ry i N ////0A h N LLLL H . N 1 Use
////0
i ////0
$hen /3464 P LLL N /3464 P LLLL N /346/4 etc
7 7
Cell 8oundaries Cell @pts &reBuency
/346/4 9 /34604 /3464 .
/34604 9 /34514 /3455 .1
/34514 9 /34424 /3440 .
/34424 9 /34334 /3437 /
/34334 9 /34244 /342. 6
/34244 9 /34154 /341 1
/34154 9 /34064 /3410 1
/34064 9 /33/44 /3401 6
.4 Create a histo"ram
R N Oh P Ol N //7/ 9 ///7 N //.1
@Pl N ///7
//.1
$ry i N ///6A h N LLLL H . N 2
///6
//.1
$ry i N ///4A h N LLLL H . N 4 Use
///4
52
//.1
$ry i N ///2A h N LLLL H . N 5
///2
i ///4
$hen @Pl P LLL N ///7 P LLL N / etc
7 7
Cell 8oundaries Cell @pts &reBuency
// 9 ///54 ///7 3
///54 9 ///04 ///2 5
///04 9 //.54 //.7 .7
//.54 9 //.04 //.2 3
//.04 9 //754 //77 5

.3 Create a histo"ram
R N Oh P Ol N .4/ P .74 N 74
@Pl N .74
74
$ry i N 6A h N LLLL H . N 0
6
74
$ry i N 4A h N LLLL H . N 3 Use
4
74
$ry i N 2A h N LLLL H . N 4
2
i 4
$hen @Pl P LL N .74 P LL N .774 etc
7 7
Cell 8oundaries Cell @pts &reBuency
51
.774 P .724 .74 7
.724 P .674 .6/ .7
.674 P .624 .64 .1
.624 P .574 .5/ 76
.574 P .524 .54 63
.524 P .474 .4/ .4
.2 Create an O9bar and R chart centerline and control limits
n N 4
O*8 N .3
R9bar N 2
UCC; N .3 H '/422( '2( N 7/ UCCr N 7..4 '2( N .4
CCC; N .3 P '/422( '2( N .7 CCCr N '/( 2 N /
.1 Create an O9bar and R chart centerline and control limits
n N 5
O*8 N 4/7
R9bar N /31
UCC; N 4/7 H '/270( '/31( N 4/2 UCCr N 7717 '/31( N .3
CCC; N 4/7 P '/270( '/31( N 502 CCCr N '/( /31 N /
.0 Create an O9bar and R chart centerline and control limits
n N 1
O*8 N 6/7
R9bar N /./
UCC; N 6/7 H '/626( '/./( N 6/3 UCCr N .135 '/./( N /.0
CCC; N 6/7 P '/626( '/./( N 701 CCCr N /.63 '/./( N //.
7/ ,nterpret the charts
8oth charts are e;hibitin" a lot o# variation On the O9bar chart, the points ?ump bac% and #orth
across the centerline Hal#9way throu"h the chart, #our points are below the centerline and then
50
travel bac% to the centerline $he points do not #loat #reely bac% and #orth across the centerline
$he points late in the chart are closer to"ether in value On the R chart, two o# the points './ and
.5( e;hibit more variation than the other points $here are no patterns, runs, or trends on either
chart
7. *escribe how an O9bar and R chart would loo% i# they were under normal statistical control
!n O9bar and R chart under normal statistical control e;hibits the #ollowin" characteristics
8oth charts will have no patterns, trends, or runs $here will be a #ew points on or near the
centerline No points will be beyond the control limits $wo9thirds o# the points will be near the
centerline $he points will be balanced on both sides o# the centerline and the points will move
#reely bac% and #orth across the centerline
77 How would accuracy and precision be reco"ni=ed on control charts>
!ccuracy is reco"ni=able on the O9bar chart by notin" how the avera"es o# the sample data are
clustered around the centerline Precision is reco"ni=able on the R chart by notin" the ma"nitude
o# the avera"e R values
76 <hy is the use and interpretation o# an R chart so critical when e;aminin" an O9bar chart>
$he use and interpretation o# an R chart is critical when e;aminin" an O9bar chart because the R
chart allows the user to see the spread o# the data $he charts show i# the spread o# the data is
reasonable #or the measurements on the O9bar chart $he R chart shows the repeatability o# the
process
75 Calculate 3 si"ma, Cp and Cp%
R9bar 2
Si"ma N LLLL N LLL N 6
d7 7673
3Si"ma N .1
7. P .6
Cp N LLLL N /55
.1
.
Cp% N LLL N /66
6
<here)
4/
7. P .3
Q'u( N LLLL N .33
6
.3 P .6
Q'l( N LLLLL N .
6
$he process is not capable and not centered


74 Calculate 3 si"ma, Cp and Cp%
R9bar ////.5
Si"ma N LLLL N LLLL N /////3
d7 7673
3Si"ma N ////63
/4/62/ P /4/66/
Cp N LLLLLLLLL N ..
////63
6.2
Cp% N LLLL N ..
6
<here)
/4/62/ P /4/650
Q'u( N LLLLLLLLL N 64
/////3
/4/650 P /4/66/
Q'l( N LLLLLLLLL N 6.2
/////3
$he process is capable and centered

4.
73 Calculate 3 si"ma, Cp and Cp%
R9bar /./
Si"ma N LLLL N LLLL N //5
d7 7152
3Si"ma N /75
6/4 P 704
Cp N LLLLLL N /57
/75
/1
Cp% N LLLL N /72
6
<here)
6/4 P 6/7
Q'u( N LLLLLL N /1
//5
6/7 P 704
Q'l( N LLLLLL N .1
//5
$he process is not capable and not centered


47

72 Calculate 3 si"ma, Cp and Cp%
R9bar .33
Si"ma N LLLL N LLLL N /1.
d7 7/40
3Si"ma N 515
0/ P 2/
Cp N LLLLL N 5.6
515
557
Cp% N LLLL N .52
6
<here)
0/ P 2641
Q'u( N LLLLLL N 7/72
/1.
2641 P 2/
Q'l( N LLLLLL N 557
/1.
$he process is capable but not centered
46
Chapter 2+ "ro=ect Management Questions
. <hy are pro?ect mana"ement s%ills important to apply in order to be e##ective>
Pro?ect mana"ement s%ills allow us to approach a dauntin" pro?ect and #i"ure out where to
be"in and where to #ocus $hey also help us determine i# there will be enou"h time Pro?ects
consume time and e##ort outside o# these day9to9day activities and employees must utili=e
"ood pro?ect mana"ement s%ills in order to inte"rate their pro?ect wor% with re"ular
activities Knowled"e and s%ills in the area o# pro?ect mana"ement are necessary any time an
employee can answer yes to the #ollowin" Buestions)
*o assi"nments have to be complete by a speci#ied deadline>
*o several tas%s have to be accomplished durin" the day>
,s there a limited set o# resources with which to complete these tas%s>
,s the involvement o# other people necessary in order to complete the wor%>
*o supervisors, collea"ues, or customers ever chan"e their mind about what they want>
*oes the team have a clear idea o# what they are tryin" to accomplish #or the ultimate
user>
*oes the team understand the perspectives o# the people a##ected by the pro?ect>
!re the constraints and directives that "overn the pro?ect %nown>
,s the pro?ect bro%en down into mana"eable chun%s>
<hen disa"reements arise, does the team have the s%ills to build e##ective a"reements>
6 <hy is completin" a pro?ect li%e eatin" an elephant>
Pro?ects are lar"e, ?oinin" to"ether a multitude o# smaller tas%s $heir si=e can be as
intimidatin" as eatin" an elephant, on the sur#ace appearin" to be too much to
accomplish
5 How is a pro?ect selected to be wor%ed on>
+##ective or"ani=ations select pro?ects based on the pro?ect-s ability to contribute to one or all
three o# the #ollowin") customer perceived value and satis#action, the or"ani=ation-s #inancial
stren"th, or operational necessities &or customer9#ocused or"ani=ations, many pro?ects will
be selected based on their ability to increase customer value, satis#action, and retention
Pro?ects may be chosen in order to enable the or"ani=ation to maintain its competitive ed"e
$hese pro?ects may involve the development o# a new product or service or an e;tension to a
product line or the development o# a product or service enhancin" #eature $hese pro?ects
will ultimately enhance an or"ani=ation-s #inancial success Some pro?ects are operational
necessities li%e meetin" "overnment re"ulations or the repair or replacement o# a"in"
eBuipment Re"ardless o# the reasons behind a pro?ect bein" selected, e##ective or"ani=ations
reco"ni=e that a pro?ect must be #inancially sound and provide a paybac% #or their
investment
Pro?ects can be identi#ied throu"h the use o# customer #eedbac%, internal customer #eedbac%
#rom the people doin" the wor%, competitive benchmar%in", e;istin" measures o#
45
per#ormance, or costs o# poor Buality studies !ny "ap that e;ists between desired
per#ormance and actual per#ormance is a candidate #or a pro?ect Pro?ects should
concentrate on a problem area #or either the internal or e;ternal customer ! #ocused pro?ect
will have boundaries and will not be so lar"e as to become unmana"eable Pro?ects have
problem statements that are speci#ic, measurable, and identi#y the "ap between the desired
and actual per#ormance
4 <hat are the components o# an e##ective pro?ect proposal>
!n e##ective pro?ect proposal provides readers with insi"ht into what needs to be
accomplished and how it will "et accomplished $hrou"h the use o# clearly stated mission,
deliverables, "oals, and ob?ectives associated with the pro?ect, proposals sell the pro?ect !s
an introduction, proposals provide bac%"round in#ormation about the need #or the pro?ect
$hey contain a description or overview o# the e;pectations o# the pro?ect, includin" details
about the technical aspects o# the pro?ect +ssential tas%s are outlined and delineated !
thorou"h pro?ect proposal will contain in#ormation concernin" #inancial reBuirements, time
constraints, and administrative and lo"istical support #or the pro?ect ,n the proposal there is
usually in#ormation about the %ey individuals associated with the pro?ect, includin" the
identity o# the pro?ect mana"er 8asic areas o# per#ormance responsibility are assi"ned
$entative schedules and bud"ets are established $he #i"ure in the te;t "ives a brie# summary
o# the typical components o# a pro?ect proposal
$he pro?ect proposal creates a "eneral understandin" o#)
9what is needed,
9what is "oin" to be done,
9why it is "oin" to be done,
9who is "oin" to do it,
9when it will be done,
9where it will be done,
9how it will be done
2 <hat are the components o# an e##ective pro?ect plan>
Pro?ect plans are si"ni#icantly more detailed than pro?ect proposals Pro?ects have three
interrelated ob?ectives) meetin" the bud"et, #inishin" on schedule, and meetin" the
per#ormance speci#ications set by the client ! "ood pro?ect plan enables an or"ani=ation to
accomplish all three $hou"h a pro?ect plan may be modi#ied several times durin" a pro?ect,
e##ective pro?ect plans remain %ey to or"ani=ational success because no pro?ect always "oes
accordin" to plan, but well planned pro?ects are less li%ely to "o astray Pro?ect plans provide
the in#ormation about)
9@ission and deliverables
9Speci#ic "oals and ob?ectives supportin" the mission and its deliverables
9$as%s reBuired to meet the "oals and ob?ectives
9$echnicalities o# who, what, where, when, why, and how
9Schedules, the time needed to support each aspect o# the plan
44
9Resources, what is needed to support each aspect o# the plan
9Cost analysis
9Jalue analysis
9Personnel, who is needed to support each aspect o# the plan
9Personnel, responsibilities and assi"nments
9+valuation measures #or %eepin" the pro?ect on trac%
9Ris% analysis, what could "o wron" and how will it be dealt with>
9Pro?ect chan"e mana"ement process
$he #i"ure in the te;t "ives a brie# summary o# the typical components o# a pro?ect plan
./ <hat di##erentiates a Gantt chart #rom a P+R$ chart #rom a CP@>
! P+R$ chart improves upon a Gantt chart by showin" the relationships between tas%s
Unli%e the Gantt chart, which is a list o# tas%s, the P+R$ chart enables the pro?ect to be
viewed as an inte"rated whole 8ecause it coordinates and synchroni=es many tas%s, it is
well desi"ned to handle comple; pro?ects
.. ! not9#or9pro#it or"ani=ation is interested in buyin" caramels and sellin" them to raise
money Create a P+R$ chart #or the #ollowin" in#ormation
:ee7s Cost
<as7 "redecessor 'ormal Crash 'ormal Crash
*esi"n !ds 'a( 9 7 7 R74/ /
Order Stoc% 'b( 9 .7 1 R7// R64Fw%
Or"ani=es Sales People 'c( 9 3 6 R.7/ R3/Fw%
Place !ds 'd( a 6 7 R74 R./Fw%
Select *istribution Sites 'e( c 5 6 R.// R7/Fw%
!ssi"n *istribution Sites '#( c, e 5 6 R.// R./Fw%
*istribute Stoc% to Sales People '"( b, c, e, # 7 . R.// R74Fw%
Sell Caramels 'h( d, " 4 6 R.// R5/Fw%
43
.. 'cont(
7
*esi"n Place
ads ads
*ist stoc% Sell caramels
. 3 6 2
Order stoc%
Or" !ssi"n
sales Sites
5 4
Select
sites
Paths) .979692
.939692
.9594939692
.7 Complete a CP@ #or the in#ormation in Buestion ..
7
*esi"n Place
ads ads
*ist stoc% Sell caramels
. 3 6 2
Order stoc%
Or" !ssi"n
sales sites
5 4
Select
sites
Paths) .979692 N ./
.939692 N .0
.9594939692 N 7. Critical Path
42
.6 ,# the leaders o# the pro?ect completed in Buestion .7 wanted to speed up their pro?ect by
three wee%s, what would be the most cost e##ective way o# accomplishin" that>
Paths
.979692 N ./
.939692 N .0
.9594939692 N 7. Critical Path
Reduce Critical Path #irst ,t costs section)
'c( .95 R3/Fwee%, 6 wee%s available
'e( 594 R7/Fwee%, . wee% available
'#( 493 R./Fwee%, . wee% available
'"( 396 R74Fwee%, . wee% available
'h( 692 R5/Fwee%, 7 wee%s available
Ceast costly to speed up) crash '#( #or R./
crash 'e( #or R7/
crash '"( #or R74
#or a total crash cost o# R44 #or the three wee%s
.5 <hat does it mean to %eep a pro?ect under control> How is a pro?ect controlled>
$hrou"hout a pro?ect, e##ective pro?ect mana"ers monitor the pro"ress a pro?ect is ma%in"
toward completion Per#ormance, cost, and time, the three aspects o# a pro?ect, all need to be
monitored and controlled in order to ensure pro?ect success Pro?ect control and monitorin"
involves "atherin" and appraisin" in#ormation on how the pro?ect-s activities compare with
the pro?ect plan !ctual pro"ress is trac%ed a"ainst the per#ormance measures established in
the pro?ect plan $hese per#ormance measures help a pro?ect mana"er assess how time,
money, and other resources have been used to produce the e;pected outcomes Costs can
also "et out o# control when the pro?ect costs are not watched closely and corrective cost
control was not e;ercised in time 8y closely monitorin" the per#ormance measures
associated with the pro?ect, an alert pro?ect mana"er can be prepared to respond Buic%ly to
deviations in order to %eep the pro?ect on trac% and under control $hou"h very #ew pro?ects
have not had their "oals and ob?ectives modi#ied in some way or another #rom their
be"innin" to the end, care#ul pro?ect control enables a pro?ect mana"er to minimi=e the
e##ects o# these chan"es on the overall pro?ect
.4 <hat are contin"ency plans> <hy is it important to have contin"ency plans>
41
Pro?ects, due to their very nature, are comple; &or every pro?ect, there is a ris% o# #ailure
Contin"ency plans are created to ensure that the pro?ect team is ready to handle potential
problems <hile all problems cannot be #oreseen, a pro?ect plan that includes contin"ency
plans %eeps the team #le;ible and aware that they may be as%ed to ma%e ad?ustments to their
pro?ect plan some time durin" its li#etime
.3 <hat is a chan"e control system> How are they structured> <hat are they used #or>
+##ective pro?ect mana"ers reco"ni=e that clients ma%e chan"es to the pro?ect as it
pro"resses !ll pro?ect proposals and plans should contain a description o# how reBuests #or
chan"es in the pro?ect-s plan, bud"et, schedule, or per#ormance deliverables will be handled
!n e##ective chan"e control system will have steps in place that review the reBuested chan"es
#or both content and procedure and identi#y how the chan"e will impact the pro?ect $his
impact must be re#lected in ad?ustments to the pro?ect-s per#ormance ob?ectives, the schedule,
and bud"et Once accepted, chan"e orders become part o# the overall pro?ect plan Part o#
the ?ob o# a chan"e control system is to clearly communicate any chan"es to any person or
part o# the pro?ect a##ected by the chan"e $he best way to ensure that this critical
communication occurs is to have all chan"es approved in writin" by all appropriate
representatives o# the impacted areas Ultimately, the chan"e should only be made i# its
bene#its outwei"h the costs o# implementin" the chan"e
.2 <hat does it ta%e to be an e##ective pro?ect mana"er>
!n e##ective pro?ect mana"er achieves the desired results within bud"et and on time and
accordin" to the desired standards +##ective pro?ect mana"ers reali=e that in order to
accomplish what needs to be done on time and within bud"et, they must ta%e time to plan
their pro?ects Once a "ood plan has been created, e##ective pro?ect mana"ers mana"e
their plan
Unli%e #unctional mana"ers, pro?ect mana"ers are "eneralists with %nowled"e and
e;perience in a wider variety o# areas ! pro?ect mana"er is responsible #or or"ani=in",
directin", plannin", and controllin" the events associated with a pro?ect $hey deal with
bud"ets and schedules Responsibility #or the pro?ect rests on their shoulders and they
must understand what needs to be done, when it must be done, and where the resources
will come #rom $hrou"hout a pro?ect, the mana"er will be the one who must clari#y
misunderstandin"sA calm upset clients, leaders, and team membersA and meet the client-s
demands while %eepin" the pro?ect on time and within bud"et Pro?ect mana"ers are
responsible #or #indin" the necessary resources, motivatin" personnel, dealin" with
problems as they arise, and ma%in" pro?ect "oal trade9o##s ,n essence, an e##ective
pro?ect mana"er is an individual who does whatever is necessary to %eep the pro?ect on
schedule, within bud"et, and able to meet per#ormance e;pectations Pro?ect mana"ers
must be prepared to ma%e ad?ustments to schedules, bud"ets, and resources in order to
deal with the une;pected &or this reason, they must be "ood at reco"ni=in" the early
si"ns o# problems and be able to cope with stress#ul situations !s discussed under the
40
headin" o# pro?ect plans, e##ective pro?ect mana"ers utili=e the chec%points, activities,
and time estimates established in the pro?ect plan to "uide those wor%in" on the pro?ect
&ollowin" a clearly laid out pro?ect schedule, with clearly delineated responsibilities,
enables e##ective pro?ect mana"ers to %eep their pro?ects on trac% in terms o# time,
per#ormance, and cost Clear pro?ect plans enable the e##ective pro?ect mana"er to direct
people individually as well as a team Pro?ect mana"ers mana"e people as well as
pro?ects $o do this, e##ective pro?ect mana"ers schedule #reBuent pro"ress reports
$hese meetin"s allow the pro?ect mana"er to react Buic%ly when he or she reco"ni=es that
a di##iculty has arisen
3/
Chapter 2, %esults Questions
. How does auditin" allow e##ective or"ani=ations to see their pro"ress>
Studyin" current per#ormance levels, trends over time and comparison data #or the %ey
measures and indicators allows an or"ani=ation to see what is wor%in" and what is not
!udits are desi"ned to appraise the activities, practices, records, or policies o# an
or"ani=ationA they determine whether a company has the ability to meet or e;ceed a
standard !udits provide many bene#its, includin" allowin" an or"ani=ation to see what
is really ta%in" place $his insi"ht can enable the or"ani=ation to ta%e action be#ore
potential problems become serious issues ,n#ormation provided by the audit can point to
where action is necessary in order to contain the problem !udits enable or"ani=ations to
see which processes, policies, procedures, and practices are e##ective and which are not
7 <hat do e##ective or"ani=ations audit>
! variety o# circumstances can initiate an audit !udit pro"rams may be part o# meetin"
customer contract reBuirements or "overnment re"ulations !udits do not have to have
an outside insti"atorA it is not unusual to see a company utili=e internal auditin" systems
to veri#y its own per#ormance to %ey metrics !udits may also e;amine aspects o#
eBuipment, so#tware, documentation, and procedures !udits o# systems such as material
handlin" or accounts receivable can reveal poor practices that need to be improved
Supplier Buality and record9%eepin" practices can also be chec%ed ,nternal and supplier
audits allow a company the opportunity to veri#y con#ormance to speci#ications and
procedures Since audits identi#y opportunities #or improvement, companies may
per#orm a product or service inte"rity audit to veri#y that the process is per#ormin" in an
optimal #ashion Usin" audits to identi#y process problems reduces opportunities #or
noncon#ormities <hatever the reason, audits provide companies with in#ormation
concernin" their per#ormance, the per#ormance o# their product or service, and areas #or
improvement !udits help or"ani=ations determine the answers to a variety o# Buestions
includin")
Have value9driven improvements been made>
Have costs o# Buality been reduced>
Have lead times been reduced>
How do the measures o# per#ormance measure up>
,s the company achievin" its ob?ectives>
!re the correct procedures bein" #ollowed>
!re new and more e##icient methods o# per#ormance documented and used where
applicable>
!re records bein" properly retained and used to solve production problems>
!re preventive maintenance schedules bein" #ollowed>
6 <hat do e##ective or"ani=ations do with the in#ormation when an audit is completed>
3.
<hether the audit is internal or e;ternal, the recommendations o# the auditors are critical
to an or"ani=ation see%in" to improve ,t is leadership-s response to these results that
matters <hen discrepancies between the actual and desired per#ormance are uncovered,
it is up to leadership to e;amine the "ap and create and implement a plan o# action to deal
with the "ap
5 +mployees at a local ban% have been trained to be polite and ea"er to serve customers $he
ban% has been able to identi#y several other #eatures that are important to their customers)
timeliness, accuracy, immediate service, %nowled"e, courtesy, convenience, and accessibility
$he ban% already conducts re"ular #inancial audits in order to comply with "overnment
re"ulations, however, they are considerin" auditin" the remainin" aspects o# their ban% in order
to determine how they can improve $he #ollowin" are some Buestions they have about auditin"
Provide answers as i# you were a consultant helpin" them with their audit
a 8esides the #inancial area, what areas would be appropriate to audit>
b *escribe the steps the ban% should ta%e to complete an audit <hat should ta%e place
durin" each step>
c !#ter completin" the audit, what should the ban% do with the in#ormation>
d <hat bene#its would auditin" various areas within the ban% provide #or the ban%>
4 <hat areas would a manu#acturin" company be interested in auditin"> ! hospital> !n ,nternet
company>
3 *escribe the steps involved in auditin"
$ypical auditin" pro"rams include a plannin" phase, the actual audit, reports recommendin"
improvement, and #ollow9up action plans
. (lan) $o be"in, those plannin" the audit need to identi#y its purpose or ob?ective ! statement
o# purpose clari#ies the #ocus o# the audit &ollowin" this, planners will need to identi#y the who,
what, where, when, why, and how related to the audit
9<ho is to be audited>
9<ho is to per#orm the audit>
9<hat does this audit how to accomplish>
9<hat are the per#ormance measures>
9<here the audit will be conducted>
9<hen will the audit be conducted>
9<hy is the audit bein" conducted>
9,s the audit to ?ud"e con#ormance to standards>
9<hat are the critical standards>
9How will the audit be conducted>
$hose about to be audited should be in#ormed by an individual in a position o# authority Clear
statements o# the reasons behind the audit 'why(, the per#ormance measures 'what(, and the
procedures 'how( should be "iven to those bein" audited
37
7 Do) Usin" the in#ormation clari#ied in the plannin" phase, the audit is conducted O#ten an
introductory meetin" is held by the participants to discuss the scope, ob?ectives, schedule, and
paperwor% considerations !#ter the openin" meetin", e;aminers be"in the process o# reviewin"
the process, product, or system under study !uditors may reBuire access to in#ormation
concernin" Buality systems, eBuipment operation procedures, preventive maintenance records,
inspection histories, or plannin" documents !uditors may conduct interviews with those
involved in the process o# providin" a product or service !ny and all in#ormation related to the
area under study is critical #or the success o# the audit *urin" the review process, auditors
document their #indin"s $hese #indin"s are presented in a "eneral summary at a closin" meetin"
o# the participants <ithin a short period o# time, perhaps ./ to 7/ days, the auditors will
prepare a written report that documents their #indin"s, conclusions, and recommendations
6 Study) !udits provide in#ormation about the participant-s stren"ths, wea%nesses, and areas #or
improvement Upon receipt, the auditor-s report is read by the participants in the audit *urin"
this phase o# the audit cycle, they respond to the report and develop an action plan based on the
recommendations o# the auditors $his action plan should speci#y the actions, assi"n
responsibilities, and provide a time #rame reBuired to deal with the issues raised by the audit
5 Act) Once adopted, the action plan becomes the #ocus o# the improvement activities related to
the audited area !uditors and company administrators should #ollow up at predetermined
intervals to evaluate the status o# the continuous improvement action plan $his ensures that the
recommendations and conclusions reached by the auditors, and supported by an action plan,
assist the company in reachin" its continuous improvement "oals
2 <hy would a company be interested in auditin"> <hat does a company hope to "ain by
auditin">
Ceadership ta%es the recommendations o# the auditors seriously %nowin" that improvements are
critical to the or"ani=ation-s #uture success <here the audit has revealed discrepancies between
the actual and desired per#ormance, leadership will want to e;amine the "ap and determine and
implement a plan o# action to deal with the "ap !uditin" reveals where improvements can be
made Creatin" responses to these needs #or improvement will ma%e the or"ani=ation much
stron"er #or the #uture Ceaders reco"ni=e that in the short term, they will be incurrin" many
costs to set the current situation ri"ht, but lon" term pro#its and competitiveness will improve
36
1ns(ers to Selected "ro!lems
1ppendiC ,


35
. n N 7//
Sn N 75//
Snp N ./
./
p9bar N LLLL N ///5
75//
///5'. P ///5(
UCCp N ///5 H 6 T LLLLLLL N //.25
7//
///5'. P ///5(
CCCp N ///5 P 6 T LLLLLLL N /
7//
7 n N .4/
Sn N 54//
Snp N .26
.26
p9bar N LLLL N //61
54//
//61'. P //61(
UCCp N //61 H 6 T LLLLLLL N //14
.4/
//61'. P //61(
CCCp N //61 P 6 T LLLLLLLL N /
.4/
34
5 n N .//
Sn N 74//
Snp N 6//
6//
np9bar N LLLL N .7
74
6//
p9bar N LLLL N /.7
74//

UCCnp N .7 H 6 T.7'. P /.7( N 77

CCCp N ///5 9 6 T.7'. P /.7( N 7
$here is one point beyond upper control limit $wo9thirds o# the points are near the centerline
! #ew points are on or close to the centerline $he points are balanced and they #loat bac%
and #orth across the centerline
4 n N 6//
Sn N 3//
Snp N 756
756
np9bar N LLLL N .7 Process Capability
7/
756
p9bar N LLLL N //5/4
3///

UCCnp N .7 H 6 T.7'.P //5/4( N 77
33
CCCp N .7 P 6 T.7'. P //5/4( N 7
3
72
c9bar N LL N .
74
UCCc N . H 6T. N 5
CCCc N . P 6T. N /
$he line ?ust meets speci#ications
2
..6
c9bar N LLL N 3
7/
UCCc N 3 H 6T3 N .6
CCCc N 3 P 6T3 N /
1
530
u9bar N LLL N 7654
7//
7654
UCCu N 7654 H 6T LLL N 6
7/
32

7654
CCCc N 7654 P 6T LLLL N .
7/
0
21/
u9bar N LLL N //01
1///
//01
UCCu N //01 H 6T LLL N /.54
5//
//01
CCCc N //01 P 6T LLL N //4.
5//
31
est Bank
30
Potential est !uestions
+nswers serve only as e"amples. -or many *uestions! several answers are possible.
Chapter " Organi#ational $ffectiveness
1. What is the difference between being efficient and being effective?
+##icient) producin" a result with a minimum amount o# e##ort, e;pense, or waste
+##ective) producin" a desired result e##iciently
+##ective companies produce what their customers want when they want it with a minimum
amount o# e##ort, e;pense, and waste
2. ?ou are about to interview for a full;time @ob with a company of your choice. 0ow will
you recognize that this company is following a .,:Acontinuous improvement
philosophy? 0ow do you 'now they are? 6e sure to sight specific e"amples of the
evidence you would loo' forB
Coo% to see i# they have any o# the #ollowin" systems in place) Customer &ocus, Si; Si"ma,
Cean @anu#acturin", Jalue Chain @ana"ement, !ctive Ceadership, &ocus on 8usiness Results,
Strate"ic Plannin", ,n#ormation and Knowled"e @ana"ement, :uality !ssurance,
Or"ani=ational Philosophy #ocused on :uality, Pro?ect @ana"ement, Jalue +n"ineerin", +##orts
to +liminate o# <aste, $eams, Human Resources @ana"ement, Just9in9time operations, :uality
$ools, Process @ana"ement ,mprovement
3. Why is an effective organization more competitive?
!n e##ective or"ani=ation is able to produce more with its e;istin" resources throu"h an
improved customer #ocus and streamlined wor% processes <ith its increased awareness o# its
internal and e;ternal customers, there is a "reater #ocus on what really needs to be accomplished
in order to meet their customers- needs and e;pectations !s the introduction to this chapter
showed, an or"ani=ation that increases its e##ectiveness will see improvements in their
pro#itability throu"h increased customer retention 8ein" able to meet customer e;pectations the
#irst time and every time will enable the or"ani=ation to increase its mar%et share as new
customers see% them out Since an e##ective or"ani=ation has #ocused and streamlined its wor%
processes they will bene#it #rom lower costs because o# reduced waste and rewor% One o# the
ma?or savin"s that occurs is #ewer customer complaints and warranty claims @ore satis#ied
customers results in "reater mar%et share ,mprovements will also e;ist internal to the
or"ani=ation 8ecause o# improved communication and teamwor%, e##ective or"ani=ations have
"ood mana"ement9employee relations !s problems are solved and the or"ani=ation be"ins to
run more smoothly, employee involvement and satis#action will increase, which will lower
turnover and absenteeism 8ene#its o# creatin" and maintainin" an e##ective or"ani=ation are
summari=ed in the #i"ure in the te;t
2. 0ow would you verify that an organization is operating effectively?
2/
+##ective or"ani=ations e;hibit improved pro#itabilityA increased customer retentionA reduced
customer complaints and warranty claimsA reduced costs throu"h less waste, rewor%, and so onA
achieve a "reater mar%et shareA increased employee involvement and satis#actionA lower
employee turnoverA an increased ability to attract new customers, improve their competitiveness,
and customer satis#actionA improved mana"ement9employee relationsA improved #ocus on %ey
"oalsA improved internal and e;ternal communicationA and improved teamwor%
8. Define organizational effectiveness.
+##ective companies produce what their customers want when they want it with a minimum
amount o# e##ort, e;pense, and waste +##ective or"ani=ations respond to their customers-
e;pectations by #ocusin" their company-s value chain on providin" Buality products and services
#or their customers +##ective or"ani=ations wor% to provide "reater value to their customers,
#indin" new sources o# customer deli"ht more rapidly than their competitors !n e##ective
or"ani=ation concentrates on what is important) meetin" both their internal and e;ternal
customers- needs and reasonable e;pectations, encoura"in" teamwor% and cooperation, trac%in"
%ey indicators o# per#ormance, maintainin" a lon"9term #ocus on continuous improvement,
ma%in" decisions based on #acts, and #indin" solutions, not #ault Such #ocus means that
everyone is involved in the process o# creatin" and maintainin" an e##ective or"ani=ation
6. ite an e"ample of an effective organization you are aware of. 0ow do you 'now it is
more effective than its competitors?
1ee answers to C2! 3! 2 and 8
2.
Chapter % Organi#ational Philosophy
1. 6ased on your own e"perience! describe a situation where one of Deming%s fourteen
points is applicable and why things would have gone smoother if the participants would
have utilized this particular point.
See &i"ure in the te;t
2. Deming! Duran! and rosby all believe in striving toward world;class *uality. (ach
gentleman has his own approach to reach such a goal. Discuss one area where
Deming! Duran! and rosby all agree. .his point must be bac'ed;up by specific aspects
from each individual%s *uality improvement plans.
See &i"ure in the te;t Notice the similarities and di##erence in their de#initions o# Buality !lso,
all three believed in mana"ement commitment bein" absolutely necessary *emin" and Juran
both had systems #or improvement, P*S! and Juran-s $rilo"yFJourney #rom Symptom to Cause
'&i"ure 7./ and $able 77( !ll three believed in continuous improvement, *emin" 'P*S! and
&ourteen Points(, Juran '$rilo"yFJourney #rom Symptom to Cause '&i"ure 7./ and $able 77(,
and Crosby with his Buality per#ormance standard o# Qero *e#ects and the Buality system o#
Prevention o# *e#ects
3. What does Deming%s first point Ecreate a constancy of purpose toward improvement
of product and service! with the aim to become competitive and to stay in business and
to provide @obsF mean?
*r *emin"-s #irst pointLcreate a constancy of purpose toward improvement of product and
service, with the aim to $ecome competitive and to stay in $usiness and to provide )o$sL
encoura"es leadership to accept the obli"ation to constantly improve the product or service
throu"h innovation, research, education, and continual improvement in all #acets o# the
or"ani=ation ! company is li%e an Olympic athlete who must constantly train, practice, learn,
and improve in order to attain a "old medal Cac% o# constancy o# purpose is one o# the deadly
diseases *r *emin" warns about in his writin"s <ithout dedication, the per#ormance o# any
tas% cannot reach its best
2. Which of Deming%s fourteen points do you agree with? Why?
See &i"ure in te;t
8. Which of Deming%s fourteen points do you disagree with? Why?
See &i"ure in te;t
6. rosby describes five erroneous assumptions about *ualityG select one and argue
why it is an erroneous assumption.
27
*r &ei"enbaum-s de#inition o# Buality mentions the word intan"ible 8y discussin" #ive
erroneous assumptions about Buality, Crosby attempted to ma%e Buality more understandable and
there#ore, tan"ible $he #irst erroneous assumption, Buality means "oodness, or lu;ury, or
shininess or wei"ht, ma%es Buality a relative term Only when Buality is de#ined in terms o#
customer reBuirements can Buality be mana"eable $he second incorrect assumption about
Buality is that Buality is intan"ible and there#ore not measurable ,# ?ud"ed in terms o#
D"oodness,E then Buality is intan"ibleA however, Buality is measurable by the costs o# doin"
thin"s wron" @ore precisely, Buality costs involve the cost o# #ailures, rewor%, scrap,
inspection, prevention, and loss o# customer "oodwill
Closely related to the #irst two assumptions is the third, which states that there e;ists Dan
economics o# BualityE Here a"ain, one errs in thin%in" that Buality means buildin" lu;uries into
a product or serviceA rather, Buality means that it is more economical to do thin"s ri"ht the #irst
time O#ten wor%ers are blamed #or bein" the cause o# Buality problems $his is the #ourth
erroneous assumption about Buality <ithout the proper tools, eBuipment, raw materials, and
trainin", wor%ers cannot produce Buality products or services @ana"ement must ensure that the
necessary items are available to allow wor%ers to per#orm their ?obs well $he #inal erroneous
assumption that Crosby discusses is that Buality ori"inates in the Buality department !ccordin"
to Crosby, the Buality department-s responsibilities revolve around educatin" and assistin" other
departments in monitorin" and improvin" Buality
=. 5se e"amples to describe the difference between Duran%s 6ig , and 4ittle *.
See &i"ure in te;t
3. ?ou are wor'ing for a company that manufactures bicycles. +t this moment! you are
attending a meeting for the purpose of setting the direction of the company for the ne"t
five years. 6ecause of your *uality management training! you would li'e to see that
direction include an emphasis on the overall *uality of the organization. olleagues
attending the meeting have as'ed you to e"plain the difference between Duran%s 6ig ,
and 4ittle *. $lease e"plain the difference to your colleagues in terms of a bicycle
manufacturing company.
His de#inition o# Buality "oes beyond the immediate product or moment o# service $o *r
Juran, Buality is a concept that needs to be #ound in all aspects o# a businessA leaders must
mana"e #or Buality $o more clearly communicate this concept, *r Juran contrasts bi" : and
little B to show the broad applicability o# Buality concepts, as shown in &i"ure in te;t Note that
bi" : e;tends to all aspects o# any or"ani=ation, re"ardless o# type
26
Chapter & !uality Systems
1. 0ow are &1# )/// and ,1 )/// related?
!mon" motor vehicle manu#acturers, thou"h each purchaser developed their own reBuirements,
stron" similarities e;isted in Buality system and documentation reBuirements Redundant
reBuirements and multiple plant visits #rom purchasers placed a si"ni#icant burden on suppliers
Con#ormin" to several di##erent, yet very similar, sets o# reBuirements meant unnecessarily
e;pended time, e##ort, and money Reco"ni=in" the overlap in reBuirements, the ma?or
automotive manu#acturersLGeneral @otors, &ord, and ChryslerLas well as truc%
manu#acturers, created a tas% #orce in the early .00/s to develop a Buality system that has as its
#oundation ,SO 0/// Named D:uality System ReBuirements :S 0///,E this comprehensive
reBuirement was intended to develop #undamental Buality systems that provide #or continuous
improvement :S 0/// eliminates redundant reBuirements while maintainin" customer9speci#ic,
division9speci#ic, and commodity9speci#ic reBuirements :S 0/// emphasi=es de#ect prevention
as well as the reduction o# variation and waste ,nternal and e;ternal suppliers o# production and
service parts, subassemblies, materials, components, or other items to the ma?or motor vehicle
manu#acturers must con#orm to the reBuirements set #orth by :S 0///
:S 0/// has two ma?or components) ,SO 0//. and Customer Speci#ic ReBuirements
!utomotive Suppliers must be ,SO 0//. certi#ied Customer speci#ic reBuirements include the
use o# methods #or Statistical Process Control 'SPC(, Production Part !pproval Process 'PP!P(,
&ailure @odes and +##ects !nalysis '&@+!(, @easurement Systems !nalysis '@S!(,
!dvanced Product :uality Plannin" and Control Plannin" '!P:P(, and :uality System
!ssessment ':S!(

2. Why would a company wish to become ,1 or &1# )/// certified?
$he purpose o# the ,SO standards is to #acilitate the multinational e;chan"e o# products and
services by providin" a clear set o# Buality system reBuirements Companies competin" on a
"lobal basis #ind it necessary to adopt and adhere to these standards $he standards provide a
baseline a"ainst which an or"ani=ation-s Buality system can be ?ud"ed $his baseline has as its
#oundation the achievement o# customer satis#action throu"h multidisciplinary participation in
Buality9improvement e##orts, documentation o# systems and procedures, and other basic
structural elements necessary to Buality systems $he "eneric nature o# the standards allows
interested companies to determine the speci#ics o# how the standards apply to its or"ani=ation
@any companies use ,SO 0/// as the #oundation #or their continuous improvement e##orts
$he hi"h cost o# certi#ication is counterbalanced by the bene#its an or"ani=ation will receive by
usin" the reBuirements as a "uide to improve their processes :uality becomes more consistent,
and the percenta"e o# Ddone ri"ht the #irst timeE ?obs increases +nhanced procedures, up"raded
record9%eepin", and the removal o# redundant operations also dramatically improve an
or"ani=ation-s e##ectiveness ,SO 0/// standards #acilitate international trade
3. +chieving &1# )/// certification is not easy. Describe one difficulty a company may
encounter.
25
Obtainin" ,SO 0//. certi#ication is a time9consumin" and costly process *ependin" on the
current state o# an or"ani=ation-s Buality system, preparation #or certi#ication may ta%e several
thousand employee9hours and cost thousands o# dollars Costs depend on the company si=e, the
stren"th o# the or"ani=ation-s e;istin" Buality system, and the number o# plants within the
company reBuestin" certi#ication !s with any ma?or process improvement, the opportunity to
#ail e;ists !ttempts to incorporate ,SO 0/// into the way a company does business may be
hindered by a variety o# #orces, includin" insu##icient mana"ement involvement in the process,
inadeBuate resources, lac% o# an implementation plan, or lac% o# understandin" about ,SO 0///
and its bene#its $his last #orce, a lac% o# understandin" about ,SO 0///, is particularly crucial
,SO 0//. reBuires si"ni#icant documentation $he additional burden o# paperwor%, without an
understandin" o# how this new#ound in#ormation can be used in decision9ma%in" and
or"ani=ational improvement, leads to problems ,t is important to reali=e that standardi=ed
procedures and or"ani=ed in#ormation "o a lon" way toward preventin" errors that lead to poor
Buality products and services Or"ani=ations that are unaware o# how to access the power that
procedures and in#ormation provide may miss out on improvement opportunities ,t is up to
leadership to encoura"e the use o# this in#ormation, thus "ainin" the ma;imum bene#its #rom ,SO
0///
2. Why would a company be interested in creating a *uality system within its
organization?
,n order to best #ul#ill customer needs, reBuirements, and e;pectations, e##ective or"ani=ations
create and utili=e Buality systems Within a *uality management system the necessary
ingredients e+ist to ena$le the organi%ation,s employees to identify, design, develop, produce,
deliver, and support products or services that the customer wants. +##ective Buality
mana"ement systems are dynamic, able to adapt and chan"e to meet the needs, reBuirements, and
e;pectations o# its customers +##ective or"ani=ations use standards such as ,SO 0/// and :S
0///, pro"rams li%e Si; Si"ma, and awards li%e the @alcolm 8aldri"e National :uality !ward
to provide "uidance #or establishin" their Buality mana"ement system-s structure, maintainin"
records, and usin" Buality techniBues to improve processes and systems
8. Describe the 1i" 1igma philosophy.
+ssentially, Si; Si"ma is about resultsA enhancin" pro#itability throu"h improved Buality and
e##iciency ,mprovement pro?ects are chosen based on their ability to contribute to the bottom
line on a company-s income statement by bein" connected to the strate"ic ob?ectives and "oals o#
the corporation Pro?ects that do not directly tie to customer issues or #inancial results are o#ten
di##icult to sell to mana"ement <hen choosin" a Si; Si"ma pro?ect or any improvement
pro?ect, care should be ta%en to avoid poorly de#ined ob?ectives or metrics Key business metrics
include revenue dollars, labor rates, #i;ed and variable unit costs, "ross mar"in rates, operatin"
mar"in rates, inventory costs, "eneral and administrative e;penses, cash #low, warranty costs,
product liability costs, and cost avoidance Si; Si"ma pro?ects are easy to identi#y, since the Si;
Si"ma methodolo"y see%s to reduce the variability present in processes, pro?ect teams see% out
sources o# waste, such as overtime and warranty claims, investi"ate production bac%lo"s or areas
24
in need o# more capacity, and #ocus on customer and environmental issues <ith hi"h volume
products even small improvements can produce si"ni#icant impact on the #inancial statement
6. Discuss two reasons why an organization would want to strive toward 1i" 1igma
performance.
Si; Si"ma pro"rams lead to improved pro#itabilityA increased customer retentionA reduced
customer complaints and warranty claimsA reduced costs throu"h less waste, rewor%, and so onA
achieve a "reater mar%et shareA increased employee involvement and satis#actionA lower
employee turnoverA an increased ability to attract new customers, improve their competitiveness,
and customer satis#actionA improved mana"ement9employee relationsA improved #ocus on %ey
"oalsA improved internal and e;ternal communicationA and improved teamwor%
=. Describe the :alcolm 6aldrige <ational ,uality +ward.
$he @alcolm 8aldri"e National :uality !ward '@8N:!( was established in .012 by the
United States Con"ress Similar to Japan-s *emin" Pri=e, it sets a national standard #or Buality
e;cellence $he award is open to companies in three areas) 8usiness, +ducation, and Health
Care ,t is mana"ed by the !merican Society #or :uality 'wwwasBor"( +very year, this
ri"orous award attracts several do=en applicants in each cate"ory ! "roup o# Buali#ied
e;aminers compares and contrasts each application with the criteria #or up to 6// hours Only a
very select #ew reach the site9visit sta"e o# the award process 8y the completion o# the on9site
visits, a company may have been e;amined #or as many as ./// hours Since .011, 55
or"ani=ations have received the United States-s hi"hest award #or or"ani=ational e##ectiveness
$he award sets standards to be used as baselines and benchmar%s #or total Buality mana"ement in
seven areas $he seven cate"ories are leadership, strate"ic plannin", customer and mar%et #ocus,
measurement, analysis and %nowled"e mana"ement, wor%#orce #ocus, process mana"ement, and
results $he #ollowin" descriptions are paraphrased #rom the @alcolm 8aldri"e National :uality
!ward criteria

2.0 6eadership $he criteria in Section ./ are used to e;amine senior9level mana"ement-s
commitment to and involvement in process improvement Company leaders are e;pected to
develop and sustain a customer #ocus supported by visible actions and values on their part $his
section also e;amines how the or"ani=ation addresses its responsibilities to the public and
e;hibits "ood citi=enship Subcate"ories include senior leadership direction, or"ani=ational
per#ormance review, responsibilities to the public, and support o# %ey communities
2.0 Strategic "lanning $o score well in this cate"ory, a company needs to have sound strate"ic
ob?ectives and action plans $he e;aminers also investi"ate how the company-s strate"ic
ob?ectives and action plans are deployed and pro"ress measured $hey study the or"ani=ation-s
strate"y development process, strate"ic ob?ectives, and action plan development and deployment
e##orts
5.0 Customer and Mar7et 8ocus $he third cate"ory o# the 8aldri"e !ward criteria deals with
the company-s relationship with their customers $his cate"ory #ocuses on a company-s
23
%nowled"e o# customer reBuirements, e;pectations, and pre#erences, as well as mar%etplace
competitiveness Reviewers also determine i# the company has put this %nowled"e to wor% in
the improvement o# their products, processes, systems, and services Success#ul e##orts in this
cate"ory lead to improved customer acBuisition, satis#action, and retention $his cate"ory
clari#ies a company-s commitment to their customers, an or"ani=ation-s customer and mar%et
%nowled"e, customer relationships, and customer satis#action determination
+.0 Measurement# 1nalysis# and 9no(ledge Management $he award reco"ni=es that
in#ormation is only use#ul when it is put to wor% to identi#y areas #or improvement $his
cate"ory investi"ates a company-s use o# in#ormation and per#ormance measurement systems to
encoura"e e;cellence Per#ormance in#ormation must be used to improve operational
competitiveness Per#ormance measurement, per#ormance analysis, data availability, hardware
and so#tware Buality, competitive comparisons, and benchmar%in" are encoura"ed
,.0 :or74orce 8ocus <ithin the wor%#orce #ocus section, reviewers #or the 8aldri"e !ward are
interested in a company-s plans and actions that enable their wor%#orce to per#orm to its #ullest
potential in ali"nment with the company-s overall strate"ic ob?ectives +mployee involvement,
education, trainin", and reco"nition are considered in this cate"ory ! company-s wor%
environment receives care#ul scrutiny in an e##ort to determine how the company has built and
maintains a wor% environment conducive to per#ormance e;cellence, as well as personal and
or"ani=ational "rowth !lso investi"ated are wor% systems, employee education, trainin" and
development, wor% environment, and employee support and satis#action
;.0 "rocess Management <ithin this cate"ory, the company is ?ud"ed on its process
mana"ement abilities Companies must provide details on their %ey business processes related to
customers, products and service delivery, desi"n, productionFdelivery, business mana"ement, and
support processes
*.0 %esults Ultimately, the purpose o# bein" in business is to stay in business $his cate"ory
e;amines a company-s per#ormance and improvement in several %ey business areas, includin"
customer satis#action, product and service per#ormance, #inancial and mar%etplace per#ormance,
wor%#orce, and operational per#ormance 8enchmar%in" is encoura"ed to see how the company
compares with its competitors ,nvesti"ators study customer results, product and service results,
#inancial and mar%et results, wor%#orce results, operational results, and public responsibility and
citi=enship results
$he award criteria are updated yearly Recipients o# the award are #rom a variety o# industries,
includin" telecommunications, ban%in", automotive, hospitality industry, education, hospitals,
buildin" products, and manu#acturin"
3. Discuss two reasons why an organization would be interested in benchmar'ing
against the :alcolm 6aldrige <ational ,uality +ward.
$he award sets standards to be used as baselines and benchmar%s #or total Buality mana"ement in
seven areas $he seven cate"ories are leadership, strate"ic plannin", customer and mar%et #ocus,
measurement, analysis and %nowled"e mana"ement, wor%#orce #ocus, process mana"ement, and
22
results Knowin" their per#ormance in these seven areas can help an or"ani=ation understand
where their stren"ths and wea%nesses lie <ith this in#ormation, they can improve their
competitiveness in the mar%et
Companies #ollowin" the "uidelines set out in the @8N:!C can improve their overall business
per#ormance ,t is interestin" to note that the 8aldri"e award winners, when their stoc%
per#ormance is reviewed, out9per#orm the stoc%s in the Standard and Poor-s 4// ,nde; by a ratio
o# about 6 to . &or more in#ormation about the 8aldri"e stoc% study, visit the #ollowin"
website) wwwnist"ovFpublic_a##airsF#actsheetFstoc%studyhtm
21
Chapter ' Creating a Customer Focus
1. Why is a focus on customers so important to the success of an organization?
$he current "lobal business environment is e;tremely competitive $oday-s consumers are more
than willin" to switch #rom supplier to supplier in search o# better service or availability or
courtesy or #eatures or #or any variety o# reasons $o attract and retain customers, e##ective
or"ani=ations need to #ocus on determinin" and providin" what their customers want and value
!dvertisin", mar%et positionin", productFservice ima"in", discountin", crisis handlin", and other
methods o# attractin" the customer-s attention are not enou"h +##ective or"ani=ations survive
because they tal% to customers, translate what their customers said into appropriate actions, and
ali"ned their %ey business processes to support what their customers want $hese critical
activities enable them to meet their customers- needs, wants, and e;pectations the #irst time,
every time
2. Why would an organization want to create an atmosphere focused on customers?
What would be the benefit of such an approach?
,n many ways, a company-s #uture survival depends on their ability to meet their customers-
needs, wants, and e;pectations, not only now, but in the #uture Companies that are truly #ocused
on the customer are better able to react to mar%et and "lobal competitive challen"es $his a"ility
enables them to attract and retain customers, ensurin" their success now and in the #uture
3. 0ow would you recognize an organization with a customer focus? What clues would
you need?
*o the company leaders and employees %now the answer to the Buestion) what are the three to
ten thin"s that we absolutely must do well in order to attract and retain customers>
2. ite an e"ample of an effective organization you are aware of. 0ow do you 'now it is
more effective than its competitors?
8. :r. :arshall -ield was the premiere department store ownerAmanager during the late
13//s. .oday! due in part to the fact that his stores still follow his management beliefs!
:arshall -ield%s stores remain very successful. .he following *uote comes from
:arshall -ield%s boo' Give the Lady What She Wants! written in 1)//! based on his
e"periences.
:r. -ield! while strolling through his store! came upon his assistant retail manager
arguing heatedly with a woman customer.
EWhat are you doing here?F as'ed -ield.
E& am settling a complaint!F replied the manager.
E<o! you%re not!F snapped -ield. E7ive the lady what she wantsBF
4ater he sent the following memo to all employees>
20
EWe wish!F he advised all employees! old and new! Eto ma'e important the point
of receiving merchandise that has been returned! without going through the
unnecessary or unpleasant *uestions or remar's! which ma'e the customer ill at ease
and really accomplish no good. We would much prefer to be occasionally imposed
upon than to feel that we have made any errors in the other direction. .he employee is
not the loser by the returning of goods! e"cept in a very slight way. .he house is the
loser! and if we prefer to stand the loss cheerfully! please let us have corresponding
cheerfulness from employees.F Give the Lady What She Wants Wendt and Hogan! +nd 6oo's
$ublishers.
0ow does this e"perience relate to what you learned about the ustomer and :ar'et
-ocus :6<,+ riteria?
,n this situation, the customer is ?ud"in" their e;perience not only with the product she has
purchased, but also the service she is receivin" 8ased on her e;perience, she %nows whether or
not she has received a Buality product or service She %nows what Buality represents to her !s
!rmand &ei"enbaum-s de#inition states)
*uality is a customer determination which is $ased on the customer,s actual e+perience
with the product or service, measured against his or her re*uirements-stated or
unstated, conscious or merely sensed, technically operational or entirely su$)ective-
always representing a moving target in a competitive mar"et.
$his determination is dependent on her actual e;perience with the product or service
Consumers ta%e their past e;periences, reBuirements, e;pectations, and needs and combine the
in#ormation into their ?ud"ment o# a product or service-s value
Providers o# products and services, li%e @arshall &ield-s, must reco"ni=e that customer-s base
their decisions about the Buality o# a product or service on the perceived value they receive
Customer value ?ud"ments, because they involve past e;perience, reBuirements, wants, needs,
and e;pectations, are comple; $he store itsel# is bein" evaluated based on its credibility and
reputation #or responsiveness to customers, employee competence, ability to communicate, and
courtesy $his customer-s perception o# value is what will ultimately ma%e a di##erence between
an unsuccess#ul customer and a success#ul one
$his customer-s perception o# the value she has received in the recent transaction will a##ect her
#uture decision to purchase the same thin" a"ain or to return to the store ,# she perceives her
overall e;perience with the product or service as valuable, she will most li%ely purchase in the
#uture, i# she does not, she won-t +##ective or"ani=ations reali=e that how the customer
perceives the value o# that transaction will determine whether or not they will buy #rom the same
or"ani=ation the ne;t time ,t is important to D"ive the lady what she wantsE so that she will
perceive value in her e;perience, leadin" to #uture transactions Customer perceived value is
what enables a company to be success#ul in the #uture $o ensure the "rowth o# their business,
or"ani=ations need loyal customers Coyalty, o#ten described as customer retention, is really the
absence o# a better alternative Hi"h customer satis#action ratin"s in the past do not necessarily
eBuate to customer loyalty in the #utureA hi"h perceived value ratin"s do +##ective or"ani=ations
continually see% to increase their customers- perceived value o# their products and services
1/
6. 1elect one of Deming%s fourteen points and discuss how it relates to creating a
customer focus.
Could discuss)
Create a constancy o# purpose 'which #ocuses on the customer(
!dopt a new philosophy 'o# customer #ocus(
Constantly and #orever improve 'the or"ani=ation-s #ocus on the customer, the or"ani=ation-s
processes directly a##ectin" the customer(
,nstitute trainin" on the ?ob '#or employees who-s wor% directly a##ects the customer, #ocused on
the customer(
,nstitute leadership '#ocusin" on the customer(
8rea% down barriers 'ma%in" wor%in" with the company appear seamless to the customer(
=. 0ow do the following paragraphs about :. -ield and his store apply to 1ection 3./!
ustomer and :ar'et -ocus of the :6<,+? 6e sure your answer ties directly to the
criteria.
-ield%s zeal for 'eeping customers and employees happy spurred him always as he
strengthened his position in the store. 0e constantly planned new departments and
new advertising techni*ues. 0e devised a simpler method of ma'ing out sales slips.
0e rearranged displays. 0e set up wor'rooms where a woman shopper might leave her
glasses to be ground! her gloves to be cleaned! her @ewelry or shoes to be shined.
-or years an employee sought permission to open a restaurant in the store. .he idea
was shunned. E.his is a dry;goods store! not a restaurant.F &n +pril! 13)/! he was
successful in setting up a tearoom on the third floor. #n the first day only fifty;si"
patrons were served. .he tearoom had sudden appeal for women shoppers. &n a year
it was enlarged and serving as many as 1!8// each day. (ventually! this tearoom
became one of several restaurants and dining rooms on the store%s seventh floor.
Give the Lady What She Wants Wendt and Hogan! +nd 6oo's $ublishers.
+##ective or"ani=ations need an accurate understandin" o# what their customers e;pect &ield
#ocused on customers 'both internal and e;ternal( and tried to determine their needs, wants, and
e;pectations He also tried to identi#y the "ap between the store-s current per#ormance and what
the customer reBuires i# the store is "oin" to properly tar"et improvement activities He
reco"ni=ed the importance o# studyin" both customer value perceptions and customer
satis#action He as%ed the customers, both internal and e;ternal, what they needed !s an
e##ective or"ani=ation leader, he tal%ed with his customers He sou"ht to understand every
aspect o# the customers- interaction with their company He understood that this process be"ins
when the customer #irst contacts the company and continues until the product has been consumed
or the service completed He reali=ed what business they are really in because they have as%ed
that Buestion #rom the point9o#9view o# the customer $heir customers have helped them de#ine
their business, their principle stren"ths and wea%nesses, as well as what they need to do in the
#uture in order to improve
1.
3. 0ow do these paragraphs about :. -ield and his store apply to 1ection 3./!
ustomer and :ar'et -ocus of the :6<,+? 6e sure you answer ties directly to the
criteria.
E&f the customer says she hasn%t received satisfaction!F he advised! Ewe are not going to
challenge the customer. Why should we say in effect to the customer! I?ou lie%? 4et us
probe our own organization and ascertain why the customer had not received the right
merchandise that we are )) percent certain was in our immense stoc'.F
EWhen we say Ethe ustomer is always right!% that means far more than the simple duty
of not remonstrating or *uestioning her actions. &t means that the customer is right in
her desire to be given an honest value and to be treated with every possible
consideration. &t means that every piece of merchandise must be precisely and e"actly
what it is represented to be. &t means the customer must 'now from our advertising
e"actly what it is we have to sell. &t means the elimination of empty and meaningless
catch phrases in every step of the sales effort. What we write and what we say about
our merchandise or our services must be strictly! scrupulously! unfailingly the truth.F
Give the Lady What She Wants Wendt and Hogan! +nd 6oo's $ublishers.
$he Customer and @ar%et &ocus criterion e;amines how an or"ani=ation see%s to understand the
voices o# the customers and the mar%etplace ,t also e;amines how the or"ani=ation builds
relationships with customers and determines %ey #actors that lead to customer acBuisition,
satis#action, and retention Section 6., Customer and @ar%et Knowled"e e;amines how the
or"ani=ation determines current and emer"in" customer reBuirements and e;pectations in order
to remain competitive in the mar%etplace Section 67 Customer Relationships and Satis#action
e;amines the or"ani=ation-s processes #or buildin" customer relationships and determinin"
customer satis#action, with the aim o# acBuirin" new customers, retainin" e;istin" customers,
and developin" new opportunities $hese para"raphs show leadership-s commitment to the
customer and creation o# customer value ,t shows how the store leadership encoura"es the
employees to build relationships with the customers and also spea%s to the %ey #actors that lead
to customer acBuisition, satis#action, and retention
17
Chapter ( Organi#ational )ea*ership
1. What did Deming mean when he said that it is important to manage with a 'nowledge
of variation?
+##ective leaders %now that people must be included in the or"ani=ation-s decision9ma%in"
processes $hey also %now that sharin" in#ormation is critical to ma%in" "ood decisions
@ana"ement by #act involves understandin" the how-s and why-s o# a situation be#ore ta%in"
action @ana"ement by #act reBuires an appreciation #or and understandin" o# the %ey systems
o# an or"ani=ation +##ective leaders reali=e that mana"ement o# systems reBuires %nowled"e o#
the interrelationships between all the components within the system and the people that wor% in
it ,n#ormation can sometimes be misleadin" <hen leaders mana"e by #act, they use ob?ective
evidence to support their decisions Ob?ective evidence is not biased and is e;pressed as simply
and clearly as possible @ore importantly, it is traceable bac% to its ori"in, whether that be a
customer, an order number, a product code, machine, or an employee +##ective leaders are the
ones who remember to as% *r *emin"-s #avorite Buestion) #ow do we "now Knowin" the
answer to this Buestion veri#ies where the in#ormation has come #rom, its importance and
relationship with the issue at hand Havin" this %nowled"e means havin" the #acts available to
support a plan o# action
Chapter 7 discussed the two sources o# variation in a process as identi#ied by *r Shewhart
Controlled variation, the variation present in a process due to the very nature o# the process, can
be removed #rom the process only by chan"in" the process Uncontrolled variation, on the other
hand, comes #rom sources outside o# the process ,t is not normally part o# the process and can
be identi#ied and isolated as the cause o# a chan"e in the behavior o# the process $o mana"e
with an understandin" o# variation means that a leader would reco"ni=e the type o# variation
present and respond accordin"ly !s *r *emin" clari#ied, it would be a mista%e to react to any
#ault, complaint, mista%e, brea%down, accident, shorta"e as i# it came #rom a special cause when
in #act there was nothin" special at all, ie, it came #rom random variation due to common causes
in the system ,t is also a mista%e to attribute to common causes any #ault, complaint, mista%e,
brea%down, accident, shorta"e when it actually came #rom a special cause !n understandin" o#
variation enables leaders to ma%e the ri"ht choices and deal with problems as they arise
appropriately
2. :r. :arshall -ield was the premiere department store ownerAmanager during the late
13//s. .oday! due in part to the fact that his stores still follow his management beliefs!
:arshall -ield%s stores remain very successful. .he following *uote comes from
:arshall -ield%s boo' Give the Lady What She Wants! written in 1)//! based on his
e"periences.
:r. -ield! while strolling through his store! came upon his assistant retail manager
arguing heatedly with a woman customer.
EWhat are you doing here?F as'ed -ield.
E& am settling a complaint!F replied the manager.
E<o! you%re not!F snapped -ield. E7ive the lady what she wantsBF
16
4ater he sent the following memo to all employees>
EWe wish!F he advised all employees! old and new! Eto ma'e important the point
of receiving merchandise that has been returned! without going through the
unnecessary or unpleasant *uestions or remar's! which ma'e the customer ill at ease
and really accomplish no good. We would much prefer to be occasionally imposed
upon than to feel that we have made any errors in the other direction. .he employee is
not the loser by the returning of goods! e"cept in a very slight way. .he house is the
loser! and if we prefer to stand the loss cheerfully! please let us have corresponding
cheerfulness from employees.F
Give the Lady What She Wants Wendt and Hogan! +nd 6oo's $ublishers.
0ow does this e"perience relate to what you have learned about the 4eadership
:6<,+ riteria?
$he @alcolm 8aldri"e National :uality !ward criteria encoura"es or"ani=ations to e;amine
how their leadership "uides the or"ani=ation Senior leaders o# the or"ani=ation are as%ed how
they direct their or"ani=ation, as well as how they review their or"ani=ation-s per#ormance
Ceaders must describe how they set and deploy or"ani=ational values, steer the or"ani=ation
toward its short and lon" term "oals, how they create value #or their customers, and how they set
and monitor per#ormance e;pectations Senior leaders are also e;pected to describe how they
create an environment supportin" or"ani=ational a"ility, employee empowerment, and employee
learnin" Ceaders are reviewed in terms o# their ability to translate priorities, strate"ies, and
ob?ectives into or"ani=ational improvement and opportunities #or innovation $hey are as%ed to
show how they create ali"nment between the customers- value propositions and the
or"ani=ation-s strate"ies, "oals, ob?ectives, and day9to9day activities $he criteria also addresses
the or"ani=ation-s responsibilities to the public by #ocusin" on how the leaders ensure that the
or"ani=ation understands and deals with the impact on society o# their products, services, and
operations ,t as%s leaders how they address their responsibilities to the public by practicin"
"ood citi=enship +mployee involvement in the or"ani=ation-s %ey communities is also
encoura"ed, as are ethical business practices
Ceadership has as its tas% creatin" and maintainin" an e##ective or"ani=ation with an unwaverin"
#ocus on the customer $heir activities must not be con#ined to a sin"le aspect o# the
or"ani=ation
3. -rom a leadership point of view! what did .homas 6erry! author of Managing the
Total Quality Transformation! mean when he said> E.oo often management attempts to
implement strategy by describing attitudes! however! management needs to ta'e steps
to influence the actions employees ta'e.F
+##ective leaders provide "uidance, o#ten in the physical Dwal% the tal%E type o# #ormat $hese
leaders ma%e sure that their actions correspond with a written vision or mission statement
+##ective leaders communicate the values o# the or"ani=ation to their employees by translatin"
the vision and mission into day9to9day activities $o do this e##ectively, leaders tal% with
customers, identi#y the or"ani=ation-s critical success #actors, and share this in#ormation about
the thin"s the or"ani=ation absolutely must do well in order to attract and retain customers
15
Creatin" ali"nment is essentially policy deployment, the step9by9step process o# translatin" the
or"ani=ation-s vision and mission into strate"ies supported by "oals and ob?ectives that in turn
become wor% activities #or the employees Ceaders ensure that the or"ani=ation-s vision, mission
and strate"ies, "oals and daily activities remain #ocused on these critical #actors
,n order to create ali"nment, e##ective leaders practice *r *emin"-s point) 'mplement
leadership Knowin" ali"nment is not created by wish#ul9thin%in", e##ective leaders are the #irst
to as%)
*oes the employee %now what heFshe is supposed to do>
*oes the employee have the means to determine whether or not heFshe is doin" the ?ob
correctly>
*oes the employee have the authority and the means to correct the process when
somethin" is wron">
+##ective leaders are also the #irst to reali=e that i# any o# these Buestions "o unanswered,
chances are, #ault lies with leadership, not with the employees $o help create ali"nment,
e##ective leaders care#ully desi"n employee policies supportin" the lin%s between the critical
success #actors identi#ied by their customers and the employees- day9to9day activities +##ective
leaders set realistic "oals #or their employees and "ive timely rewards to those who meet these
"oals Ceaders see% out employees- ideas and actively support the "ood ones Jery #ew people
are truly "ood listeners, e##ective leaders have learned to listen to their employees ,t helps to
%eep in mind that the same letters spell the words DlistenE and DsilentE Ceaders also place a hi"h
priority on e;pandin" employee capabilities and responsibilities throu"h education and trainin"
2. 1elect one of Deming%s fourteen points and discuss how it relates to leadership.
Could discuss)
Create a constancy o# purpose 'only leadership can create and en#orce a #ocus on the customer(
!dopt a new philosophy 'leadership sets the direction o# the or"ani=ation(
Constantly and #orever improve 'leadership is responsible #or "uidin" the or"ani=ation-s #ocus on
the customer and the or"ani=ation-s processes directly a##ectin" the customer(
,nstitute leadership 'mana"in" with a %nowled"e o# variationA creatin" a #ocus on customer(
Remove barriers 'ma%in" wor%in" with the company appear seamless to the customer(
+liminate slo"ans, etc 'institute leadership(
+liminate arbitrary wor% standards, etc 'institute leadership(
8. E&t ta'es personal involvement to understand and remove the barriers that stand
between current capabilities and improvement.F W. (dwards Deming
0ow does this statement relate to leadership?
< +dwards *emin" states that, ./he aim of leadership should $e to improve the performance
of man and machine, to improve *uality, to increase output and simultaneously to $ring pride of
wor"manship to people. (ut in a negative way, the aim of leadership is not merely to find and
record failures of men, $ut to remove the causes of failure0 to help people to do a $etter )o$ with
14
less effort.1 Ceaders "et people to do what they have not done be#ore thus leadin" to the
or"ani=ation bein" more success#ul Ceaders provide direction and purpose #or an or"ani=ation
,t is leaders that provide the ultimate encoura"ement and reasonin" behind why people would
want to do their ?obs People per#orm on the ?ob #or a variety o# reasons $hey may per#orm #or
the #inancial incentives involved, the presti"e, or the possibility o# #uture rewards Sometimes
people even per#orm out o# #ear, #ear o# ?ob loss, #ear o# retribution, #ear o# unpleasant
conseBuences $he most e##ective reason why people wor% is because they want to !ccordin"
to *r *emin", leadership-s overall aim should be to create a system in which everyone ta%es ?oy
in their wor%
6. 0e loved to go through the store a dozen times a day. 0e swept through the aisles!
his eyes ta'ing in every detail about him. 0e too' a deep interest in everything that
went on in the storeJfrom the welfare of the cash boys! to the e"act wording of the
linen department%s advertisement for the wee'. 0e stayed in his office until long after
the store closed each night! yet was one of the first to arrive in the morning. 0e never
criticized wor'ers in the presence of customers or other cler's. #n his trips through the
store he always stopped to tal' with customers. +ny customer! whether her mission
involved a small purchase or one that ran into the thousands of dollars! was assured a
personal interview if she desired it. When a shipping cler' alerted him that -ield%s was
no longer shipping merchandise to some of the customers they used to! he set up a
tas' force that determined many of the reasons why these customers had stopped
shopping at -ield%s. .hrough their efforts! changes were made that returned customers
to the store. .here was but one test for the success or failure of changes implemented>
they had to bring profits.
Give the Lady What She Wants Wendt and Hogan! +nd 6oo's $ublishers.
Deming said! E?our @ob as a leader is to manage the change necessary.F What does
that mean?
Chan"e is rarely easy Ceaders need to understand that resistance to chan"e is a natural human
reaction +##ective leaders %now that in order #or chan"e to occur, resistance to those chan"es
must be e;pected and planned #or +ssentially, they mana"e chan"e by mana"in" resistance
$hey may ta%e a preventive approach, meetin" resistance head on be#ore it starts &or instance,
they may reali=e that resistance to a chan"e may occur because additional %nowled"e or s%ills
will be reBuired !n e##ective leader, mana"in" resistance, will provide the appropriate trainin"
and education Or they may ta%e a wait and see approach, plannin" to deal with resistance as it
arises ,n either case, leaders must clearly communicate the new e;pectations and the reasons
behind the chan"es so that people understand why they are bein" as%ed to chan"e $heir
employees are loo%in" to them and as%in" the Buestion) D<hat do you want me to do tomorrow
that is di##erent #rom what , am doin" today>E +##ective leaders provide trainin" and time to
ma%e the chan"e $hey also structure their reward system to support the chan"e !s the &i"ure
in the te;t describes, i# an individual can see the reason #or the chan"e and has time to ma%e the
chan"e, they will usually ma%e the e##ort to ma%e the chan"e !s a person becomes more
com#ortable with the new way o# doin" thin"s, the old way slowly dies out $his chan"e process
must be supported by pressure #rom the reward system, time to ad?ust to the chan"e, and any
necessary trainin" to #acilitate the chan"e
13
=. E.o do the right thing! at the right time! in the right wayG to do some things better than
they were ever done beforeG to eliminate errorsG to 'now both sides of the *uestionG to
be courteousG to be an e"ampleG to wor' for the love of the wor'G to anticipate
re*uirementsG to develop resourcesG to recognize no impedimentsG to master
circumstancesG to act from reason rather than ruleG to be satisfied with nothing short of
perfection.F
Give the Lady What She Wants Wendt and Hogan! +nd 6oo's $ublishers.
What needs to happen from a leadership point of view in order to translate the mission?
+##ective leaders communicate the values o# the or"ani=ation to their employees by translatin"
the vision and mission into day9to9day activities $o do this e##ectively, leaders tal% with
customers, identi#y the or"ani=ation-s critical success #actors, and share this in#ormation about
the thin"s the or"ani=ation absolutely must do well in order to attract and retain customers
Creatin" ali"nment is essentially policy deployment, the step9by9step process o# translatin" the
or"ani=ation-s vision and mission into strate"ies supported by "oals and ob?ectives that in turn
become wor% activities #or the employees Ceaders ensure that the or"ani=ation-s vision, mission
and strate"ies, "oals and daily activities remain #ocused on these critical #actors
12
Chapter + Strategic Planning
1. -rom a strategic planning point of view! what did .homas 6erry! author of Managing
the Total Quality Transformation! mean when he said> E.oo often management attempts
to implement strategy by describing attitudes! however! management needs to ta'e
steps to influence the actions employees ta'e.F
People-s attitudes and belie#s must be turned into actions in order to move a company #orward
$o be e##ective, a strate"ic plan must be deployed +##ective leaders enable members o# the
or"ani=ation to ma%e the transition between the strate"ic plan and daily business activities by
translatin" <H!$ needs to be accomplished into HO< it will be accomplished +##ective
leaders ma%e sure that the day9to9day activities and the "oals o# the strate"ic plans o# the
or"ani=ation are in harmony and #ocused on what is critical to the success o# the or"ani=ation
$hey want to ensure that i# they push on the strate"ic plan, the actions o# their employees will "o
in the desired direction &or this reason, the strate"y must be clearly communicated throu"hout
the or"ani=ation +##ective leaders ma%e sure that the strate"ic plan contains clear ob?ectives,
provides and utili=es measures o# per#ormance, assi"ns responsibilities to speci#ic individuals,
and denotes timin" ,n order #or a strate"ic plan to be e##ectively deployed, the or"ani=ation-s
reward and reco"nition system must support plan deployment
2. + school system has decided to adopt a ,uality :anagementA#rganizational
(ffectiveness approach to doing business. Describe three 'ey things they need to do to
ma'e a success of this venture. K&n other words! where do they go from here?L 6e sure
to relate your answer to the 1trategic $lanning section of the :alcolm 6aldrige <ational
,uality +ward riteria.
$he @alcolm 8aldri"e National :uality !ward criteria section 7) Strate"ic Plannin", e;amines
how the or"ani=ation sets strate"ic directions and how it creates %ey action plans to support the
strate"y !pplicants are as%ed to describe their or"ani=ation-s strate"y development process
$his process must address a variety o# %ey #actors, includin" customer and mar%et needs,
e;pectations and opportunities, the competitive environment and the or"ani=ation-s reaction to it,
the or"ani=ation-s stren"ths and wea%nesses, includin" technolo"y or human resources
Reco"ni=in" that strate"y development and strate"y deployment "o hand in hand, this section
as%s how the strate"y developed by or"ani=ational leaders is deployed in order to be e##ective
Or"ani=ational strate"y should address supplier or partner stren"ths and wea%nesses
Or"ani=ations are also a##ected by e;istin" economic, societal, and #inancial conditions, so these
must be addressed Strate"ic plans must include ob?ectives and the time #rame #or accomplishin"
those ob?ectives $he award is interested in how an or"ani=ation translates its strate"ic plan into
actions, both short and lon" term +mphasis is placed on the use o# per#ormance measures to
trac% the or"ani=ation-s pro"ress toward reachin" the strate"ic "oals and ob?ectives outlined in
the strate"ic plan $o create and deploy a strate"ic plan, the or"ani=ation must #ind the answers
to the #ollowin" three Buestions)
<hat business are we really in>
<hat are the or"ani=ation-s principle stren"ths and wea%nesses #or competin" in this
mar%et compare to what it ta%es to compete success#ully>
11
<hat does the or"ani=ation wish to become in the #uture>
3. What did Deming mean when he spo'e of the Ehope for instant puddingF?
*emin" is tal%in" about how people wish somethin" were true, but don-t ta%e the time and e##ort
necessary to ma%e it become true
2. 1elect one of Deming%s fourteen points and discuss how it relates to strategic
planning.
Could discuss)
Create a constancy o# purpose 'a strate"ic plan describes and deploys the constancy o# purpose(
!dopt a new philosophy 'the strate"ic plan sets the direction o# the or"ani=ation(
Constantly and #orever improve 'how to do this is detailed in the strate"ic plan(
,nstitute leadership 'leadership creates and deploys strate"ic plans(
+nd the practice, etc 'a strate"ic plan can detail how to do this(
+liminate slo"ans, etc 'a strate"ic plan can detail how to do this(
+liminate arbitrary wor% standards, etc 'a strate"ic plan can detail how to do this(
Remove barriers, etc 'a strate"ic plan can detail how to do this(
Put everybody in the company to wor% 'a strate"ic plan can detail how to do this(
8. E&t ta'es personal involvement to understand and remove the barriers that stand
between current capabilities and improvement.F W. (dwards Deming
0ow does this statement relate to strategic planning?
<ith this statement, *r *emin" points out to leaders that their or"ani=ation must be "uided
toward its ultimate "oals Strate"ic plans allow leadership to put down in writin" the direction
the or"ani=ation is headin" and how it plans to "et there $o be e##ective, a strate"ic plan must
be deployed !s livin" documents, they are not meant to sit on a shel#, only to be touched when
it is time #or an annual revision +ssentially, creatin" ali"nment is policy deployment
Alignment means that if you push on one end, the other end will move in the direction you want.
+##ective leaders enable members o# the or"ani=ation to ma%e the transition between the strate"ic
plan and daily business activities by translatin" <H!$ needs to be accomplished into HO< it
will be accomplished +##ective leaders ma%e sure that the day9to9day activities and the "oals o#
the strate"ic plans o# the or"ani=ation are in harmony and #ocused on what is critical to the
success o# the or"ani=ation $hey want to ensure that i# they push on the strate"ic plan, the
actions o# their employees will "o in the desired direction &or this reason, the strate"y must be
clearly communicated throu"hout the or"ani=ation +##ective leaders ma%e sure that the strate"ic
plan contains clear ob?ectives, provides and utili=es measures o# per#ormance, assi"ns
responsibilities to speci#ic individuals, and denotes timin" ,n order #or a strate"ic plan to be
e##ectively deployed, the or"ani=ation-s reward and reco"nition system must be support plan
deployment
6. Why is a well;thought;out strategic plan critical to the success of a company?
10
Strategic planning is a process of involving everyone in matching the vision, mission, and core
values of an organi%ation with the current situation to focus tactical activities now and in the
future. Strate"ic plans set the direction and pace #or the entire or"ani=ation $he purpose o# a
strate"ic plan is to develop and achieve the or"ani=ation-s mission in a manner consistent with its
vision and values $he strate"ic plan spells out the speci#ic "oals and ob?ectives that must be
accomplished in order to reach the vision ! strate"ic plan see%s to ali"n how the customers-
needs are met throu"h daily business activities with the values, mission, vision, and "oals o# the
or"ani=ation Per#ormance measures, inte"rated into the strate"ic plan, allow leaders to ?ud"e
how the or"ani=ation is pro"ressin" toward its "oals and ob?ectives and there#ore its vision
Strate"ic Buality plannin" ta%es a broader view o# the plannin" process than traditional strate"ic
plannin"
Success#ul lon"9term strate"ic plannin" answers some basic Buestions)
9What $usiness is the organi%ation really in
2What are the organi%ation,s principal strengths and wea"nesses for competing in this
mar"et
2What does it ta"es to compete successfully
2What does the organi%ation wish to $ecome in the future
=. :any companies have missions and visions and strategic plans. Describe one
possible reason why they might not be as successful as they could be if good strategic
planning in not practiced.
$hese or"ani=ations lac% the #ocus and direction when strate"ic plans are not deployed !
strate"ic plan, once deployed, allows leadership to measure their company-s per#ormance, it-s
ability to close the "ap between what is needed and what they are currently doin" ,n a
competitive business environment, an e##ective or"ani=ation utili=es care#ully desi"ned strate"ic
plans in order to create and sustain its competitive advanta"es and pro#it position ! well9
structured strate"ic plan outlines the rational #or bestin" the competition in the mar%et by
e;ploitin" mar%et opportunities, ma;imi=in" or"ani=ational stren"ths, and playin" o## o# the
competitions- wea%nesses +##orts to address these issues typically result in a product or service
that provides the customer with "reater value, either throu"h improved Buality, #avorable
economics, or enhanced service or per#ormance Strate"ic plans are the battle plans that enable
an or"ani=ation to accomplish their ob?ectives 8y implementin" these plans, or"ani=ations are
able to better place their products or services in the mar%et Strate"ic plans establish a direction
#or the or"ani=ation, the results o# implementin" these plans is dependent upon the plans
themselves, the individuals implementin" them, and the #orces at wor% in the mar%et
Remember, other or"ani=ations are implementin" their strate"ic plans, too
3. 0e loved to go through the store a dozen times a day. 0e swept through the aisles!
his eyes ta'ing in every detail about him. 0e too' a deep interest in everything that
went on in the storeJfrom the welfare of the cash boys! to the e"act wording of the
linen department%s advertisement for the wee'. 0e stayed in his office until long after
the store closed each night! yet was one of the first to arrive in the morning. 0e never
0/
criticized wor'ers in the presence of customers or other cler's. #n his trips through the
store he always stopped to tal' with customers. +ny customer! whether her mission
involved a small purchase or one that ran into the thousands of dollars! was assured a
personal interview if she desired it. When a shipping cler' alerted him that -ield%s was
no longer shipping merchandise to some of the customers they used to! he set up a
tas' force that determined many of the reasons why these customers had stopped
shopping at -ield%s. .hrough their efforts! changes were made that returned customers
to the store. .here was but one test for the success or failure of changes implemented>
they had to bring profits.
Give the Lady What She Wants Wendt and Hogan! +nd 6oo's $ublishers.
Deming said! E?our @ob as a leader is to manage the change necessary.F What does
that mean from a strategic planning point of view?
<ithout "ood strate"ic plannin" and deployment, there is short9term achievement at the e;pense
o# lon"9term or"ani=ational health Ceaders must create strate"ic plans that ali"n actions with
the critical success #actors as identi#ied by the customers $hey must wor% toward total system
ali"nment, ali"nin" the or"ani=ation-s vision, mission, strate"ic ob?ectives, processes, and day9
to9day activities with their customers- needs, wants, and e;pectations +##ective or"ani=ations
must chan"e #aster than their competitors and the e;ternal mar%et environment in order to "ain
and sustain a competitive advanta"e +##ective leaders use the strate"ic plan to provide clarity
,n the comple;ity o# day9to9day operations, it is easy #or members o# the or"ani=ation to become
distracted and lose si"ht o# the strate"ic "oals and ob?ectives +##ective leaders continually re9
emphasi=e the ultimate tar"ets established in the strate"ic plan
0.
Chapter , -uman Resource Development an* .anagement
1. Why won%t an individual%s best efforts necessarily ensure *uality?
@odern manu#acturin" methods and the comple;ities o# today-s wide variety o# services prevent
many employees #rom havin" direct contact with the ultimate end user o# their or"ani=ation-s
product or service &or this reason, employees cannot meet the needs and e;pectations o# their
ultimate customers without support #rom all areas o# the or"ani=ation Jaluable customer
in#ormation must be made available to employees in a #ormat that is use#ul to them +mployees
must be able to see the correlation between their day9to9day activities and the value o# their
product or service as perceived by the ultimate customer ,n order to create customer value,
employees must be #amiliar with how meetin" the Buality, cost, and schedule reBuirements o#
their ?ob impacts customer value perception, pro#its, and ultimately their #uture employment
$his means that re"ular communication must ta%e place between leadership and all employees
+mployees at all levels must be involved in decisions that a##ect their ?obs ,# they cannot ?ud"e
how their activities impact the customer, they cannot be e##ective in increasin" customer
satis#action and value perception
2. :r. :arshall -ield was the premiere department store ownerAmanager during the late
13//s. .oday! due in part to the fact that his stores still follow his management beliefs!
:arshall -ield%s stores remain very successful. .he following *uote comes from
:arshall -ield%s boo' Give the Lady What She Wants! written in 1)//! based on his
e"periences.
:r. -ield! while strolling through his store! came upon his assistant retail manager
arguing heatedly with a woman customer.
EWhat are you doing here?F as'ed -ield.
E& am settling a complaint!F replied the manager.
E<o! you%re not!F snapped -ield. E7ive the lady what she wantsBF
4ater he sent the following memo to all employees>
EWe wish!F he advised all employees! old and new! Eto ma'e important the point
of receiving merchandise that has been returned! without going through the
unnecessary or unpleasant *uestions or remar's! which ma'e the customer ill at
ease and really accomplish no good. We would much prefer to be occasionally
imposed upon than to feel that we have made any errors in the other direction. .he
employee is not the loser by the returning of goods! e"cept in a very slight way. .he
house is the loser! and if we prefer to stand the loss cheerfully! please let us have
corresponding cheerfulness from employees.F
Give the Lady What She Wants Wendt and Hogan! +nd 6oo's $ublishers.
0ow does this e"perience relate to what you learned about the Wor'force -ocus
:6<,+ riteria?
$he @alcolm 8aldri"e National :uality !ward criteria section 4) <or%#orce &ocus, e;amines
how an e##ective or"ani=ation encoura"es its wor%#orce to develop to its #ullest potential $his
section o# the award concentrates on how the e##orts o# the wor%#orce must be ali"ned with the
07
or"ani=ation-s overall ob?ectives @arshall &ield mana"ed by wal%in" around, encoura"in" his
wor%#orce to not lose si"ht o# the customer His hands9on approach to mana"ement enabled him
to %eep in touch with his wor%#orce and their needs, allowin" him to #ind ways to help them
reach their #ull potential He helped his or"ani=ation achieve a hi"h level o# per#ormance
throu"h ?ob and compensation structure and wor%#orce practices His employees were %ey to his
business success because he reco"ni=ed that e##ective or"ani=ations have e##ective employees
His communications with his employees was part o# an in#ormal employee education and
trainin" pro"ram that built employee %nowled"e, s%ills, and capabilities He created a wor%
environment within an or"ani=ation that contributed to employee well9bein" and motivation
$hrou"h his actions and words, he tried to ensure that employee activities throu"hout his
or"ani=ation were ali"ned with the "oals and ob?ectives o# the or"ani=ation
3. &n the :alcolm 6aldrige <ational ,uality +ward! 1ection 8./! Wor'force -ocus!
companies are as'ed to> EDescribe how your organization%s wor' and @ob design!
compensation! career progression! and related wor' force practices enable employees
to achieve high performance in your operations.F &f you were a new manager of a hotel!
select one hotel area! describe it! and discuss how you would create alignment with
1ection 8./.
8e sure the answer discusses the @alcolm 8aldri"e National :uality !ward criteria section 4)
<or%#orce &ocus, which e;amines how an e##ective or"ani=ation encoura"es its wor%#orce to
develop to its #ullest potential $his section o# the award concentrates on how the e##orts o# the
wor%#orce must be ali"ned with the or"ani=ation-s overall ob?ectives !pplicants are as%ed to
describe how their or"ani=ation achieves a hi"h level o# per#ormance throu"h ?ob and
compensation structure and wor%#orce practices $he criteria reco"ni=es that e##ective
or"ani=ations have e##ective employees ,t as%s Buestions about employee education and
trainin" pro"rams that build employee %nowled"e, s%ills, and capabilities $he criteria studies
how the wor% environment within an or"ani=ation contributes to employee well9bein" and
motivation +mployee activities throu"hout an or"ani=ation must be ali"ned with the "oals and
ob?ectives o# the or"ani=ation
2. -rom a human resources point of view! what did .homas 6erry! author of Managing
the Total Quality Transformation! mean when he said> E.oo often management attempts
to implement strategy by describing attitudes! however! management needs to ta'e
steps to influence the actions employees ta'e.F
$oo o#ten, mana"ement tries to chan"e the culture by describin" the new attitudes it see%s but
#ails to ta%e the steps needed to in#luence the action employees ta%e !ctions, not words, are the
#oundation o# both a chan"e and a culture !n e##ective mana"er ma%es sure they have the
answers to the #ollowin" Buestions)
How does the reward system rein#orce behavior>
!re e;pectations ali"ned with results>
!re e;pectations and their ali"nment with or"ani=ational "oals and ob?ectives clearly
communicated>
06
Re"ardless o# the or"ani=ation, to be e##ective, people need to %now what their "oals and
ob?ectives are $hey need to %now how to proceed toward those "oals and ob?ectives $hey also
need to have the appropriate tools, trainin", and s%ills to move toward those "oals *urin" their
movement toward their "oals, they need to have a means o# %nowin" how well they are doin"
and they need to have a means #or ad?ustin" their wor% in process !ll o# this reBuires action and
communication #rom leadership Since customer needs and perceptions must drive the activities
in an e##ective or"ani=ation, internal employee cooperation and coordination must prevail ,t is
critical that corporate policies and pro"rams rein#orce the desired culture <hen creatin" or
modi#yin" reward systems, consideration should be "iven to)
9customer e;pectations
9?ob descriptions based on identi#ied customer e;pectations
9e;pected results
9per#ormance e;pectations includin" behavioral s%ills
Reward systems should reco"ni=e achievement !s employees enable an or"ani=ation to reach
its "oals and ob?ectives, they should be rewarded #or their e##orts Since they are the direct
recipients o# rewards, employees should play an active role in creatin" or modi#yin" a reward
system $o ensure #airness, the reward system should employ a ratin" scale that accurately
re#lects actual employee per#ormance, whether in a team or individually
8. 1elect one of Deming%s fourteen points and discuss how it relates to human
resources management.
Could discuss)
Create a constancy o# purpose 'only leadership can create and en#orce a #ocus on the customer(
!dopt a new philosophy 'leadership sets the direction o# the or"ani=ationA leaders create
ali"nment #or their employees(
Constantly and #orever improve 'leadership is responsible #or "uidin" the or"ani=ation-s #ocus on
customerA the or"ani=ation-s processes directly a##ectin" the customer(
,nstitute leadership 'mana"in" with a %nowled"e o# variationA creatin" a #ocus on customer(
8rea% down barriers between departments 'processes should be seamless to the customerA
everybody comes to wor% to do a "ood ?obA #i; the processes so that they can do a "ood ?ob(
+liminate slo"ans, etc 'institute leadership(
+liminate arbitrary wor% standards, etc 'institute leadership(
Remove barriers that rob people 'everybody comes to wor% to do a "ood ?obA #i; the processes so
that they can do a "ood ?ob(
,nstitute a vi"orous pro"ram o# education and sel#9improvement 'never allow the wor%#orce to
become sta"nantA help them improve themselves(
6. E&t ta'es personal involvement to understand and remove the barriers that stand
between current capabilities and improvement.F W. (dwards Deming 0ow does this
statement relate to human resources?
05
Chan"in" conditions in the mar%etplace o#ten place pressure on or"ani=ations to ma%e
modi#ications to the way they do business Ceadership must communicate to the employees the
desired chan"e and motivate individuals to ma%e the chan"e People resist chan"e because
humans are control9oriented and when their environments are disrupted, they perceive that they
have lost the ability to control their lives Ceaders need to understand that resistance to chan"e is
a natural human reaction +##ective leaders %now that in order #or chan"e to occur, resistance to
those chan"es must be e;pected and planned #or +ssentially, they mana"e chan"e by mana"in"
resistance Ceaders must clearly communicate the new e;pectations and the reasons behind the
chan"es so that people understand why they are bein" as%ed to chan"e $heir employees are
loo%in" to them and as%in" the Buestion) D<hat do you want me to do tomorrow that is di##erent
#rom what , am doin" today>E +##ective leaders provide trainin" and time to ma%e the chan"e
$hey also structure their reward system to support the chan"e @a%in" chan"e in any
or"ani=ation can be di##icult, i# not impossible without the involvement and cooperation o# the
employees ,t only ta%es moments to proclaim a new culture or a new method, but it ta%es a "reat
deal lon"er to "et people to act di##erently +##ective leaders reali=e that employees mold their
behavior accordin" to their interpretations o# the si"nals leadership sends them $hese si"nals
may come #rom policies, reBuests, edicts, or #rom the day9to9day actions ta%en by leadership
+##ective leaders reco"ni=e that actions spea% louder than words Communicatin" throu"h
leadership actions and e;amples is paramount to chan"in" behavior +##ective leaders reco"ni=e
that the tools need to be in place to support the desired chan"e $o ma;imi=e the chan"e process,
e##ective leaders ensure that the wor%erFmachineFcomputer inter#ace, as well as, the wor%er9to9
wor%er inter#ace, is compatible with the needs, capabilities, and limitations o# the wor%er
&urther, e##ective leaders ensure that the reward system matches the desired e;pectations in order
to chan"e behavior !li"nment must e;ist between rewards, e;pectations, leadership actions,
and customer needs 8oth rewards and punishments must rein#orce behavior e;pectations
,nterestin"ly enou"h, many people do not %now what is involved in ma%in" a chan"e
=. .he Wor'force -ocus section of the :6<,+ emphasizes wor' systems and
employee learning and motivation. 0ow do these paragraphs about :. -ield and his
store apply to these sections? 6e sure your answer ties directly to the criteria.
+lthough -ield! li'e other e"ecutives! frowned on unions! he believed in trying to 'eep
wor'ers reasonably happy. Wages were not high! but many preferred to wor' for a
lower salary at -ield%s because! as one of the wor'ers later stated! E?ou were
considered low class if you cler'ed in some other store. &f you wor'ed at -ield%s it was
all right.F
(mployees were treated with great care. :arshall -ield%s o. opened for them a
special restaurant! recreation rooms! a gymnasium! loc'er rooms with separate shower
baths! and a library. 0e devised a three;day training system whereby new wor'ers
were taught the ways of the big store! how to display goods! how to ma'e out sales
slips! and! most important of all! how to deal with customers.
(ven in their dealings with fellow;employees! -ield%s wor'ers were advised to be
circumspect. .hey were to use Edignity! respect and careF in addressing each other
while on duty.
Give the Lady What She Wants Wendt and Hogan! +nd 6oo's $ublishers.
04
8e sure the answer discusses the @alcolm 8aldri"e National :uality !ward criteria section 4)
<or%#orce &ocus, which e;amines how an e##ective or"ani=ation encoura"es its wor%#orce to
develop to its #ullest potential $his section o# the award concentrates on how the e##orts o# the
wor%#orce must be ali"ned with the or"ani=ation-s overall ob?ectives !pplicants are as%ed to
describe how their or"ani=ation achieves a hi"h level o# per#ormance throu"h ?ob and
compensation structure and wor%#orce practices $he criteria reco"ni=es that e##ective
or"ani=ations have e##ective employees ,t as%s Buestions about employee education and
trainin" pro"rams that build employee %nowled"e, s%ills, and capabilities $he criteria studies
how the wor% environment within an or"ani=ation contributes to employee well9bein" and
motivation +mployee activities throu"hout an or"ani=ation must be ali"ned with the "oals and
ob?ectives o# the or"ani=ation
03
Chapter / .anaging the Supply Chain
1. -or a company that you are familiar with! describe or map their supply chains.
2. Describe a supply chain to someone who is unfamiliar with the concept.
! supply chain is the networ% o# or"ani=ations involved in the movement o# materials,
in#ormation, and money as raw materials #low #rom their source throu"h production until they
are delivered as a #inished product or service to the #inal customer &or instance, a supermar%et
will place orders 'trans#er in#ormation( with their produce, meat, and mil% distributors $his
order in#ormation will be trans#erred #rom the distributors to the processin" plants and on to the
#armers $hese materials 'produce, meat, and mil%( will #low #rom their source throu"h
production 'the processin" plants( and on to the store shelves $he supermar%et will pay the
suppliers 'distributors(, and the distributors will pay their suppliers '#armers(
3. Discuss why an effective organization would develop an effective supply chain
with care. What benefits would they receive?
+##ective supply chains optimi=e the value creation process by improvin" processes,
implementin" lean principles, optimi=in" asset allocation, and monitorin" Buality assurance
2. -or a company that you are familiar with! describe the types of information and
material that flow bac' and forth through the supply chain.
&or instance, a supermar%et will place orders 'trans#er in#ormation( with their produce, meat, and
mil% distributors $his order in#ormation will be trans#erred #rom the distributors to the
processin" plants and on to the #armers $hese materials 'produce, meat, and mil%( will #low
#rom their source throu"h production 'the processin" plants( and on to the store shelves $he
supermar%et will pay the suppliers 'distributors(, and the distributors will pay their suppliers
'#armers(
8. Why is information so critical to a well;designed supply chain?
Or"ani=ations need to %now what is needed, how much, and when <ithout this in#ormation the
wron" thin"s will be ordered in the wron" Buantities ,n#ormation enables an e##ective
or"ani=ation to provide #or their customers- reBuirements
6. 0ow does e;commerce support supply chain management?
&aster purchasin" cycle times
Reduced inventory
&ewer order processin" people
&aster product or service search and orderin"
Greater in#ormation availability
!utomated validation and pre9approved spendin"
02
Cess printed material li%e order #orms or invoices
,ncreased #le;ibility
On9line communication
=. Why is purchasing so 'ey to effective supply chains?
Purchases are driven by several #actors, includin" Buality o# the "oods or services, timin" o# the
deliveries o# the "oods or services, and the costs associated with purchasin" the "oods or
services Purchasin" plays a %ey role in e##ective supply chains because they are the department
that has the closest contact with the suppliers in the supply chain $hey ma%e %ey decisions
a##ectin" supply chain per#ormance
3. 0ow does logistics differ from supply chain management?
Co"istics is the process o# determinin" the best methods o# procurin", maintainin", pac%a"in",
transportin", and storin" materials and personnel in order to satis#y customer demand Supply
chains are supported by the lo"istical activities o# the individual or"ani=ations in the overall
supply chain &or instance, at the supermar%et, leadership may have created teams to deal with
the lo"istical issues o# the produce procurement process in order to ma%e chan"es to purchase
more or"anic #ruits and ve"etables !s they ma%e their internal lo"istical chan"es 'maybe a
chan"e in #orms(, the or"ani=ations they wor% with will also be ma%in" internal lo"istical
chan"es to ali"n their or"ani=ation with their customer 'maybe by #indin" a "reater number o#
or"anic #arms to buy #rom(
). What challenges face the development of an effective supply chain?
Synchroni=ation
&luctuations in demand
Con" supply chains
Great distances between supply chain members
Cac% o# communication
Cac% o# in#ormation sharin"
01
Chapter 0 .easures of Organi#ational Success
1. What did Deming mean when he said> E-igures are only useful if we 'now how to use
themG they are no use on their ownF? 7ive an e"ample to support your conclusions.
Jery simply, what "ets measured "ets done +##ective or"ani=ations reco"ni=e that i# they
cannot measure, they cannot mana"e $hey %now that i# they do not have su##icient in#ormation
about a process, product, or service, they cannot control it !nd i# a process cannot be
controlled, the or"ani=ation is at the mercy o# chance So, e##ective or"ani=ations choose their
measures wisely and then use the in#ormation they provide to ma%e decisions
2. + local financial institution considers itself a full;service ban'. .hey offer a number of
products and services to individual customers! businesses! trusts! and credit card
customers. 9ecently! the ban' completed an internal *uality improvement process that
included the development of a mission statement and resolving issues that prevented
employees from performing to the best of their ability. +t this time! they also evaluated
e"ternal customer service. .he ban' wishes to monitor their performance using the
balanced scorecard approach. -or this ban'! develop two measures in each of the
following areas> -inancial! &nternal! ustomer! and 4earning and 7rowth.
&inancial) pro#it, return on investment, asset "rowth
,nternal) account errors, employee retention, new service o##erin"s
Customer) customer retention, customer complaints, new accounts, people ta%in" advanta"e o#
new service o##erin"s
Cearnin" and Growth) employee trainin" completed, new eBuipment implementation, process
improvement e##orts
6 +ssume that you are a hotel manager. ?ou would li'e to monitor the hotel%s performance
using the 6alanced 1corecard approach. -or the hotel! develop two measures of performance
in each of the following areas> -inancial! &nternal! ustomer! 4earning and 7rowth.
&inancial) pro#it, return on investment, room occupancy rates
,nternal) bill errors, employee retention
Customer) customer retention, customer complaints, new customers, #ree rooms "iven due to
errors
Cearnin" and Growth) employee trainin" completed, new eBuipment implementation, process
improvement e##orts
2. Describe three 'inds of information that a hotel would need in order to improve
customer service. #nce they had this information! how could it be put to use? &n other
words! who should share in this information and how would this information be used to
improve the way the hotel does business?
Customer complaint records, customer return records, customer needs, wants, e;pectations '#ree
internet lin%s, in room re#ri"erators, ease o# chec%9in, etc(
00
$his in#ormation can be put to use to improve processes #rom a customer point9o#9view !nyone
who has contact with a customer or ta%es part in a process that a##ects the customer needs to have
access to this in#ormation
8. 1elect one of Deming%s fourteen points and discuss how it relates to measuring
organizational performance.
Could discuss)
Create a constancy o# purpose 'measurements can show whether or not there is a #ocus on the
customer(
!dopt a new philosophy 'measurements can show the direction an or"ani=ation is headin"(
Constantly and #orever improve 'measurements let users %now whether or not the or"ani=ation is
improvin", especially those processes directly a##ectin" the customer(
,nstitute leadership 'measures allow leaders to mana"e with a %nowled"e o# variation, creatin" a
#ocus on customer(
8rea% down barriers between departments 'measures can "uide process improvement e##orts(
+liminate slo"ans, etc 'measures enable an or"ani=ation to determine what needs to be done(
+liminate arbitrary wor% standards, etc 'measures enable an or"ani=ation to determine what
needs to be done(
Remove barriers that rob people 'measures can "uide process improvement e##orts(
6. .he :easurement! +nalysis! and Hnowledge :anagement section of the :6<,+
emphasizes measurement and analysis of organizational performance and information
and 'nowledge management. 0ow does this paragraph about :. -ield and his store
apply to these sections? 6e sure your answer ties directly to the criteria.
1elfridge started with -ield%s in 13=)! rising from cler' to retail general manager. During
his first 1/ years at -ield%s! he made many changes including adding telephones!
improved lighting! greeters to assist people in and out of their carriages! a Ebudget
floor!F and revolutionary shelf displays. .here would be but one test for the success or
failure of his schemes> they had to bring profits.
&n 133)! 1elfridge confronted -ield and as'ed to be made a partner. -ield was startled!
for never in the history of his firm had anyone dared to utter such a demand. ?et! as he
reflected on this! he realized that during 1elfridge%s tenure as retail general manager!
annual sales had risen from M2!///!/// to M6!=//!/// and the profits from M3=/!/// to
M8=/!///. 1elfridge%s idea for the Ebudget floorF added M2!8//!/// in sales yearly.
Give the Lady What She Wants Wendt and Hogan! +nd 6oo's $ublishers.
$he @alcolm 8aldri"e National :uality !ward criteria encoura"es or"ani=ations to ta%e a
care#ul loo% at their %nowled"e mana"ement systems, particularly the measures o# per#ormance
used as indicators o# process improvement ,n Section 5, @easurement, !nalysis, and
Knowled"e @ana"ement, or"ani=ations are as%ed to describe how their per#ormance
measurement system operates $hey are as%ed to provide details about how their or"ani=ation
"athers, analy=es, and uses per#ormance data and in#ormation &ield used #inancial in#ormation
.//
to "uide chan"es to the or"ani=ation $his enabled him to understand, ali"n, and improve
per#ormance at all levels and in all parts o# the or"ani=ation He analy=ed per#ormance data and
in#ormation in order to assess and understand overall or"ani=ational per#ormance He
understood that %nowled"e mana"ement and in#ormation analysis are crucial #or creatin" and
maintainin" an e##ective or"ani=ation with an unwaverin" #ocus on the customer $hou"h this
e;ample does not elaborate about other measures, "atherin" and analy=in" in#ormation should
not be limited to one area o# the or"ani=ation or one type o# measurement
./.
Chapter "1 Benchmarking
1. What does an organization do during benchmar'ing?
U 8enchmar%in"
P *urin" the benchmar%in" process, a company compares its per#ormance a"ainst a
set o# standards or a"ainst the per#ormance o# a best9in9class company <ith the
in#ormation provided by the comparison, a company can determine how to
improve
2. What is benchmar'ing?
P ,t is a tool that will)
U Compare and measure your business per#ormance a"ainst world class
companies
U Hi"hli"ht areas where your business per#ormance is sub9optimal
U Hi"hli"ht worldwide best practices that lead to superior per#ormance
U Help to continuously improve your per#ormance
U ,ntroduce new ideas
3. Describe to your organization%s leadership why they should underta'e
benchmar'ing. Describe the ma@or benefits of benchmar'ing.
P ,denti#ies opportunities #or improvement
P @otivates action by e;ternal e;ample o# e;cellence
P Clari#ies current status
P Stimulates chan"e
P ,denti#ies stren"ths and assets
P Provides an ur"ency #or chan"e
P Uncovers new ways o# improvin" processes
P Promotes improvement
2. Describe the steps in benchmar'ing.
P *etermine the #ocus
P Understand your or"ani=ation
P *etermine what to measure
P *etermine whom or what to benchmar% a"ainst
P 8enchmar%
P ,mprove per#ormanceV
8. Discuss two reasons why an organization would be interested in benchmar'ing
against the :alcolm 6aldrige <ational ,uality +ward.
$he award sets standards to be used as baselines and benchmar%s #or total Buality mana"ement in
seven areas $he seven cate"ories are leadershipA strate"ic plannin"A customer and mar%et #ocusA
./7
measurement, analysis, and %nowled"e mana"ementA wor%#orce #ocusA process mana"ementA and
business results Knowin" their per#ormance in these seven areas can help an or"ani=ation
understand where their stren"ths and wea%nesses lie <ith this in#ormation, they can improve
their competitiveness in the mar%et
Companies #ollowin" the "uidelines set out in the @8N:!C can improve their overall business
per#ormance ,t is interestin" to note that the 8aldri"e award winners, when their stoc%
per#ormance is reviewed, out9per#orm the stoc%s in the Standard and Poor-s 4// ,nde; by a ratio
o# about 6 to . &or more in#ormation about the 8aldri"e stoc% study, visit the #ollowin"
website) wwwnist"ovFpublic_a##airsF#actsheetFstoc%studyhtm
./6
Chapter "" Process .anagement
1. Describe the difference between a system and a process.
! process ta%es inputs and per#orms value9added activities on those inputs to create an output
! system is more comple;, involvin" the input #rom several processes
2. What does Reduce Variation! Deming%s summarized message! mean?
Companies interested in providin" a Buality product or service use statistical process control
techniBues to care#ully study the variation present in their processes *eterminin" the reasons
why di##erences e;ist between similar products or services and then removin" the causes o# these
di##erences #rom the processes that produce them enable a company to more consistently provide
a hi"h Buality product or service $hin% o# it this way) ,# you are carpoolin" with an individual
who is sometimes late, sometimes early, and sometimes on time, it is di##icult to plan when you
should be ready to leave ,#, however, the person is always #ive minutes late, you may not li%e it,
but you can plan around it $he #irst person e;hibits a lot o# variation, you never %now when to
e;pect him or her $he second person, althou"h late, has very little variation in his or her
processA hence you %now that i# you need to leave at e;actly 4 pm, you had better tell that
person to be ready at 5)44 $he best situation would be to be on time every time
3. Why is it a good idea to create process flow charts?
,n most or"ani=ations, very #ew people truly understand the myriad o# activities in a process that
it ta%es to create a product or service Process maps are power#ul communication tools that
provide a clear understandin" o# how business is conducted within the or"ani=ation ,denti#yin"
and writin" down the process in pictorial #orm helps people understand ?ust how they do the
wor% that they do Process maps have the ability to accurately portray current operations and can
be used to evaluate these operations ! process map also identi#ies the activities that have been
added to the process over time in order to adapt older processes to chan"es in the business Once
chan"es have been proposed, process maps are eBually power#ul #or communicatin" the
proposed chan"es to the process
2. Why does mapping a process from the customer%s point of view ma'e a company
more effective?
Process improvement e##orts should concentrate on what is important to the customer, whether
that customer is the ne;t wor%station in the line or the ultimate end user o# the product or service
<ithout this #ocus, an or"ani=ation can "o about improvin" processes without ever improvin"
the ri"ht processes, the ones the customer cares about ,# the customer doesn-t care about it,
chances are improvement e##orts will have no e##ect on the or"ani=ation-s bottom line because
they are not related to creatin" success#ul customers who return
8. What is meant by the term 'ey process? 5sing a hotel as an e"ample! describe two
of their 'ey processes.
./5
Key processes are the business processes that have the "reatest impact on customers- value
perceptions about the product or service and the "reatest impact on customer retention +##ective
or"ani=ations concentrate system and process improvement e##orts on those business processes
that will increase their competitiveness
Chec%9in, chec%9out, room service, bill preparation, cleanin" o# rooms, etc

6. 0ow can 'ey processes be identified?
Study the or"ani=ation #rom the customer-s point9o#9view ,denti#y those processes that have the
"reatest impact on the customers- value perceptions about the product or service and the "reatest
impact on customer retention
=. +ssume you wor' for a bicycle manufacturing company. &n a meeting! your
colleagues are arguing about who is responsible for a particular problem and where the
problem originated. ?ou would li'e to propose the use of a process flow diagram to
help alleviate the arguing. Why would a process flow diagram be such a useful tool in
this circumstance?
Processes are improved throu"h value9added process mappin", problem isolation, root cause
analysis, and problem resolution @any processes develop over time, with little concern #or
whether or not it is the most e##ective manner in which to provide a product or service $o
remain competitive in the world mar%etplace, companies must see% out waste#ul processes and
improve them $he processes providin" the products and services will need to be improved with
the aim o# preventin" de#ects and increasin" productivity by reducin" process cycle times and
eliminatin" waste $he %ey to re#inin" processes is concentratin" on the process #rom the
customer-s point9o#9view and identi#yin" and eliminatin" non9value added activities Process
maps enable or"ani=ations to do all o# the above
Process improvement e##orts should concentrate on what is important to the customer, whether
that customer is the ne;t wor%station in the line or the ultimate end user o# the product or service
<ithout this #ocus, an or"ani=ation can "o about improvin" processes without ever improvin"
the ri"ht processes, the ones the customer cares about ,# the customer doesn-t care about it,
chances are improvement e##orts will have no e##ect on the or"ani=ation-s bottom line because
they are not related to creatin" success#ul customers who return Process maps enable
or"ani=ations to study the process #rom the customers- point9o#9view
3. +ssume you wor' for a bicycle manufacturing company. 0ow would you convince
your colleagues at the bicycle company to utilize process flow diagrams to diagram their
processes? .o aid in their understanding! give two e"amples of processes that might
be found at a bicycle manufacturing company to support your description. Who would
be the process owners in the e"amples you used above?
See answer to W2
Process) !ssembly
Process owner) @anu#acturin", speci#ically the manu#acturin" en"ineer in char"e o# the line
./4
Process) Customer order ta%in"
Process owner) @ar%etin" or sales, the individual who spea%s with the customer about their
order
Process) Purchasin" o# raw materials
Process owner) Purchasin" a"ent
). Describe what is meant by process ownership.
Process ownership re#ers to identi#yin" who is ultimately responsible #or seein" that the process is
completed in a manner that results in customer satis#action $hese individuals are in a position to and
have the power to ma%e chan"es to the process
1/. .he $rocess :anagement -ocus section of the :6<,+ emphasizes the 'ey
aspects of an organization%s process management! including 'ey product! service! and
business processes for creating customer and organizational value. 0ow do these
paragraphs about :. -ield and his store apply to these sections? 6e sure you answer
ties directly to the criteria.
:r. Dohn 1hedd had been instrumental bac' in 13)6 in persuading -ield to e"tend the
wor'room methods by which the firm was able to offer e"clusive products to its
customers. 1ome such wares had long been made in the shops on the top floors of the
retail store! and -ield%s also owned several small plants.
&r'ed by the fact that specifications for certain articles were not always met by
manufacturers! 1hedd had gone to -ield with a plan for final processing of goods in their
own factories. -ield had been hard to convince! but when 1hedd showed! by small;
scale e"periments with yarns and thread! that manufacturing could reduce costs by 1/
percent and also provide the company with e"clusive merchandise! he was given the
change to probe further into the possibilities.
Give the Lady What She Wants Wendt and Hogan! +nd 6oo's $ublishers.
$he @alcolm 8aldri"e National :uality !ward criteria section 3, Process @ana"ement, invites
or"ani=ations to e;amine their process mana"ement e##orts by loo%in" at their %ey product, service,
support, and business processes &ield and Shedd were able to study their current processes, measure
de#ects in the process, and use this in#ormation to ma%e improvements to their business processes $heir
e##ective or"ani=ation desi"ned and mana"ed these processes in a manner that created customer value and
enabled the or"ani=ation to achieve business success and "rowth !s this e;ample about Dprivate labelE
"oods shows, Shedd and &ield identi#ied and mana"ed their %ey processes $he criteria reBuire that
or"ani=ations use per#ormance measures #or the control and improvement o# processes $hey used
measures o# per#ormance to ma%e process improvements in order to achieve better per#ormance $hey
encoura"ed or"ani=ational ali"nment by emphasi=in" the use o# input #rom internal and e;ternal
customers when desi"nin" and improvin" processes
./3
Chapter "% )ean ools an* echni2ues
1. ?ou are about to interview for a full;time position with a company. 0ow will you
recognize that this company is following lean principles? 0ow do you 'now they
are? 6e sure to sight specific e"amples of the evidence you would loo' for.
Coo% to see i# they have any o# the #ollowin" systems in place) customer #ocus, value chain
mana"ement, active leadership, #ocus on business results, strate"ic plannin", in#ormation and
%nowled"e mana"ement, Buality assurance, or"ani=ational philosophy #ocused on Buality, value
en"ineerin", e##orts to eliminate waste, process improvement teams, human resources
mana"ement, ?ust9in9time operations, Kai=en events, #ive S-s, ta%t time calculations, sin"le
minute e;chan"e o# dies

2. 0ow does lean thin'ing generate process improvement?
Cean thin%in" "enerates process improvement by #ollowin" these #ive steps)
9Study the process by directly observin" the wor% activities, their connections and #low
9Study the process to systematically eliminate waste#ul activities, their connections and
#low
9+stablish a"reement amon" those a##ected by the process in terms o# what the process
needs to accomplish and how the process will accomplish it
9!ttac% and solve problems usin" a systematic method
9,nte"rate the above approach throu"hout the or"ani=ation
3. Why is it a good idea to create value stream process maps?
,n most or"ani=ations, very #ew people truly understand the myriad o# activities in a process that
it ta%es to create a product or service Process maps are power#ul communication tools that
provide a clear understandin" o# how business is conducted within the or"ani=ation ,denti#yin"
and writin" down the process in pictorial #orm helps people understand ?ust how they do the
wor% that they do Process maps have the ability to accurately portray current operations and can
be used to evaluate these operations ! process map also identi#ies the activities that have been
added to the process over time in order to adapt older processes to chan"es in the business Once
chan"es have been proposed, process maps are eBually power#ul #or communicatin" the
proposed chan"es to the process
2. Describe what happens at a Haizen event.
!t a Kai=en event, people study a process and loo% #or ways to eliminate waste while
standardi=in" processes $his means that people #ollow the #low o# the process and study it #or
where people are doin" thin"s that can be eliminated, combined, or removed #rom the process
Kai=en events eventually want to "et down to the wor% that absolutely must be done in order to
produce the product or provide the service
8. Why do effective organizations do value stream process mapping on a process
from a customer%s point of view?
./2
Process improvement e##orts should concentrate on what is important to the customer, whether
that customer is the ne;t wor%station in the line or the ultimate end user o# the product or service
<ithout this #ocus, an or"ani=ation can "o about improvin" processes without ever improvin"
the ri"ht processes, the ones the customer cares about ,# the customer doesn-t care about it,
chances are improvement e##orts will have no e##ect on the or"ani=ation-s bottom line because
they are not related to creatin" success#ul customers who return
6. What are the five 1%s? Describe your own wor' area. What could you do to Efive
1F it?
Seiri 'separate( &or e;ample) ,n my wor% area, , would ta%e away the thin"s , don-t need, li%e
my C*s , would only %eep the printer, the computer unit, my wor%9holdin" device, and the
lamp
Seiton 'arran"ementForderliness( &or e;ample) ,n my wor% area, , would rearran"e the printer
and the wor% holdin" device so that the arran"ement had a better #low
Seiso 'cleanliness( &or e;ample) ,n my wor% area, , would dust it re"ularly and throw out
unnecessary scraps o# paper
Sei%etsu 'repeat 6 S-s re"ularly( &or e;ample) , would ma%e a point o# usin" the 6 S-s in my
wor% area every &riday a#ternoon
Shitsu%i 'discipline( &or e;ample) , would %eep up with the pro"ram and perhaps e;pand it to
other areas
=. What are the benefits of using a Hanban system?
! Kanban system improves process mana"ement by #ocusin" on visual control o# the process
People in the process can-t produce anythin" more than the Kanban cards allow them to $hese
cards are visual symbols #or what needs to be produced and what has been produced
3. 0ow does error;proofing relate to Dr. Deming%s point that states Eremove the
barriers that rob people of pride of wor'manshipF?
+rror proo#in" is another way to eliminate waste ,t involves #ive principles) elimination,
replacement, #acilitation, detection, and miti"ation +ach o# these three help to ma%e processes
and wor% simpler #or people $hey don-t have to wor% as hard, they don-t ma%e mista%es in their
?ob, and they produce the ri"ht thin"s #or the customer without any problems in it $his means
that they can do their ?ob well and have pride in their wor%manship +rror proo#in" allows this
to happen because it ta%es the potential #or ma%in" a mista%e out o# the ?ob
). What is .a't time? alculate the .a't time for an 3 hour day that has two 1/
minute brea's and one 3/ minute lunch. .he customer would li'e to have 1//
parts per day.
./1
$a%t time re#ers to how o#ten a sin"le part should be produced in order to meet customer
demand
23/ N 1/ N 1/ N 3/
.a't time O JJJJJJJJJJ O 2.3 minutes per part
1//
1/. Define line balancing.
Cine balancin" occurs when wor% is per#ormed by each operator evenly over time with no pea%s
or valleys +ach wor%er or machine on the assembly line has wor% that #ills the same amount o#
time, so no one or no machine is waitin" #or somethin" to do or is havin" to rush to %eep up
./0
Chapter "& Pro3lem4Solving ools an* echni2ues
1. Why would an organization be interested in reducing the variation present in a
process?
Jariation means that no two products or services are e;actly ali%e Customers want their
products and services to be consistent ,# variation is present in a process, they won-t be 8y
removin" the variation present in the process, or"ani=ations can provide customers with more
consistent products and services Processes e;hibitin" little variation are predictable and stable
<hen a process is predictable and stable, process owners have a rational basis #or plannin" the
#uture Jariation is an indication o# a problem
2. 0ospitals are very interested in improving health care processes. :any hospitals
have begun to implement statistical process control techni*ues in order to monitor and
improve their processes. #ne such hospital is using P;bar and 9 charts to monitor the
turnaround time for stress tests. .urnaround time refers to the amount of time that
elapses between the patient ta'ing the test and the doctor receiving the results of the
test. 5se the following information to calculate the control limits and centerline for an P;
bar and 9 chart. n O 6.
P;bar 9
)3 33
)3 2)
)= 23
)= 32
=/ 23
6/ 2/
3) 3/
=/ 23
6/ 2/
83 1)
8= 1)
28 18
63 23
=/ 23
8= 1)
6) 22
83 2/
88 13
8/ 16
88 13
JJJJJJJJ
1331 282
O*8 N 30
R9bar N 76
../
UCC; N 30 H /516'76( N 1/
CCC; N 30 P /516'76( N 41
UCCr N 7//5'76( N 53
CCCs N /'76( N /
2. 1tress tests are used to study the heart muscle after a person has had a heart
attac'. .he team investigating stress tests turnaround time has managed to reduce the
amount of time it ta'es for a doctor to receive the results of a stress test from 63 to 32
hours on average. .he team had a goal of reducing test turnaround times to between
3/ and 36 hours. 7iven that the new average for test turnaround time is 32 hours! with
a standard deviation of s;bar O 1! and n O )! calculate and interpret p and p'. Wor'
this problem to whole numbers.
s9bar .
X N LLLL N LLLL N .
c
5
/306
63 P 6/
Cp N LLLL N ./
3'.(
7
Cp% N LLL N /33
6
<here)
63 P 67
Q'u( N LLLL N 5
.
67 P 6/
Q'l( N LLLLL N 7
.
$he process is not centered and ?ust barely capable
...
3. reate a cause and effect diagram for the problem> $eople wait too long for food
during the noon hour at a local fast food restaurant. omplete the diagram by
brainstorming potential corrective actions.
@aterial @ethod
&ood not available @ovement Coo% times too lon"
Coo% times too lon" $imes Pic%in" style
*istance
Number o# parts in order
,tems carriedFtrip
People
wait too
lon" #or
#ood
Sta##in"
Part timers Not enou"h
$ime o## space #or Not enou"h cash
people to re"isters
+;perience wor%
$rainin" Not enou"h coo%in"
eBuipment
People +Buipment
..7
2. &f you were the pro@ect manager responsible for solving a problem! based on
Deming%s $D1+ cycle! describe the steps you would ta'e to plan a problem
investigation.
, would #ollow these steps)
a Reco"ni=e the problem, establish priorities
b &orm a team
c *e#ine the problem
d *evelop per#ormance measures
e !naly=e the problemFprocess usin" root cause analysis tools
# *etermine possible causes
" Select and implement a solution
h +valuate the solution and institute #ollow9up i# necessary
i +nsure permanence o# the solution
8. What does using a structured problem;solving process do for us?
! well9structured problem9solvin" process aids in the isolation and analysis o# a problem and the
development o# a permanent solution Problem solvin" should be lo"ical and systematic
6. Why is the use and interpretation of an 9 chart so important when studying a
process? What does the P;bar chart show? What does the 9 chart show?
$he use and interpretation o# an R or s chart is critical when e;aminin" an O9bar chart because
the R or s charts allow the user to see the spread o# the data $he charts show i# the spread o# the
data is reasonable #or the measurements on the O9bar charts $he R or s charts show the
repeatability o# the process
=. 0otels often monitor the services they provide with P;bar and 9 charts. .he following
information comes from customer satisfaction data related to brea'fast. (ach point
represents one wee' of scores. .he average given is for a sample size n O = Kone wee'
of brea'fastsL. + 1// is the highest possible score! on a scale of 1;1//. reate an P;
bar and 9 chart with the following information.
31 8
3/ 3
=) 2
31 6
3/ 2
32 2
31 3
33 2
32 2
33 8
31 3
..6
32 2
3/ 2
3/ 2
31 8
33 3
32 6
33 2
32 6
31 8
162= 3/
O*8 N 1.
R9bar N 5
UCC; N 1. H /5.0'5( N 16
CCC; N 1. P /5.0'5( N 20
UCCr N .075'5( N 1
CCCr N //23'5( N /6
..5
Chapter "' Pro5ect .anagement
1. 0ow would you recognize that a pro@ect is being managed well? What clues would
you loo' for?
<ell9mana"ed pro?ects meet per#ormance, timin", and cost ob?ectives with a minimum amount
o# hassle <ell9mana"ed pro?ects are selected because o# their ability to support the overall
"oals and ob?ectives o# an or"ani=ation and move the or"ani=ation toward its vision and mission
<ithout pro?ects, or"ani=ations lac% a lon" term #ocus and #ocus solely on doin" day9to9day
activities Conversely, poorly mana"ed pro?ects are costly #or an or"ani=ation ! pro?ect that
does not achieve its "oals and ob?ectives, or one that is not ali"ned with the "oals and ob?ectives
o# the or"ani=ation, wastes time, money, and other resources @isdirected pro?ects, poorly
planned pro?ects, or poorly "uided pro?ects are demorali=in" #or the or"ani=ation-s members
Care must be ta%en to ensure that pro?ects are appropriately selected to support the or"ani=ation-s
mission and "oals Once selected, a pro?ect must be served well by its pro?ect mana"er and team
in order to ensure its success#ul completion
2. Why is it critical to manage a pro@ect well?
Poorly mana"ed pro?ects are costly #or an or"ani=ation ! pro?ect that does not achieve its "oals
and ob?ectives, or one that is not ali"ned with the "oals and ob?ectives o# the or"ani=ation, wastes
time, money, and other resources @isdirected pro?ects, poorly planned pro?ects, or poorly
"uided pro?ects are demorali=in" #or the or"ani=ation-s members Care must be ta%en to ensure
that pro?ects are appropriately selected to support the or"ani=ation-s mission and "oals Once
selected, a pro?ect must be served well by its pro?ect mana"er and team in order to ensure its
success#ul completion +##ectively mana"ed pro?ects ensure that time, cost, and per#ormance
"oals are met
3. What is involved in pro@ect control?
$hrou"hout a pro?ect, e##ective pro?ect mana"ers monitor the pro"ress a pro?ect is ma%in" toward
completion Per#ormance, cost, and time, the three aspects o# a pro?ect, all need to be monitored
and controlled in order to ensure pro?ect success Pro?ect control and monitorin" involves
"atherin" and appraisin" in#ormation on how the pro?ect-s activities compare with the pro?ect
plan !ctual pro"ress is trac%ed a"ainst the per#ormance measures established in the pro?ect
plan $hese per#ormance measures help a pro?ect mana"er assess how time, money, and other
resources have been used to produce the e;pected outcomes @any reasons e;ist why costs need
to be monitored durin" a pro?ect $echnical di##iculties may reBuire more resources than
ori"inally planned Client9related chan"es in speci#ications may have chan"ed the scope o# the
wor% si"ni#icantly, thus a##ectin" the total costs o# the pro?ect Or the bud"et may have been
inadeBuate in the #irst place due to inadeBuate estimates, poor pro?ections #or in#lation, or
additional costs due to client9related chan"es Costs can also "et out o# control when the pro?ect
costs are not watched closely and corrective cost control was not e;ercised in time 8y closely
monitorin" the per#ormance measures associated with the pro?ect, an alert pro?ect mana"er can
be prepared to respond Buic%ly to deviations in order to %eep the pro?ect on trac% and under
control $hou"h very #ew pro?ects have not had their "oals and ob?ectives modi#ied in some way
..4
or another #rom their be"innin" to the end, care#ul pro?ect control enables a pro?ect mana"er to
minimi=e the e##ects o# these chan"es on the overall pro?ect
2. What are the components of a sound pro@ect plan? Why is it worth ma'ing the effort
to develop a sound pro@ect plan?
Pro?ects have three interrelated ob?ectives) meetin" the bud"et, #inishin" on schedule, and
meetin" the per#ormance speci#ications set by the client Pro?ect plans provide the in#ormation
about)
9@ission and deliverables
9Speci#ic "oals and ob?ectives supportin" the mission and its deliverables
9$as%s reBuired to meet the "oals and ob?ectives
9$echnicalities o# who, what, where, when, why, and how
9Schedules, the time needed to support each aspect o# the plan
9Resources, what is needed to support each aspect o# the plan
9Cost analysis
9Jalue analysis
9Personnel, who is needed to support each aspect o# the plan
9Personnel, responsibilities and assi"nments
9+valuation measures #or %eepin" the pro?ect on trac%
9Ris% analysis, what could "o wron" and how will it be dealt with>
9Pro?ect chan"e mana"ement process
! "ood pro?ect plan enables an or"ani=ation to accomplish all three ob?ectives $hou"h a pro?ect
plan may be modi#ied several times durin" a pro?ect, e##ective pro?ect plans remain %ey to
or"ani=ational success because no pro?ect always "oes accordin" to plan, but well planned
pro?ects are less li%ely to "o astray
8. What do readers loo' for in a well;constructed pro@ect proposal?
!n e##ective pro?ect proposal provides readers with insi"ht into what needs to be accomplished
and how it will "et accomplished $hrou"h the use o# clearly stated mission, deliverables, "oals,
and ob?ectives associated with the pro?ect, proposals sell the pro?ect !s an introduction,
proposals provide bac%"round in#ormation about the need #or the pro?ect $hey contain a
description or overview o# the e;pectations o# the pro?ect, includin" details about the technical
aspects o# the pro?ect +ssential tas%s are outlined and delineated ! thorou"h pro?ect proposal
will contain in#ormation concernin" #inancial reBuirements, time constraints, and administrative
and lo"istical support #or the pro?ect ,n the proposal there is usually in#ormation about the %ey
individuals associated with the pro?ect, includin" the identity o# the pro?ect mana"er 8asic areas
o# per#ormance responsibility are assi"ned $entative schedules and bud"ets are established
$he #i"ure in the chapter "ives a brie# summary o# the typical components o# a pro?ect proposal
$he pro?ect proposal creates a "eneral understandin" o#)
9what is needed,
9what is "oin" to be done,
..3
9why it is "oin" to be done,
9who is "oin" to do it,
9when it will be done,
9where it will be done,
9how it will be done>
6. What are contingency plans? Why are they useful?
Pro?ects, due to their very nature, are comple; &or every pro?ect there is a ris% o# #ailure
Or"ani=in" a pro?ect into mana"eable sections can reduce the #ear o# #ailure as well as the
potential #or #ailure +##ective pro?ect mana"ers don-t #or"et to as%) what i#> $hey wor% with
their pro?ect team members to identi#y potential activities or events that may derail a pro?ect
Contin"ency plans are created to ensure that the pro?ect team is ready to handle potential
problems <hile all problems cannot be #oreseen, a pro?ect plan that includes contin"ency plans
%eeps the team #le;ible and aware that they may be as%ed to ma%e ad?ustments to their pro?ect
plan some time durin" its li#etime
+##ective pro?ect mana"ers reco"ni=e that clients ma%e chan"es to the pro?ect as it pro"resses
!ll pro?ect proposals and plans should contain a description o# how reBuests #or chan"es in the
pro?ect-s plan, bud"et, schedule, or per#ormance deliverables will be handled !n e##ective
chan"e control system will have steps in place that review the reBuested chan"es #or both content
and procedure and identi#y how the chan"e will impact the pro?ect $his impact must be
re#lected in ad?ustments to the pro?ect-s per#ormance ob?ectives, the schedule, and bud"et Once
accepted, chan"e orders become part o# the overall pro?ect plan
=. Why are measures of performance critical to pro@ect evaluation and control?
Pro?ect control and monitorin" involves "atherin" and appraisin" in#ormation on how the
pro?ect-s activities compare with the pro?ect plan !ctual pro"ress is trac%ed a"ainst the
per#ormance measures established in the pro?ect plan $hese per#ormance measures help a
pro?ect mana"er assess how time, money, and other resources have been used to produce the
e;pected outcomes Costs can "et out o# control when the pro?ect costs are not watched closely
and corrective cost control was not e;ercised in time 8y closely monitorin" the per#ormance
measures associated with the pro?ect, an alert pro?ect mana"er can be prepared to respond
Buic%ly to deviations in order to %eep the pro?ect on trac% and under control $hou"h very #ew
pro?ects have not had their "oals and ob?ectives modi#ied in some way or another #rom their
be"innin" to the end, care#ul pro?ect control enables a pro?ect mana"er to minimi=e the e##ects o#
these chan"es on the overall pro?ect
..2
Chapter "( Results
1. What did .homas 6erry! author of Managing the Total Quality Transformation! mean
when he said> E.oo often management attempts to implement strategy by describing
attitudes! however! management needs to ta'e steps to influence the actions
employees ta'e.F
@ana"ers o# e##ective or"ani=ations reco"ni=e the need to e;amine their overall business results
in order to determine whether or not the strate"y they have developed and the actions they have
ta%en to support that strate"y are helpin" the business prosper ,# they merely describe attitudes,
then no steps will be ta%en by the employees to act upon the strate"y +##ective mana"ers who
ta%e steps to in#luence actions ta%en by employees use current levels and trends #or %ey
per#ormance measures as indicators o# an or"ani=ation-s ability to achieve solid business results
in these critical areas
2. What does the term EalignmentF mean in relation to *uality management?
Studyin" business results, usin" measures o# per#ormance li%e those discussed in Chapter 1,
enables an or"ani=ation to con#irm whether or not they are ma%in" pro"ress toward their vision,
mission, "oals, and ob?ectives 8usiness results should indicate that the "oals and ob?ectives
described in the or"ani=ation-s strate"ic plan are not pipe dreams but achievable realities
!li"nment means that there is a connection between the customer-s needs, reBuirements, and
e;pectations and the day9to9day activities o# the or"ani=ation e;ists
3. 1elect one of Deming%s fourteen points and discuss how it relates to achieving overall
business success.
!ll o# *emin"-s #ourteen points relate to achievin" business success ,n their discussion, see i#
they use e;amples to e;plain how they are related
2. .he 6usiness 9esults -ocus section of the :6<,+ e"amines an organization%s
performance and improvement in 'ey business areas. .he following *uotes are from :.
-ield. &n today%s day and age! we are taught that there are other measures of
performance besides profits. What are the five other areas besides financial and
mar'etplace performance covered by the 6usiness 9esults section of the :6<,+?
$he criteria investi"ate the %ey business areas o# customer satis#action, product and service
per#ormance, #inancial and mar%etplace per#ormance, wor%#orce results, operational
per#ormance, and or"ani=ational "overnance and social responsibility

.here would be but one test for the success or failure of his schemes> they had to bring
profits.
1ometimes! -ield pointed a thic' finger at the firm%s ledgers and gave 1elfridge%s critics
a sharp answer> E0e may not be doing things to your li'ing! gentlemen. 6ut! he%s
ma'ing a lot of money for us. +re you?F
..1
E#ffice boy today! partner tomorrow.F
Give the Lady What She Wants Wendt and Hogan! +nd 6oo's $ublishers.
+##ective mana"ers and e##ective employees have an understandin" o# how the %ey ideas
presented in this te;t wor% to"ether to move the company #orward $hey use measures o#
per#ormance to monitor whether or not the actions they are ta%in" are stren"thenin" the
company People who use in#ormation e##ectively are Do##ice boys today, and partners
tomorrowE
8. ?ou are about to interview for a full;time @ob with a company of your choice. 0ow will
you recognize that this company is following a .,:Acontinuous improvement
philosophy? 0ow do you 'now they are? 6e sure to cite specific e"amples of the
evidence you would loo' forB
$hese characteristics will be present in an e##ective or"ani=ation)
9an or"ani=ational philosophy #ocused on customers
9a Buality mana"ement system
9a customer #ocus and satis#action
9or"ani=ational leadership
9a strate"ic plan
9a well9mana"ed human element
9measures o# per#ormance
9process improvement e##orts
9utili=in" Buality tools
9mana"in" pro?ects
9trac%in" %ey business results
,n an e##ective or"ani=ation the #ocus is on the %ey processes that provide the or"ani=ation-s
customers with products or services
6. Why is organizational effectiveness such an important concept when operating a
business?
,n today-s hi"hly competitive, "lobal mar%etplace, a company that e;cels is one that continually
strives to identi#y and #ocus on #actors critical to its customers and improve its processes in order
to provide the hi"hest Buality product or service possible $his tas% is #ar #rom simple
Companies have a variety o# strate"ies that can be used to enhance their position in the
mar%etplaceA teams, Buality assurance, ?ust9in9time, total Buality mana"ement, Si; Si"ma, lean
manu#acturin", and others Ci%e pieces o# a pu==le that won-t #all into place, many o# these
e##orts have not created an overall ali"nment o# an entire or"ani=ational system that #ocuses on
the #actors critical to the or"ani=ation-s success ,nstead, e##orts are dis?ointed pro"rams that #ail
to reach their #ull potential, "eneratin" success in some areas and disillusionment in others ,n
truth, what is needed is an or"ani=ation9wide approach that improves and enhances the whole
process o# providin" a product or service enablin" the or"ani=ation to e;ceed customer
e;pectations, each and every time
..0
=. 0ow would you verify that an organization is operating effectively?
$hese characteristics will be present in an e##ective or"ani=ation)
9an or"ani=ational philosophy #ocused on customers
9a Buality mana"ement system
9a customer #ocus and satis#action
9or"ani=ational leadership
9a strate"ic plan
9a well9mana"ed human element
9measures o# per#ormance
9process improvement e##orts
9utili=in" Buality tools
9mana"in" pro?ects
9trac%in" %ey business results
,n an e##ective or"ani=ation the #ocus is on the %ey processes that provide the or"ani=ation-s
customers with products or services
3. 0ow are audits used to @udge organizational effectiveness?
+##ective or"ani=ations will #ocus their auditin" e##orts to ensure that leadership and the strate"ic
plan have #ocused the or"ani=ation on improvin" and enhancin" the or"ani=ation-s customer and
mar%et #ocus, product and service per#ormance, #inancial per#ormance, human resource
development and mana"ement, process and operational per#ormance ,n other words, they will
use audits to chec% the or"ani=ation-s pro"ress toward improvin" their products and services
#rom the customers- point9o#9view
!udits can "au"e or"ani=ational e##ectiveness by analy=in" %ey measures or indicators o#
customer satis#action and dissatis#action !udits can help e##ective or"ani=ations care#ully study
results related to the products and services they o##er &inancial and mar%et results o#ten receive
the "reatest amount o# #ocus durin" audits ,n this arena, the levels and trends o# %ey measures
or indicators o# #inancial per#ormance may include such measures o# #inancial return and
economic value as revenues, pro#its, "rowth rates, earnin"s, return on investments !udits may
reveal trends o# %ey measures or indicators o# mar%etplace per#ormance, includin" mar%et share
or position, business "rowth, and new mar%ets per#ormance Human resource development and
mana"ement o#ten receives less emphasis than the other areas However, it is critical to
determine i# the human resource policies and practices o# the or"ani=ation have resulted in a
stron" or"ani=ation !udits can address this issue Process or operational audits should #ocus on
%ey value creation processes $hese measures that may be audited may include productivity
comparisons, cycle time reductions, supplier or partner per#ormance, and other appropriate
indicators o# or"ani=ational e##ectiveness and e##iciency Key support processes appropriate #or
auditin" should also be audited usin" productivity rates, cycle times, supplier or partner
per#ormance, and other measures o# e##ectiveness and e##iciency
). 0ow are audits conducted?
.7/
. (lan) $o be"in, those plannin" the audit need to identi#y its purpose or ob?ective ! statement
o# purpose clari#ies the #ocus o# the audit &ollowin" this, planners will need to identi#y the who,
what, where, when, why, and how related to the audit
9<ho is to be audited>
9<ho is to per#orm the audit>
9<hat does this audit hope to accomplish>
9<hat are the per#ormance measures>
9<here the audit will be conducted>
9<hen will the audit be conducted>
9<hy is the audit bein" conducted>
9,s the audit to ?ud"e con#ormance to standards>
9<hat are the critical standards>
9How will the audit be conducted>
$hose about to be audited should be in#ormed by an individual in a position o# authority Clear
statements o# the reasons behind the audit 'why(, the per#ormance measures 'what(, and the
procedures 'how( should be "iven to those bein" audited !n audit is a valuable tool #or
improvement ,t is important to determine how the results will be used and who will have access
to the results be#ore the auditin" process be"ins
7 Do) Usin" the in#ormation clari#ied in the plannin" phase, the audit is conducted O#ten an
introductory meetin" is held by the participants to discuss the scope, ob?ectives, schedule, and
paperwor% considerations !#ter the openin" meetin", e;aminers be"in the process o# reviewin"
the process, product, or system under study !uditors may reBuire access to in#ormation
concernin" Buality systems, eBuipment operation procedures, preventive maintenance records,
inspection histories, or plannin" documents !uditors may conduct interviews with those
involved in the process o# providin" a product or service !ny and all in#ormation related to the
area under study is critical #or the success o# the audit *urin" the review process, auditors
document their #indin"s $hese #indin"s are presented in a "eneral summary at a closin" meetin"
o# the participants <ithin a short period o# time, perhaps ./ to 7/ days, the auditors will
prepare a written report that documents their #indin"s, conclusions, and recommendations
6 Study) !udits provide in#ormation about the participant-s stren"ths, wea%nesses, and areas #or
improvement Upon receipt, the auditor-s report is read by the participants in the audit *urin"
this phase o# the audit cycle, they respond to the report and develop an action plan based on the
recommendations o# the auditors $his action plan should speci#y the actions, assi"n
responsibilities, and provide a time #rame reBuired to deal with the issues raised by the audit
5 Act) Once adopted, the action plan becomes the #ocus o# the improvement activities related to
the audited area !uditors and company administrators should #ollow up at predetermined
intervals to evaluate the status o# the continuous improvement action plan $his ensures that the
recommendations and conclusions reached by the auditors, and supported by an action plan,
assist the company in reachin" its continuous improvement "oals
.7.
1/. What would be your role as an auditor?
!uditors are the people who conduct the Plan9*o9Study9!ct cycle o# an audit
. (lan) $o be"in, those plannin" the audit need to identi#y its purpose or ob?ectives &ollowin"
this, planners will need to identi#y the who, what, where, when, why, and how related to the
audit $he auditor helps develop clear statements o# the reasons behind the audit 'why(, the
per#ormance measures 'what(, and the procedures 'how( should be "iven to those bein" audited
7 Do) Usin" the in#ormation clari#ied in the plannin" phase, the auditor conducts the audit
O#ten an introductory meetin" is held by the participants to discuss the scope, ob?ectives,
schedule, and paperwor% considerations !#ter the openin" meetin", auditors be"in reviewin"
the process, product, or system under study !uditors may reBuire access to in#ormation
concernin" Buality systems, eBuipment operation procedures, preventive maintenance records,
inspection histories, or plannin" documents !uditors may conduct interviews with those
involved in the process o# providin" a product or service !ny and all in#ormation related to the
area under study is critical #or the success o# the audit *urin" the review process, auditors
document their #indin"s $hese #indin"s are presented in a "eneral summary at a closin" meetin"
o# the participants <ithin a short period o# time, perhaps ./ to 7/ days, the auditors will
prepare a written report that documents their #indin"s, conclusions, and recommendations
6 Study) !udits provide in#ormation about the participant-s stren"ths, wea%nesses, and areas #or
improvement Upon receipt, the auditor-s report is read by the participants in the audit *urin"
this phase o# the audit cycle, they respond to the report and develop an action plan based on the
recommendations o# the auditors $his action plan should speci#y the actions, assi"n
responsibilities, and provide a time #rame reBuired to deal with the issues raised by the audit
$he auditor is available to answer any Buestions concernin" the #indin"s
5 Act) Once adopted, the action plan becomes the #ocus o# the improvement activities related to
the audited area !uditors and company administrators should #ollow up at predetermined
intervals to evaluate the status o# the continuous improvement action plan $his ensures that the
recommendations and conclusions reached by the auditors, and supported by an action plan,
assist the company in reachin" its continuous improvement "oals
11. ite an e"ample of an effective organization you are aware of. 0ow do you 'now it
is more effective than its competitors?
@a%e sure their answers tie in true evidence o# e##ectiveness throu"h comparisons with the
business results criteria o# the @8N:!C See answers to Chapter .) .6, .5, and .4 and
Chapter .7) .74, .73, and .72
.77

S-ar putea să vă placă și